You are on page 1of 84

C H A P T E R

3
DIFFERENTIABILITY

3.1 INTRODUCTION coefficient of the given function f(x) and is denoted


f (x). Thus, by definition,
Our first application of the concept of limit will be to the dy y
problem of determining the instantaneous rate of change = lim
of a function. Geometrically, this problem is equivalent dx x  0 x
to that of finding a tangent line to the graph of the f(x  x)  f(x)
f (x) = lim ...(3)
function. Both of these problems are solved by finding x  0 x
the derivative of the given function, as will be described
in this chapter.  x is a single symbol and does not
The Definition of Derivative mean delta times x. Do not forget that as x  0, x is
getting close to 0, but is not equal to 0.
Let there be a function y = f(x) defined in a certain
interval. The function y = f(x) has a definite value for Consequently, the derivative of a given function
each value of the argument x in this interval. Let the y = f(x) with respect to the argument x is the limit of the
argument x receive a certain increment x (it is immaterial ratio of the increment in the function  y to the
whether it is positive or negative). Then the function y increment in the argument  x, when the latter
will receive a certain increment y. approaches zero, provided the limit exists.
Thus, for the value of the argument x we will have Symbolically, the above equation is written as
y = f(x), dy df (x)
for the value of the argument x + x we will have = = f (x)
y + y = f(x + x). dx dx
where x represents finite change and dx represents
Let us find the increment of the function y
infinitesimally small change in x.
y = f(x + x) – f(x) ...(1)
Forming the ratio of the increment of the function to the It is convenient to write x as h while calculating the
increment of the argument, we get the difference quotient derivative. With such notation the derivative of the
 y f (x   x)  f (x) function f with respect to the variable x is given by
 ...(2)
x x f (x  h)  f (x)
We then find the limit of this ratio as x  0. If this f (x) = lim ...(4)
h 0 h
limit exists, it is called the derivative or differential provided the limit exists.
3.2  DIFFERENTIAL CALCULUS FOR JEE MAIN AND ADVANCED
Usually, when we say that a function f has a derivative If f(x) = x3 – 1, plot the graph of the
f (x) at a point x, it is implied that this derivative is derived function f .
finite, i.e. the above limit is finite. But it may occur that
there exists an infinite limit equal to  or –. In such For any real number x,
cases it is useful to say that the function f has at point f (x  h) – f (x)
f (x) = lim .
x an infinite derivative (equal to  or –). t 0 h
It should be emphasized that although the notation We have f(x + h) – f(x) = ((x + h)3 – 1) – (x3 – 1)
df = 3x2h + 3xh2 + h3,
suggest a ratio, the derivative as we have defined f (x  h) – f (x)
dx and so = 3x2 + 3xh + h2
it, is not a ratio – even though it is the limit of one. h
df Consequently, f (x) = lim (3x2 + 3xh + h2) = 3x2.
is simply an abbreviation of f . h 0
dx The graph of the function f (x) = 3x2 is the parabola
It will be noted that in the general case, the derivative shown in figure, on which the graph of the original
f (x) has a definite value for each value of x. If we function f(x) = x3 – 1 has also been drawn.
associate with each value of x the value of the
derivative of f(x), if it exists, we obtain a new function
of x, called simply the derivative of f(x) with respect to
x. This is denoted by f (x).
The domain of f , the set of points in the domain of f
for which the limit (3) exists, may be smaller than the
domain of f. If f (x) exists, we say that f has a derivative
(or f is differentiable) at x. f (x) often has the same
domain of definition as f(x), but not always.
The operation of finding the derivative of a function Here, we considered the derivative of a function f at a
f(x) is called differentiation of the function. If we can general point x. If we change our point of view and let
find a mathematical expression for f (x), we can then the number x be a fixed number a then we replace x in
find the value of the derivative for a particular value of Equation (4) by a. We obtain
x by mere substitution. f(a  h)  f(a)
f (a) = lim ...(5)
Find f  if f(x) = 1  x . h0 h
2x Note that we can also define
f (x  h ) – f (x ) f(x)  f(a)
f (x) = lim f (a) = xlim , provided the limit exists.
h 0 h a
xa
1  (x  h) 1  x Thus, the derivative of a function f is the function f 
– defined by
2  (x  h) 2  x
= lim f ( x )  f (a )
h 0 h f (a) = lim .
xa xa
(1  x  h) (2  x) – (1  x ) (2  x  h)
= hlim Instantaneous Rate of Change–The
0 h (2  x  h ) (2  x )
Physical Meaning of Derivative
(2  x  2h  x 2  xh) – (2  x  h  x 2  xh)
= lim
h 0 h(2  x  h) (2  x) Suppose y is a function of x, say y = f(x).
Corresponding to a change from x to x + x, the variable
– 3h
= hlim
0 h ( 2  x  h ) ( 2  x )
y changes from f(x) to f(x + x).
–3 3 The change in y is y = f(x + x) – f(x), and the average
= lim – rate of change of y with respect to x over the interval
h 0 (2  x  h) (2  x) (2  x) 2 .
from x to x + x is
DIFFERENTIABILITY  3.3
Average rate of change words, we let h approach 0. We define the velocity (or
change in y y f(x  x)  f(x) instantaneous velocity) v(a) at time t = a to be the limit
=   of these average velocities :
change in x x x
For example, let y = f(x) = x2 + 2x. Starting at x0 = 1, f(a  h)  f(a)
v(a) = lim
change x to 1.5. Then x = 0.5. The corresponding h0 h
change in y is y = f(1.5) – f(1) = 5.25 – 3 = 2.25. Hence The position of a particle is given by
the average rate of change of y on the interval between the equation of motion s = f(t) = 1/(1 + t), where t is
x = 1 and x = 1.5 is y  2.25  4.5 . measured in seconds and s in metres. Find the velocity
 x 0.5 and the speed after 2 seconds.
As the interval over which we are averaging becomes
The derivative of f when t = 2 is
shorter (that is, as x  0), the average rate of change
approaches what we would intuitively call the 1 1

instantaneous rate of change of y with respect to x, and f (2) = lim f(2  h) – f(2) 1  (2  h) 1  2
 lim
dy h 0 h h 0 h
the difference quotient approaches the derivative .
dx 1 –1 3  (3  h )
Thus, the instantaneous rate of change at x 3 (3  h )
dy y f(x  x)  f(x) = lim 3  h 3 = lim
= lim  lim = f (x) h 0 h h 0 h
dx  x  0 x  x  0 x –h
= hlim lim – 1  – 1 .
provided that the limit exists. 0 3 (3  h ) h = h 0 3 (3  h ) 9
It is conventional to use the word instantaneous even 1
when x does not represent time. The word is, however, Thus, the velocity after 2 second is f (2) = – m/s,
9
frequently omitted. When we say rate of change, we 1
mean instantaneous rate of change. and the speed is | f (2) | = – 1 = m/s.
9 9
If the dependence upon time t of the distance s of a
Suppose that a ball is dropped from
moving point is expressed by the formula
the top of a tower, 450 m above the ground.
s = f(t)
(a) What is the velocity of the ball after 5 seconds?
The velocity v at time t is expressed by the formula (b) How fast is the ball travelling when it hits the ground?
s f(t  t)  f(t) We first use the equation of motion
v = lim  lim
t  0 t t  0 t 1
Hence v = st  = f (t) s = f(t) = (9.8) t2 = 4.9t2
2
or, the velocity is equal to the derivative of the distance to find the velocity v(a) after a seconds :
with respect to the time.
f(a  h)  f(a) 4.9(a  h)2  4.9a 2
In general, suppose an object moves along a straight v(a) = lim  lim
line according to an equation of motion s = f(t), where
h 0 h h 0 h
s is the displacement (directed distance) of the object 4.9(a 2  2ah  h 2  a 2 ) 4.9(2ah  h 2 )
= lim  lim
from the origin at time t. The function f that describes h 0 h h 0 h
the motion is called the position function of the object. 4.9(2a  h)  9.8a
= lim
h0
In the time interval from t = a to t = a + h the change in
position is f(a + h) – f(a). The average velocity over (a) The velocity after 5s is v(5) = (9.8)(5) = 49 m/s.
(b) Since the height of tower is 450 m above the ground,
this time interval is the ball will hit the ground at the time a when
displacement f(a  h)  f(a) s(t1) = 450, that is, 4.9a2 = 450
average velocity = 
time h 450
Now suppose we compute the average velocities over This gives a2 = and a = 450  9.6 s.
4.9
shorter and shorter time intervals [a, a + h]. In other 4.9
3.4  DIFFERENTIAL CALCULUS FOR JEE MAIN AND ADVANCED
The velocity of the ball as it hits the ground is therefore As another example, both x and y-axes have one
common point each with the parabola y = x2, but the
450
v(a) = 9.8a = 9.8  94 m/s. x-axis is tangent to the parabola, while the y-axis is
4.9 not. Thus, we see that for more complicated curves,
Slope of tangent – The Geometrical more careful definitions must be made.
Meaning of Derivative If we have a simple continuous curve, our intuition
The concept of the line tangent to a curve at a point is and experience tell us that a point which moves along
an important one in geometry. We shall now consider this curve constantly changes its direction of motion,
the problem of defining the tangent line to the graph but that at each point P of the curve there is a definite
of a function f at a point. We will soon find that the straight line which gives the direction of motion and
concept of tangent can be expressed in purely analytic closely approximates the curve near P. The line is called
terms involving the function f. In fact, the problem the tangent at P and may be approximated in drawing
leads directly to the definition of the derivative of a by turning a ruler about P until it appears to have the
function, the central idea in differential calculus. proper position.
What does it mean to say that a line is tangent to a In the figure below, Q, Q, Q, Q are several
curve at a point ? School geometry defines the tangent successive positions of the point Q as it approaches
to a circle as a straight line lying in the plane containing P. Evidently the secant PQ rotates about P and
the circle and having a single point in common with it. approaches a limiting position PT, which is the tangent.
In other words, the tangent line to a circle at point P is
the line that is perpendicular to the radial line at point A secant line is a line that intersects a
P, as shown in the first figure below. However, it is not curve in two or more points.
so simple an idea as it may first appear.
For example, how would one define the tangent lines
shown in the other figures? We might say that a line is
tangent to a curve at point P if it touches, but does not
cross, the curve at point P. This definition would work
for the first curve shown in the figure, but not for the
second. Or we might say that a line is tangent to a Find the slope of the secant line
curve if the line touches or intersects the curve at joining the points (1, – 2) and (1.2, – 0.56) of the parabola
exactly one point – this definition would work for a y = x2 + 5x – 8.
circle but not for more general curves, as the third Given y = f(x) = x2 + 5x – 8, we find y and
curve in the figure shows. Here, the tangent "cuts" y/x as x changes from x0 = 1 to
the curve at the point of tangency instead of
"touching" it. In the fourth curve, the tangent "touches" x1 = x0 + x = 1.2
the curve at P, but also "cuts" it again on the y-axis. x = x1 – x0 = 1.2 – 1 = 0.2 and
y = f(x0 + x) – f(x0) = f(1.2) – f(1)
= – 0.56 – (–2) = 1.44.
y 1.44
So   7. 2
 x 0 .2
The slope of the secant line joining the points (1, – 2)
and (1.2, – 0.56) is 7.2.
Essentially, the problem of finding the tangent line at a
point P boils down to the problem of finding the slope
of the tangent line at P. We can approximate this slope
using a secant line through the point of tangency and
a second point on the curve. This method of
approximation leads to the following definition.
DIFFERENTIABILITY  3.5
Definition The tangent to the curve at the point P point (a, f(a)) is perpendicular to the x-axis and we call
is the limit of a secant line through P and another point such a line as a vertical tangent. The equation of the
Q on the curve as Q approaches P. vertical tangent is x = a. Here, the function f(x) is said
Let P(a, f(a)) and Q (a + h, f(a + h)) be two points very to have an infinite derivative at the point x = a.
close to each other on the curve y = f(a). Draw PM and Note that not every curve has a tangent at any of its
QN perpendiculars from P and Q on x-axis, and draw points because the existence of tangent depends on
PL as perpendicular from P on QN. Let the chord PQ the existence of the above limit.
produced meet the x-axis at R and QPL = QRN = . Definition The tangent line to the graph of f at the
Now in right angled triangle QLP point P(a,f(a)) is
QL NQ  NL NQ – MP (i) the line y – f(a) = f (a)(x – a) through P with slope
tan =   f (a), if f (a) exists;
PL MN ON  OM
f (a  h) – f (a) f (a  h) – f (a) f ( x )  f (a )
(ii) the line x = a if lim = .
= (a  h)  a = ...(1) x a xa
h
If neither (i) nor (ii) holds, then the graph of f does not
have a tangent line at the point P(a, f(a)).
Find the equation of the tangent line
to the parabola y = x2 – 8x + 9 at the point (3, – 6).
f (a  h ) – f (a )
We have f (a) = lim
h 0 h
2 2
[(a  h ) – 8(a  h )  9] – [a  8a  9]
= lim
When h  0, the point Q moving along the curve h 0 h
2 2 2
tends to P, i.e., Q  P. The chord PQ approaches the = lim a  2ah  h  8a  8h  9 – a  8a  9
tangent line PT at the point P and then . Now h 0 h
applying 2ah  h 2  8h lim
= lim = h  0 (2a + h – 8) = 2a – 8.
f (a  h) – f (a) h 0 h
lim tan   lim Since the derivative of f(x) = x2 – 8x + 9 at the number a
h 0 h 0 h
is f (a) = 2a – 8, the slope of the tangent line at (3, – 6)
f (a  h) – f (a)
tan  = lim = m = f (a) is f (3) = 2(3) – 8 = – 2.
h 0 h
Thus, the equation of the tangent line is
If the limit exists, then the function f(x) is said to have y – (– 6) = (– 2) (x – 3) or y = – 2x.
a finite derivative at the point x = a and the line passing
through (a, f(a)) with slope m is the tangent line to the Let f(x) = x2 – 4x + 7.
graph of f at the point (a, f(a)). (i) Find the average rate of change of f with respect
The slope of the tangent line to the graph of f at the point to x between x = 3 and 5.
(a, f(a)) is also called the slope of the graph of f at x = a. (ii) Find the instantaneous rate of change of f at x = 3.
If we use the point slope form of the equation of a line, Interpret the result geometrically.
we can write an equation of the tangent line to the
curve y = f(x) at the point (a, f(a)) as : (i) The (average) rate of change from x = 3 to x = 5 is
y – f(a) = f(a) (x – a) found by dividing the change in f by the change
Further, if lim f (x) does not exist in the ordinary sense, in x. The change in f from x = 3 to x = 5 is
x a
f(5) – f(3) = [52 – 4(5) + 7] – [32 – 4(3) + 7] = 8.
but lim | f (x) | =  then the secant line gets steeper Thus, the average rate of change is
x a
and steeper as x approaches a. In this case the line f(5) - f(3) 8
tangent to the graph of the function y = f(x) at the = = 4.
5-3 2
3.6  DIFFERENTIAL CALCULUS FOR JEE MAIN AND ADVANCED
The slope of the secant line is 4, as shown in the figure. 5
1  x – 1 1
(b) f (1) = lim = lim = .
x0 x x 0 5 ( x ) 4

Essentially, both the functions donot have a finite


derivative at the given points.
Meaning of Sign of Derivative
Remember that a positive derivative signals an
increasing quantity and that a negative derivative
signals a decreasing quantity.
This is obvious also from the geometrical interpretation
dy dy
of . For, if x and y are increasing together is the
dx dx
tangent of an acute angle and therefore positive, while
dy
if, as x increases y decreases, represents the
dx
tangent of an obtuse angle and is negative.
f (3  h) – f (3)
(ii) f (3) = lim This aspect will be studied in detail in the chapter of
h 0 h monotonicity.
[(3  h)2 – 4(3  h)  7] – [32  8.3  7]
= lim 3.2 DIFFERENTIABILITY
h 0 h
2h  h 2 The derivative of the function f with respect to the
= lim = 2. variable x at x = a is given by
h 0 h
The derivative of the function at x = 3 is 2. f (a  h)  f (a)
f (a) = lim
Thus, the instantaneous rate of change of f at x = 3 is
h 0 h
provided the limit exists.
f (3) = 2.
If the derivative of f(x) exists for x = a, we say that f(x)
The tangent line at x = 3 has slope 2, as shown in the
is differentiable (derivable) for x = a, otherwise, it is
figure.
non-differentiable.
Find equation of the tangent line to If a function f (x) is differentiable at x = a, the graph of
3 f (x) will be such that there is a tangent with finite slope
the graph of the function f(x) = x at the origin.
to the graph at the corresponding point. But if f (x) is
We have non-differentiable at x = a, there will be either vertical
f (x)  f (0) 3
x 1 tangent or no tangent with finite slope at the
lim  lim  lim 2/3   . corresponding point of the graph.
x 0 x0 x 0 x x 0 x
Therefore, the tangent is the vertical line x = 0. If f(x) is differentiable for every value of x in an interval
I, we say that f(x) is differentiable in the interval I.
Show that the following functions
If f is differentiable at every point in its domain, it is
do not have finite derivative at the indicated points : simply called a differentiable function.
(a) y= 5
x 2 at x = 0 (b) y = x  1 at x = 1
5 If f is differentiable on (– ) we will say that f is
differentiable everywhere.
5
( x) 2 lim 1
(a) f(0) = lim = x 0 5 . If g(x) = 1/x, find the derivative g(x).
x  0 x ( x)3
Also find the domain of g(x).
Now the left hand limit is – and right hand limit is . 1 1
Hence, the limit does not exists and there is no 
g(x) = lim x  h x
derivative at x = 0. h 0 h
DIFFERENTIABILITY  3.7

x  (x  h) 1 Let us choose a point B to the right of A. If the slope of


= lim
h 0 hx(x  h)
= lim
h 0 x(x  h) tangent at A is evaluated by allowing B to approach A
along the curve, then we have
1
= 2 f (a  h)  f (a)
x Slope of right tangent = lim  f '(a  )
Evidently, the domain of g(x) is the set of all non-zero h 0 h
numbers, just as is the domain of g(x). That is, g is which is called the Right Hand Derivative (R.H.D.) of
differentiable for all x except x = 0. f at x, provided the limit exists..
Let us choose a point C to the left of A. If the slope of
One-sided derivatives tangent at A is evaluated by allowing C to approach P
For the derivative of f at a point x to exist, it is necessary along the curve, then we have
that the function f be defined in a certain f (a)  f (a  h)
neighbourhood of the point x including the point x
Slope of left tangent = lim  f '(a  )
h 0 h
y f(x  x)  f(x) which is called the Left Hand Derivative (L.H.D.) of f
itself. Then the expression 
x x at x, provided the limit exists..
is defined for all sufficiently small x different from zero. Sometimes, we prefer to write the left hand derivative
f(x  x)  f(x) f (a  h)  f (a)
Now, the limit f (x) = lim does not as f (a–) = lim .
x 0 x h0 h
always exist for any function f defined in a
neighbourhood of the point x. Differentiability at an Interior Point
The one-sided limit form of the derivative is useful in A function f(x) is said to be differentiable at some
investigating the existence of derivative. interior point x = a in its domain if both L.H.D. and
If in the above limit it is assumed that x tends to zero R.H.D. exist at that point, and are equal, that is, if
attaining only positive values (x > 0), then the f  (a+) = f (a– ) = a finite quantity.
corresponding limit (wherever it exists) is called the This geometrically means that a unique tangent with
right hand derivative of the function f at a point x. We finite slope can be drawn at x=a as shown in the figure.
shall denote it as f (x+). There are cases of failure of existence of f (a) and even
Analogously, the limit, when x tends to zero running of f '(a  ) and f '(a  ) at the point x = a, i.e. when the
through negative values (x < 0), is termed as the left graph of the function has neither a right, nor a left
hand derivative of f at x (denoted as f (x–). tangent at the given point.
Of course, to compute f (x+) (or f (x–)) it is only It may happen that f has at x = a, left and right
necessary that the function f be defined at the point x derivatives different from each other : i.e. f (a–)  f (a+).
and on the right of it in a certain neighbourhood (or at
x and on the left of x). Consider the existence of
derivative of y = f(x) at the point A(a, f(a)).

Then such a point is called a sharp corner. In this case


the tangent line does not exist at the point, but we say
that there exist right and left tangents with different
slopes at that point.
Differentiability at Endpoints
At the left endpoint of domain, the existence of R.H.D.
is a sufficient condition for differentiability and at the
3.8  DIFFERENTIAL CALCULUS FOR JEE MAIN AND ADVANCED
right endpoint of domain, the existence of L.H.D. is a  sin x 2
sufficient condition for differentiability.  if x  0
Let f (x) =  x
If x = a is the left endpoint of the domain of the function 0
 if x  0
f, then f is differentiable at x = a if the right hand
derivative Find the slope of tangent at x = 0, if it exists.
f (a  h)  f (a) f(0  h)  f(0)
lim exists. L.H.D. = f (0–) = lim
h 0 h h0 h
If x = b is the right endpoint of the domain of the function sin h 2
f, then f is differentiable at x = b if the left hand = lim  0 = 1.
h 0  h 2
derivative
f(0  h)  f(0)
f (b  h)  f (b) R.H.D. = f (0+) = lim
lim h 0 h
exists.
h 0 h sin h 2
= lim  0 = 1.
Examine the differentiability of the h 0 h 2

function f(x) = ln2x at x = 1. Since L.H.D.= R.H.D., f(x) is differentiable at x = 0 and


f(1  h)  f(1) the slope of tangent at x = 0 is 1.
L.H.D. = f(1–) = hlim
0
h Comment on the differentiability of
ln 2 (1  h)  0 x , x 1
= lim = 0. f(x) =  2 at x = 1.
h 0 h x , x 1
f(1  h)  f(1) f(1  h)  f(1)
R.H.D. = f (1+) = hlim 0 L.H.D. = f(1–) = hlim
h 0
h
2
ln (1  h)  0 1  h  1
= lim = 0. = hlim = 1.
h 0 h 0
h
Since L.H.D.= R.H.D. f(x) is differentiable at x = 1 and f(1  h)  f(1)
f (1) = 0. R.H.D. = f (1+) = hlim 0
h
Examine the differentiability of the (1  h)2  1 lim 1  h 2  2h  1
= hlim
0 = h0
function f(x) = | ln x | at x = 1. h h
ln x if ln x  0 lim
= h  0 (h + 2) = 2.
f (x) = | ln x | = 
  ln x if ln x  0 Since L.H.D.  R.H.D., f(x) is not differentiable at x = 1.
ln x if x  1 If f(x) = A + Bx2 , x < 1
=
  ln x if 0  x  1 = 3Ax – B + 2, x1
f(1  h)  f(1) then find A and B so that f(x) becomes differentiable at
L.H.D. = f(1–) = hlim x = 1.
0
h
 ln(1  h)  0 f(1  h)  f(1)
= lim = – 1. L.H.D. = f(1–) = hlim
h 0 h
0
h
f(1  h)  f(1) A  B(1  h)2  3A  B  2
R.H.D. = f (1+) = hlim = hlim
0
0
h h
ln(1  h)  0 ( 2A  2B  2)  Bh 2  2Bh
= lim = 1. = hlim
0
h 0 h h
Since L.H.D.  R.H.D., f(x) is not differentiable at Hence, for this limit to exist – 2A + 2B – 2 = 0
x = 1. i.e., B = A + 1. Then,
DIFFERENTIABILITY  3.9

f (1–) = hlim Show that the function f(x) = |cos x|


 0 (Bh – 2B) = 2B
2k  1
f(1  h)  f(1) does not have derivative at x = , k  I.
R.H.D. = f (1+) = hlim 2
0
h  2k  1 
cos    x 
3A (1  h)  B  2  3A  B  2  2k  1    2 
= hlim
0 f   = lim 
h  2  x  0 x
3Ah sin x
= hlim
0 = 3A. = lim   –1
h x  0 x
Since f is differentiable at x = 1, we have f (1–) = f(1–)  2k  1   sin x
 3A = 2B  3A = 2(A + 1) f   = lim  1.
 2  x 0 x

 A = 2, B = 3.
Since L.H.D.  R.H.D., f(x) is not derivable at
[cos x] x  1 2k  1
Let f(x)=  .Comment x= , k  I.
 2{x}  1 x  1 2
on the derivability at x = 4. Examine the differentiability of the
f(1  h)  f(1) 
f(1–) = hlim function f(x) = cosx + | cos x | at x = .
0
h 2
cos(π - πh) + 3 lim 1  cos h
= hlim = h0 
0
-h h 
 2cos x if 0  x  2
h f (x) = cos x + | cos x | = 
2 sin 2 
lim
= h0 2 = 0. 0 if  x  
h  2
f(1  h)  f(1)
f(1+) = hlim
0
h
2{1  h}  1  1 lim
= hlim
0 = h  0 2h = 2.
h
Since L.H.D.  R.H.D., f(x) is not derivable at x = 3.
A function f(x) is such that  –  2sin h - 0
f   = lim =–2
2
  h ®0 h
   
fx    – |x|  x. Find f   , if its exists.   
 2 2 2 00
f   = lim 0.
   2
  h  0 h
Given that f  x    – |x| Since the left and right derivatives are different from each
 2 2
other, the graph of the function has a corner point at
 
f   h   f     | h |   
    2   2 2 2  1 . x= and f(x) is not differentiable here.
f     lim  2
2
  h 0 h h
Given f (x) = [x] tan ( x) where [.]
 
f   h   f     |  h |   denotes greatest integer function, find the LHD and
  
f     lim  2  2  2 2  1.
RHD at x = k, where k  I.
2
  h 0  h h
f (x) = [x] tan (x)

 f    does not exist. f(k  h)  f(k)
2 f  (k+) = lim
h 0 h
3.10  DIFFERENTIAL CALCULUS FOR JEE MAIN AND ADVANCED

[k  h]· tan(k  h)  0 7


= lim 
7  3 
3
h 0 h  a = 2 and hence b = 1 =   
9 
3    26 
k tan( h)
= lim = k 3
h 0 h 7 b 7
=  = .
f(k  h)  f(k) 26 a 52
f  (k–) = lim
h0 h ax 2 – bx  2 if x3
Let f(x) =  .
[k  h]· tan(k  h)  0  bx 2  3 if x3
= lim
h 0 h Find the values of a and b so that f(x) is differentiable
(k  1) tan( h) everywhere.
= lim f (3–) and f (3+) exist finitely.
h0 h
=  (k – 1). f (3  h )  f (3)
Hence R.H.D. = f (k+) = kand L.H.D. = f  (k–) = (k – 1).  f (3–) = lim
h 0 –h
a (3  h )2  b(3  h )  2  (9b  3)
= lim
h 0 –h
 x 2  x tan x  x tan 2x
 ax  tan x  tan 3x , x  0 ah 2  h( b  6a )  9a  12 b  5
= lim ...(1)
 h 0 (– h )
Let g(x) =  .
 f (3 –) exists, the numerator must tend to zero
 0, x0 as h  0
If g  (0) exists and is equal to a non-zero value b, then  9a – 12b + 5 = 0 ...(2)
b Now from (1) and (2)
find the value of .
a ah 2  h( b – 6a )
f (3–) = lim = 6a – b,
x 2  x tan x  x tan 2x h 0 h
g(0) = b = lim f (3  h )  f (3)
x0 x(ax  tan x  tan 3x) and f (3+) = lim
h 0 h
x  tan x  tan 2 x b(3  h ) 2  3  (9b  3)
= lim = lim
x0
ax  tan x  tan 3x h 0 h
(3  h ) 2 – 9
 x3 2  = b lim = 6b. ...(3)
x   x   x 5  ........  h 0 h
 3 15  Given f (3–) = f (3+)
 8x3 2 
  2x   ·32x 5  .......   6a – b = 6b  b = 6a . ...(4)
 3 15  7
= lim
x 0
 x 3
2 5  From (2) and (4),
ax   x   x  ..... 
 3 15  9a – 72a + 5 = 0  – 9a + 5 = 0
7 7
 27x3 2  10
  3x 
3
 243x 5  ..... 
15
 a = 35 , b = .
  9 3
7 62 2 A function f is defined by f(x2) = x3
x3    x  ...  for all x > 0. Show that f is differentiable at 4.
 3 15 
= lim
x0  1  3 2
(a  1  3) x  9  x  ( 242)x 5  ...
3  15
f (4)  lim
f(4  h)  f(4)
 lim
 
f ( 4  h )2  f  2 2 
limit b can be non - zero if a + 1 – 3 = 0 h 0 h h0 h
DIFFERENTIABILITY  3.11

(4  h)3/2  8 8[(1  h / 4)3/2  1] f '( a )


 lim  lim  f ' (a )  f ( a )
h 0 h h 0 h = (exp.)   = e .
3h 3 3  h2    f (a ) 
8 1   ....  1 8 h     ...
 24  8 8  16
 lim   lim    Let f : R  R satisfying |f(x)|  x2,
h0 h h 0 h
= 3.  x  R then show f(x) is differentiable at x = 0.
Hence f is differentiable at 4. Since, |f(x)|  x2 ,  x  R
Let f be differentiable at x = a and let at x = 0, |f(0)|  0  f(0) = 0 ...(1)
n
f(a  1 n)  . f(h)  f(0) f(h)
f(a)  0. Evaluate lim 
   f (0) = lim = lim ....(2)
n  f(a)  h0 h h0 h
n {f(0) = 0 from (1)}
 f(a  1 n) 
l = lim   (1 form) f(h)
n  f(a)  f(h)
Now, |h|  – | h |  | h |
  f(a  1 n)  f(a)   h h
l = (exp.)  nlim n 
   f(a)  f(h)
1  lim =0 ...(3)
put n =
h0 h
h {using Sandwitch theorem}
= (exp.)  lim f(a  h)  f(a) · 1   from (2) and (3) , we get f(0) = 0.
 h 0 h f(a)  i.e. f(x) is differentiable at x = 0.

A
1. Examine the differentiability of the function (c) Show that y=x2/3 has a vertical tangent line
f(x) = e– | x | at x = 0. at(0, 0).
(d) Illustrate part (c) by graphing y = x2/3.
sin x if x  
2. Let f (x) =  where m and n are 5. Compute the difference quotient for the function
 mx  n if x    sin x if x  0
constants. Determine m and n such that f is defined by f(x) =  x
derivable at x = .  1 if x  0
3. Discuss the differentiability of f at x = 1 Do you think f(x) is differentiable at x = 0? If so,
x what is the equation of the tangent line at x = 0 ?
f(x) = 3 , – 1  x  1
= 4 – x, 1 < x < 4  x tan 1 1 / x, x  0
4. (a) If g(x) = x2/3, show that g(0) does not exist. 6. Let f(x) =  .
 0 , x0
(b) If a  0, find g(a). Comment on the derivative of f(x) at x = 0.

A
7. Find the slope of the secant to the parabola 8. Find f (0+) and f (0–) if
y = 2x – x2, if the abscissas of the points of (a) f(x) = sin(x 2 )
intersection are equal to : x1 = 1, x2 = 1 + h. To
what limit does the slope of the secant tend if a 2  x2
h0? (b) f(x) = sin–1
a 2  x2
3.12  DIFFERENTIAL CALCULUS FOR JEE MAIN AND ADVANCED

(c) f(x) = x , x  0; f(0) = 0 (iii) a vertical tangent, where the absolute value of
1
slope of PQ approaches 
1 ex (iv) a discontinuity.
1
(d) f(x) = x2 sin , x  0; f(0) = 0. Such slopeless points may be roughly classified as
x
"sharp points", "oscillation points", "steep points",
9. Discuss the differentiability of the function and "points of discontinuity".
x(3e1/x  4)
f(x) = , x0 CASE I : Sharp corner or kink
2  e1/x
=0, x=0 If it happens that in a continuous function f at x = a,
at x = 0. the left hand and right hand derivatives are different
from each other i.e. f (a–)  f (a+), then such a point is
10. Prove that the function
called a sharp corner or kink.
| x |3/2 sin 1 if x  0, x  R,

f(x) =  x
0 if x  0,
is differentiable at the point x = 0, and f (0) = 0.
11. It is known that f(0) = 0 and there exists a limit of
f (x)
the expression as x  0. Prove that the limit
x
is equal to f (0).
For example, we see that the function y = |x| as shown
12. Prove that if f(x) and (x) are equal to zero for in the figure has a graph which changes its direction
x = 0 and have derivatives at x = 0, (0) being not abruptly at x = 0. The point x = 0 is called a corner and
f ( x ) f ' (0 ) the function y = |x| is not differentiable at 0.
equal to zero, then xlim  0 ( x )
 .
' (0)
13. Prove that if f(x) has a derivative at x = a, then
lim xf (a ) – af ( x )  f (a ) – af ' (a ) .
x a xa
14. Let f(x) be an even function defined for x  R. In general, if the graph of a function f has a “corner” or
If f (0) exists, find its value. “kink” in it, then the graph of f has no tangent at this
x g(x) point and f is not differentiable there. This can be
15. Let f(x) = , g(0) = g’(0) = 0 and f(x) be verified by computing the left and right hand
|x|
continuous at x = 0. Find f’(0), if it exists. derivatives at the point.

3.3 REASONS OF NON-


DIFFERENTIABILITY But one should be careful about predicting a corner at
We now find the reasons because of which a function a point. For instance, one may feel that the function
does not have a derivative at a point. f(x) = |x 3 | has a corner at x = 0 and hence
A function has a derivative at a point x = a if the slopes non-differentiable there. This is found to be wrong.
of the secant lines through P(a, f(a)) and a nearby point
Q on the graph approach a limit, as Q approaches P.
However, a function will fail to have a derivative at a
point where the graph has
(i) a corner, where the one-sided derivatives differ.
On computing, we find that f (a–) = 0  f (a+).
(ii) an oscillation point, where the one-sided
So, f is differentiable at x = 0.
derivative(s) does (do) not exist.
DIFFERENTIABILITY  3.13

CASE II : Oscillation point Since both L.H.D. and R.H.D are infinite, the function
If in a continuous function f, either the left hand derivative has a vertical tangent at x = 0.
or right hand derivative does not exist at x = a, because of  The y-axis i.e. x = 0 is the vertical tangent to
too much oscillation of the graph in the neighbourhood f(x) = x1/3 at origin. Note that f is not differentiable at x = 0.
of the point, then such a point is called an oscillation point. The graph of a continuous function f having a vertical
 1 tangent at (x0, f(x0)) can have
For example, the function f(x) = 

x sin , x  0
x
(a) f (x) approaching either  or – as x  x0– and
0, x 0 as x  x0+, i.e. where the slope of the secant line
approaches  from both sides or approaches
has an oscillation point at x = 0.
–  from both sides, or,
This is illustrated in the following figure.
(b) f (x) approaching  from one side and – from
the other side. In such a case, the function f is
said to have a cusp at x0.

CASE III : Vertical Tangent


A continuous function may also fail to be differentiable
at x = x0 if the absolute value of its difference quotient
approaches infinity. In this case, the function is said
to have a vertical tangent at x = x0.
Figure (a) Figure (b)
In figure (a), the slope of PQ approaches –  from both
sides. In figure (b), the slope of PQ approaches  from
left side and –  from right side.
Thus, the graph of the continuous function f(x) = x2/3
has a vertical tangent line at the origin, even though f
is not differentiable at x = 0, since, the function f(x) = x2/3
Definition The curve y = f(x) has a vertical tangent is continuous at x = 0, and has infinite derivative at x = 0:
line at the point (x0, f(x0)) provided that f is continuous
(0  h) 2/3  0
at a and L.H.D. = lim = –
f (x)   as x  x0 ...(1) h0 h
Note that the requirement that f be continuous at x = x0 (0  h) 2/3  0
R.H.D = lim = 
implies that f(x0) must be defined. Thus, it would be h0 h
pointless to ask about a line (vertical or not) tangent Further, since the one-sided derivatives are infinity
to the curve y = 1/x where x = 0. opposite signs, we say that its graph has a cusp at x = 0.
If f is defined on only one side of x = x0, we mean by (1) The graph of the function f(x) = 1– 5 x 2 , has a
that f (x)   as x approaches x0 from that side. cusp"(rather than a corner) at the point (0,1) because
For example, the function f(x) = x1/3 is continuous at 2 3/5
x = 0, and the derivative f (x) =  x approaches  as x  0–
5
(0  h)1/3  0 and approaches –  as x  0+. The curve also has a
L.H.D. = lim =
h0 h vertical tangent at the point.
(0  h)1/3  0 A function cannot have a vertical
R.H.D = lim =
h0 h tangent at a point of discontinuity. For example,
3.14  DIFFERENTIAL CALCULUS FOR JEE MAIN AND ADVANCED
y = sgn (x) does not have a vertical tangent at x = 0 There is a vertical tangent and cusp at x = 4
even if both the one- sided derivatives at x = 0 are
infinite. since f(x) = (x – 4)2/3 is continuous at x = 4 and
lim f (x) = lim 2
1/ 3 = 
The figures give below show four curve elements that
x 4  x  4 3( x  4)

are commonly found in graphs of functions that involve
radicals or fractional exponents. In all four cases, the lim f (x) = lim 2
x 4  1 / 3 = –
function is not differentiable at x0 because the secant x4 3( x  4)

line through (x0, f(x0)) and (x, f(x)) approaches a vertical Now consider f(x) = 6x1/3 + 3x4/3.
position as x approaches x0 from either side. Thus, in 2(2 x  1)
each case, the curve has vertical tangent line at (x0, f(x0)). f (x) = 2x–2/3 + 4x1/3 = 2x–2/3(1 + 2x) = .
x2 / 3
There is point of vertical tangency at x = 0, since
lim f (x) = lim 2( 2 x  1) = 
x 0  x 0  x2 / 3
lim f (x) = lim 2(2 x  1) = 
x 0  x0  x2 / 3
It does not have a cusp at x = 0.
lim f (x) =  lim f (x) = 
x  x 0 x  x 0
CASE IV : Discontinuity
There is another way for a function not to have a
derivative. We shall prove in the next section that if f
is discontinuous at x = a, then f is not differentiable at
x = a. So, at any discontinuity, f fails to be differentiable.

lim f (x)= –  lim f (x)= – 


x  x 0 x  x 0

The following figures of discontinuous functions


suggest that the secant lines donot approach a finite
lim f (x) =  lim f (x) = –  slope from atleast one side.
x  x 0 x  x 0

lim f (x)= –  lim f (x) = 


x  x 0 x  x 0

Note that the third and fourth figures represent a cusp.


Consider f(x) = (x – 4)2/3.
2 2
f (x) = (x – 4)1/3 =
3 3( x  4)1 / 3
DIFFERENTIABILITY  3.15

3.4 RELATION BETWEEN h2  0


f (0+) = hlim =0
CONTINUITY AND 0
h
DIFFERENTIABILITY h2  0
f (0 ) = hlim  =0
Theorem If a function f is differentiable at x = a 0 h
then it must be continuous at x = a.  f is derivable at x = 0.
Proof Since f is differentiable at x = a,  f is continuous at x = 0.
f(a  h)  f(a)
f (a) = lim exists. ...(1) The converse of the above theorem is not
h0 h
To prove that f is continuous at x = a, we must prove
necessarily true
If f is continuous at x = a, then f may or may not be
that lim  f (a  h)  f (a)   0 .
h 0 differentiable at x = a.
For example, the functions f(x) = x and
 f (a  h)  f (a)  .h
Now lim  f (a  h)  f (a)   lim 1
h 0 h 0 h g(x) = x sin if x  0; g(0) = 0 are
x
 lim
 f (a  h)  f (a)  .lim h  f '(a).0  0 continuous at x = 0 but not differentiable at x = 0.
h 0 h h 0
Since f is continuous at x = a, we have
{using (1)}
lim  f (a  h)  f (a)   0 . ...(1)
 lim f (a  h)  f (a)  f is continuous at x = a. h 0
h 0
We need to check whether
Further, if f is differentiable at every point of its
f(a  h)  f(a)
domain, then it is continuous in that domain. f (a) = lim exists. ...(2)
h0 h
Examine the differentiability of the 0
Now (1) suggests that the limit (2) is in   form, but
 cos x  1 0
 ,x0 we know that only some of these limits exist and hence
function f(x) =  x at x = 0 and comment
 0, x0 we are not certain about the existence of the derivative.
on continuity at that point. In fact, continuity of a function at a point is a necessary
condition for its differentiability at this point. However,
We have it should be noted that the continuity of a function at
cos x – 1 – 0 a point is not a sufficient condition for the derivative
f (x) – f (0) x of this function to exist at that point, i.e. the continuity
f (0) = lim = lim
x 0 x–0 x0 x–0 of a function at a point does not necessarily mean its
cos x – 1  lim – sin 2 x differentiability at that point.
= xlim 2 2 Thus, differentiability of a function is a stronger
0 x x 0 x (cos x  1)
condition than continuity alone.
  sin x    lim
2
1  1  x p sin 1 , x  0
= – xlim    =– .
 0  x   x0 cos x  1  2 
Let f(x) =  x .
Since, f is differentiable at x = 0, and using the above  0 , x0
theorem, it is also continuous at x = 0.
Find the values of p for which (i) f is continuous but
Given f (x) = x2 . sgn (x), examine the not differentiable at x = 0 (ii) f is continuous and differ-
continuity and derivability at x = 0 . entiable at x = 0.
 x 2 if x  0 f(0) = 0
f (x) =  0 if x  0
 For continuity, lim
x 0
f(x) = 0
  x if x  0
2
3.16  DIFFERENTIAL CALCULUS FOR JEE MAIN AND ADVANCED
1  1 1 1
 lim xp sin 0.
x 0 x  | x | , x   2 , x  2
This is possible only when p > 0. ...(1) 
For differentiability, f (x) =  1 1
a  bx 2 ,   x 
1  2 2
h p sin 0
f (0  h)  f (0) h  1 1
f (0)  lim  lim , x
h 0 h h 0 h  x 2
p 1 1 
should exist i.e. lim h sin should exist.  1 1
h 0 h  , x
Thus, f (0) will exist only when p > 1 = x 2
 1 1
 f(x) will not be differentiable if p  1 ...(2) a  bx 2 ,   x 
From (1) and (2), 2 2
(i) f is continuous but not differentiable at x = 0 1
 1   1 2
if 0 < p  1. 
1  f   h f   1
  h
f '   = lim  2  2 =  
lim  
(ii) f is continuous and differentiable at x = 0 if p > 1. 2
 2  h 0 h h0 h
ax  b for for x  1
1
1  2   h 
If f (x) =  3
ax  x  2b x  1
2  = lim  2h = – 4.
is differentiable for at x = – 1, find a and b. = lim
1 h 0  1
  h   h 
h 0
For differentiability, continuity is a h  h
2  2 
necessary condition. Hence L.H.L. = R.H.L. at x = – 1.
1 1
– a + 1 = – a – 1 + 2b f   h   f  
 1 
f '   = lim   2
– a – 1 + b + a = 0 b = 1. 2
f(  1  h)  f( 1) 2
  h  0  h
Now, f (–1–) = lim 2
h 1  a  b  2   bh  h 2
a  b   h   2
h0
a( 1  h)  b  (b  a)  
= lim = lim 2  = lim  4 
h0 h h 0 h h0 h
a  ah  a b
= lim =a For existence of this limit a  2 ....(1)
h0 h 4
and, f (–1+) = lim
3
[a( 1  h)  ( 1  h)  2b]  [b  a]  f must be continuous at x = ± 1/2.
h 0 h
 1 
ah(h 2  3h  3)  h  b  1 Then, f '   = b.
= lim { using b = 1} 2 
h0 h
= 3a + 1.  1   1 
1 Now f '   = f '  
2 
Hence f  (–1+) = f  (–1–)  3a + 1 = a  a = – .   2 
2
1  b = – 4.
Thus, a = – , b = 1. Using (1), a – 1 = 2  a = 3.
2
Hence, a = 3 and b = – 4.
A function f is defined as , Theorem If a function f is discontinuous at x = a
 1 if | x |  1 then it is non-differentiable at x = a.
 |x| 2 Proof If a function f is discontinuous at x = a then
f (x) =  .
a  b x 2 if | x |  1 the equation lim  f (a  h)  f (a)   0 does not hold true.
 2 h 0
If f (x) is derivable at x = 1/2 find the values of a and b. For f to differentiable at x = a,
DIFFERENTIABILITY  3.17
f(a  h)  f(a) f(a  h)  f(a)
f (a) = lim must exist. f (a+) = lim exists. ...(4)
h0 h h0 h
0  f (a  h)  f (a)  .h
We know that this limit can only exist in   form,
0 Now lim  f (a  h)  f (a)   lim
h 0 h 0 h
which is not possible because the numerator does not
approach to 0 because the condition  lim
 f (a  h)  f (a)  .lim h  f '(a).0  0
h 0 h h 0
lim  f (a  h)  f (a)   0 does not hold true. {using (4)}
h 0
Hence, f is not differentiable at x = a. Thus, both the conditions (1) and (2) are established.
Hence, the function is continuous at x = a.
Comment on the differentiability of
Note that if any of the one-sided derivatives at x = a
x , x 1 does not exist then, f may or maynot be continuous at
f(x) =  at x = 1.
x  2 , x  1 x = a.
f(1  h)  f(1)
L.H.D. = f(1–) = lim
h0 h
1 h  3
= lim = . Differentiable  Continuous
h0 h 
Continuous  Differentiable
f(1  h)  f(1) Discontinuous  Non-differentiable
R.H.D. = f (1+) = lim
h0 h Both one-sided derivatives exist  Continuous
(1  h  2)  3 h Non-differentiable  
  Discontinuous
= hlim
0 = hlim
0 = 1.
h h xe1/x
Since L.H.D.  R.H.D. f(x) is not differentiable at x = 1. If f(x) = , x  0, f(0) = 0,
1  e1/x
If we use the previous theorem, we see that
examine the continuity and differentiability at x = 0.
L.H.L. = f(1–) = 1 and R.H.L. = f(1+) = 3
We check the differentiability first.
f(1) = 3. Thus, the function is discontinuous and hence
non-differentiable. We need not find L.H.D. and R.H.D. xe1/x
f (x)  f (0) 1/x
0
here to find differentiability. f (0–) = lim = lim 1  e
Theorem If a function f is non-differentiable at x =
x 0 x 0 x 0  0 x
1/x
a but both the one-sided derivatives exist (though e
= lim = 0.
being unequal), then f is continuous at x = a. x0  0 1  e1/x

Proof To prove that f is continuous at x = a, we must xe1/ x


0
prove that lim  f (a  h)  f (a)   0 ...(1) f (x)  f (0) 1  e1/ x
f (0+) = lim = lim
h 0 x 0 x 0 x 0 x
and lim  f (a  h)  f (a)   0 ...(2) 1/x
1
h 0 e
= lim lim
1/ x = x 0 1/ x = 1.
Since L.H.D. exists at x = a, x 0 1  e e 1
f (a  h)  f (a) Since f (0– )  f (0+), f(x) is non-differentiable at x = 0.
f (a–) = lim exists. ...(3)
h 0 h To check continuity, we use the previous theorem.
 f (a  h)  f (a)  .(h) Since both the one-sided derivatives exist (though they
Now lim  f (a  h)  f (a)   lim
h 0 h 0 h are unequal), f is continuous at x = 0.
 f (a  h)  f (a)  We verify this result below :
 lim .lim(  h)  f '(a  ).0  0 .
h 0 h h 0
xe1/x
{using (3)} We have f(0–) = lim =0
Since R.H.D. exists at x = a, x 0 1  e1/x
3.18  DIFFERENTIAL CALCULUS FOR JEE MAIN AND ADVANCED

xe1/ x x Now, differentiability at x = 1,


f (0+) = lim lim
1/ x = x 0 =0 f(1  h)  f(1)
x 0 1 e 1  e 1/ x f (1–) = lim
Also f(0) = 0
h0 h
2
(1  h) sgn[1  h]  1  h  1
 f(0–) = f(0+) = f(0)  f is continuous at x = 0. = lim =1
h0 h
Show that the function f : R  R f(1  h)  f(1)
f (1+) = hlim
 0
 1 2  h
defined by f(x) = x 1  sin(ln x ) , x  0 and f(0) = 0 2
 3  (1  h) sgn[1  h]  {1  h}  1
is everywhere continuous, but has no differential = hlim
 0
h
coefficient at the origin. (1  h)2  h  1 h 2  3h
Let us first check the differentiability = lim = lim = 3.
h0 h h0 h
f (0 – h ) – f (0 ) Here, f (1+)  f(1–).
f (0) = lim
h0 –h Hence f(x) is non-differentiable at x = 1.
Now at x = 2,
(0 – h )1  1 sin (ln h 2 ) – 0
= lim
 3  lim f(x) = lim x2 sgn [x] + {x} = 4 . 0 + 1 = 1
x2 x 2
h 0 (– h )
lim f(x) = lim (sinx + |x – 3|) = 1 + sin 2
 1 2  x2 x 2
= lim 1  sin(ln h ) 
h 0  3  Since, L.H.L  R.H.L, f(x) is discontinuous at x = 2 and
= does not exist. hence it is non-differentiable at x = 2.
Hence f(x) is not differentiable at x = 0. We cannot say
anything about continuity immediately. It can be proved that a function f is con-
Now we check the continuity at x = 0 : tinuous from the left at those points where it is differen-
lim f(x) = lim f(0 + h) tiable from the left, and f is continuous from the right at
x0 h0 those points where it is differentiable from the right.
 1 2  Now, consider the the function,
= lim 1  sin(ln h )  = 0
h 0  3  f(x) = 2x x 3  1 + 5 x 1  x 4 + 7x2 x  1 + 3x
Finally f(x) is continuous at x = 0 but not differentiable
at x = 0. Here the domain of the function is x {1}
Since, it is a point function, there is no question of
 x 2 sgn[ x]  {x} , 0  x  2 continuity and hence differentiability at x = 1.
If f(x) =  ,
 sin x  | x  3 | , 2  x  4  f (x) is neither continuous nor differentiable at x = 1.
comment on the continuity and differentiability of f(x) Suppose that the function f is continuous everywhere.
at x = 1, 2. At how many points do you suspect that f can fail to
Looking at the kind of functions involved be differentiable ? One might think that if a function is
we check the continuity first. continuous on an interval, then it might fail to be
Continuity at x = 1 derivable at finitely many points at the most. This,
lim f(x) = lim (x2 sgn [x] + {x}) however, is far from the truth.
x 1 x 1 Whats the worst such function you can think of ?
=1 sgn (0) + 1 =1 Can a continuous function fail to have a derivative at
lim f(x) = lim (x sgn[x] + {x})
2 every point? The answer, surprisingly enough, is yes.
x  1 x 1
There exist functions which are continuous on R but
=1+0=1 which are not derivable at any point whatsoever. It did
f(1) = 1. give a jolt to mathematicians when, Weierstrass gave
Since, L.H.L = R.H.L = f(1), f(x) is continuous at x = 1. an example of such a function.
DIFFERENTIABILITY  3.19
Consider one of his functions f(x)  f(c)
 n e(x) =  f '(c)  0 as x  c
xc
f(x) =   23  cos (9 n x ), Conversely, given f(x) = f(c) + K(x – c) + e(x)(x – c),
n 0
where e(x)  0, then
a formula that expresses f as an infinite sum of cosines with
f(x)  f(c)
increasingly higher frequencies. By adding wiggles to = K + e(x)  K
wiggles infinitely many times, the formula produces a graph xc
Hence the derivative exists and equals K.
that is too bumpy in the limit to have a tangent anywhere.
Check the continuity and differen-
Another example of such a function defined on R is
 tiability of f(x) at x = 1, where f(x) = |x – 1| ([x] – {x}).
1 f (1  h )  f (1)
f(x) =  n cos(3 x), for all x  R,
n
f (1+) = lim
n 1 2 h0 h
It can be shown that f is continuous everywhere but is | h | ([1  h ]  {1  h})  0 h(1  h )
not derivable anywhere. The proof of these assertions = lim = lim 1.
h 0 h h0 h
is, however, beyond the scope of the present book. f(1)  f(1  h)
Remark In the definition of derivative we are f (1–) = lim
h0 h
regarding the derivative f (x0) as a real number
0  |  h | ([1  h]  {1  h})
associated with the difference quotient (f(x) – f(x0))/  lim
(x – x0) considered as a function which has values for h 0 h
each x in the domain of f with the exception of x0. In  h[0  (1  h )]
= lim  1
fact, the definition merely equates the ideas of h0 h
derivability of f at x0 with the notion of continuous Since f (1+) and f(1–) are not equal, hence f is not
extension of the difference quotient to x0. differentiable at x = 1. Since, the one-sided derivatives
exist, f is continuous at x = 1.
For if we write Q(x) = (f(x) – f(x0))/(x – x0), the domain of
Q(x) is Df – {x0} so that x0 is a removable discontinuity for x 2 1
Q(x), and we have the continuous extension Q*(x) defined Let f (x) = e x ,
x<0
to be Q(x) for x  x0 with Q*(x0) = f (x0). In symbols, = 0, x=0
= x2, x>0
 f (x) – f (x 0 ) Discuss continuity and differentiability of f (x) at x = 0.
 , x  D f – {x 0}
Q*(x) =  x – x 0 At x = 0
 f ' (x 0 ) , x  x0 f(0  h)  f(0)
Thus, for instance, if f is continuous on its domain, f (0–) = lim
h0 h
then Q(x) is continuous on the same domain minus x0, 1
  h  
1

but the continuous extension Q*(x) would be e  h
 0 = lim e– h · e h
continuous on the whole domain of f. The latter idea is = lim
h0 h 0
h h
expressed geometrically by rotating secant line 1

through a given point on a curve continuously to a e h 1
tangent line as a limiting line. = lim
h 0
put t = – we get
h h
Theorem Suppose f is differentiable at x = c. Then t t
f(x) = f(c) + f (c) (x – c) + e(x)(x – c), = lim t et = lim = lim
t  t  et t   et
where e(x)  0 as x  c and e(c) = 0. (applying L’ Hospital rule)
Conversely, let f be defined on an interval D and let = 0.
c  D. Suppose there is a constant K such that f ( h )  f ( 0) h2  0
f(x) = f(c) + K(x – c) + e(x)(x – c), where e(x)  0 as f (0+) = lim
h 0
= lim
h0
x  c. Then f is differentiable at c and f (c) = K. h h
f(x)  f(x) = lim
h0
h = 0.
Proof If lim  f '(c) exists, then
xc xc Hence, function is continuous and differentiable at x = 0.
3.20  DIFFERENTIAL CALCULUS FOR JEE MAIN AND ADVANCED

B
1. Consider the graph of a function f.

(a) For which numbers a does lim


x a
f(x) exist, but
f is not continuous at a ? 2  x if x  1
5. Let f(x) =  2 Is f differentiable
(b) For which numbers a is f continuous at a, but x  2 x  2 if x  1
not differentiable at a ? at 1? Sketch the graphs of f and f .
2. Consider the graph of a function f.  – 2x if x  1
6. Let f(x) =  . Show that f is
 x – 3 if x  1
continuous but not differentiable at x = 1.
 ax 2  1, x  1
7. If f(x)   2 is differentiable at
 x  ax  b, x  1
x = 1, then find a and b.
1 2
(a) For which numbers a does lim f(x) exist, but  sin x , x  0
x a
8. If f(x) =  x , discuss the continuity
f is not continuous at a ? 0, x0
(b) For which numbers a is f continuous at a, but and differentiability of f(x) at x = 0.
not differentiable at a ? 9. A function f is defined as follows :
3. Consider the graph of a function f. 1 for    x  0

f(x) =  1  sin x for 0  x  2 .

2   x  2  for 2  x   
 2 

Discuss the continuity and differentiability at


x = 0 and x = /2 .
 ex  x , x  0
2

(a) For which numbers a does lim f(x) exist, but 10. If f(x)   is differentiable at x = 0,
f is not continuous at a ?
x a
ax  b, x  0
then find a and b.
(b) For which numbers a is f continuous at a, but
not differentiable at a ? [2x]  x , x  1
11. If f(x) =  comment on the
4. The graph of f is shown. State, with reasons, the  {x}  1 , x  1
number at which f is not differentiable. continuity and differentiable at x = 1.

B
12. Check the continuity and differentiability of 13. Test the continuity and differentiability of the function
 –x, x  0  x(21x  2 –1x )
 ,x0
f(x) =  x 3 , 0  x  1 at x = 0 and x = 1.
2
f(x) =  (21x  2 –1x )
 x  x  1, x  1 0 , x0

at x = 0.
DIFFERENTIABILITY  3.21
14. Find the value of ‘a’ for which of f at x (denoted as f (x+)). If this exists, then it is
sgn(cos x) , x   / 2 considered as the derivative of the function at x.
f(x) =  is differentiable at Definition If a function f is defined only in the left
 a  sin x , x   / 2
x = /2. neighbourhood of a point x = a (for x  a), it is said to
15. Check the differentiability of the function be differentiable at a if
f (a  h)  f (a)
 2 cos x 
–1 L.H.D. = f (a–) = hlim exists.
f(x) = sin   at x = 0, . 0
h
 1  cos2 x  In such a case f (a) = f (a ).

16. Examine for continuity and differentiability at the Similarly, if a function f is defined only in the right
points x = 1 and x = 2, the function f defined by neighbourhood of a point x = a (for x  a), it is said to
 x [x] , 0 x 2 be differentiable at a if
f(x) =  f (a  h)  f (a)
(x  1) [x] , 2  x  3 R.H.D. = f (a+) = hlim exists.
0
where [x] = greatest integer less than or equal to x. h
17. Show that the function In such a case f (a) = f (a+).
3/2
For example, f(x) = x is differentiable at x = 0 but
sin x.cos(1 / x) at x  0,
f(x) =  g(x) = x is not differentiable at x = 0.
 0 at x  0.
Since, f and g are defined only in the right
is continuous at the point x = 0, but does not have
neighbourhood of a point x = 0, we calculate only R.H.D.
even one sided derivatives.
h 3/ 2  0
 sin x , x  1 f (0+) = hlim  lim h1/2  0 exists.
 0
h h 0
18. If f(x)   2 , where [.] denotes the Hence f (0) = 0.
[2x  3]x, x  1
h1/ 2  0
greatest integer function, then find whether it is g (0+) = hlim
0  lim h 1/ 2  
differentiable at x = 1. h h 0
does not exist.
3.5 DERIVABILITY AT Hence, x is not differentiable at x = 0.
ENDPOINTS
The four inverse trigonometric
f(x  x)  f(x)
Till now, the derivative f (x) = lim was –1 –1 –1 –1
functions sin x, cos x, cosec x, and sec x are not
x0 x
calculated when the function f was defined in a differentiable at the points x = ± 1.
–1
neighbourhood of the point x, where x tends to zero Consider f(x) = sin x.
attaining both positive and negative values. At x = 1, f is defined only in the left neighbourhood.
Suppose that a function is defined only in the left f (1  h)  f (0)
neighbourhood of x, then the above limit can be Hence, f (1–) = lim
h 0 h
calculated when x tends to zero running through
sin 1 (1  h)  sin 1 1
negative values (x < 0) only. This is termed as the left = lim
hand derivative of f at x (denoted as f (x–)). If this
h 0 h
1
exists, then it is considered as the derivative of the sin (1  h)   / 2  cos 1 (1  h)
= lim  lim
function at x. h 0 h h 0 h
Similarly, if a function is defined only in the right neigh- sin 1 2h  h 2
bourhood of x, then the above limit can be calculated = lim 
when x tends to zero running through positive values
h 0 h
–1
(x > 0) only. This is termed as the right hand derivative Thus, f(x) = sin x is not differentiable at x = 1.
3.22  DIFFERENTIAL CALCULUS FOR JEE MAIN AND ADVANCED
(ii) L.H.D. = f (a–) exists at the right endpoint b.
If a function is defined in a closed interval then, we by
derivatives at the end points of the interval, we always
mean the one-sided derivatives : the right hand
derivative at the left end point and the left hand
derivative at the right end point. This interpretation of
differentiability at the end points of the interval of
definition is analogous to that of continuity.
There is a vertical tangent to the graph of f at x = 1. For example, f(x) = x3/2 is differentiable in [0, 1].
Discuss the differentiability of the f(x) = sin x is differentiable in [0, 2].
function It should be noted that not all the elementary functions
f(x) = x  
x – ( x  1) at x = 0.
are differentiable everywhere in their domain.

Since domain of f(x) is [0, ), we find the


R.H.D. 1. All polynomial, exponential, logarithmic and
f (0  h )  f (0) trigonometric functions (inverse trigonometric not
 f (0+) = lim included) are differentiable at each point in their
h0 h
domain.
h( h – h  1) – 0
= lim
h 0
2. Modulus function and signum function are non
h differentiable at x = 0. Hence, y =  f(x)  and
lim
= h 0 ( h – h  1) = – 1 y = sgn(f(x)) should be checked at points where
f(x) = 0.
Hence, f is differentiable at x = 0. 3. The inverse trigonometric functions
–1 –1 –1 –1
y = sin x, cos x, cosec x, and sec x are not
3.6 DIFFERENTIABILITY differentiable at the points x = ± 1. Hence,
–1 –1 –1 –1
OVER AN INTERVAL y = sin (f(x)), cos (f(x)), cosec (f(x)), and sec (f(x))
should be checked at points where f(x) = ± 1.
f (x) is said to be differentiable in an interval if it is 4. Greatest integer function and fractional part
differentiable at each and every point of the interval. functions are non differentiable at all integral x.
Differentiability over an open interval Hence, y = [f(x)] and y = {f(x)} should be checked
at points where f(x) = n, n I.
A function f(x) is said to be differentiable in an open 5. Further, a function should be checked at all those
interval (a, b) if it is differentiable at all interior points points where discontinuity may arise.
in (a, b). x
For example, f(x) =  x – 1 is differentiable in (0, 1). Show that f(x) = is differen-
1 | x |
–1
y = sin x is differentiable in (–1, 1). tiable for all x  R.
Similarly, a function f(x) is said to be differentiable in  x
the open interval (a, ) or (–a) or (– , ) if it is 1  x , x  0

differentiable at all interior points in the interval. Since f(x) =  0 , x0,
For example, f(x) = x is differentiable in (0, ).  x
1  x , x  0
Differentiability over a closed interval
clearly f is differentiable for all x  R – {0}. We need to
A function f(x) is said to be differentiable in a closed check at x = 0.
interval [a, b] if
h
(i) it is differentiable at all interior points in (a, b).
f ( 0  h )  f ( 0 ) –0
(ii) R.H.D. = f (a+) exists at the left endpoint a. f (0+) = lim = lim 1  h
h 0 h h0 h
DIFFERENTIABILITY  3.23

1 1 2 4 3
= lim 1 x = 0, , , 1, , , 2.
h0 1  h 2 3 3 2
–h We check continuity at x = 1.
f ( 0 – h )  f ( 0 ) –0  1
f (0–) = lim = lim 1 h lim f(x) = lim x   [sin x] = 0
h 0 –h h0 –h x1 x1  3
1  1
= lim 1 lim f(x) = lim [2x] sgn x   = 2 {– 1} = – 2
h0 1  h x1 x1  3
 f (0+) = f (0–). Hence f is differentiable at x = 0. Since, L.H.L  R.H.L. f(x) is discontinuous at
Finally, f is differentiable for all x R. x = 1 and hence it is non-differentiable at x = 1.
Check differentiability of f(x) in
 1 [–2, 2], if
  x  3 [sin x] 0  x  2 cos  (| x | {x})
   x 1
Let f(x) =  f(x) =  2
[2x]sgn  x  4  1  x  2  4 x 2  12 x  9 {x} x 1
  3 
 where {.} denotes the fractional part function.
where [.] and {.} denote the greatest integer function
We have
and fractional part function respectively. Find the
points at which continuity and differentiability should cos  (| x | {x}) x 1

be checked. Also check the continuity and f(x) =  2
differentiability of f(x) at x = 1.  4 x 2  12 x  9 {x} x 1
 1 
cos (| x | {x}) x 1
  x  3 [sin x] 0  x  2 i.e. f(x) =  2
  | 2 x  3 | {x} x 1
f (x) = 
[2x]sgn  x  4  1  x  2 Now, we have
  3 
 |x| – {x} = |x| – x + [x] = –2x – 2, –2  x < –1
The suspicious points for continuity and = –2x – 1, –1  x < 0
differentiability are obtained when : = 0, 0x<1
1 and |2x – 3|{x} = |2x–3|(x – [x])
(i) x  becomes an integer for 0  x  2 = (3 – 2x)(x – 1), 1  x < 3/2
3
2 4 = (2x – 3)(x – 1), 3/2  x < 2
 x= , . = 0, x=2
3 3
(ii) sin x becomes an integer for 0  x  2 Thus, we have
1 3 cos  (2x  2),  2  x  1
 x = 0, , 1, .  2
2 2 
(iii) The point of change in definition is x = 1. 
cos (2x  1), 1  x  0
 2
(iv) 2x becomes an integer for 1  x  2 f(x) = cos 0, 0  x 1
3 
 x = 1, , 2. (3  2x)(x  1), 1 x  3/ 2
2 (2x  3)(x  1), 3/2  x 2
4 4 0, x2
(v) x  becomes zero for 1  x  2 x = .
3 3 We need to check differentiability of f(x) at
(vi) The end points of closed domain [0, 2] x = 0, 2. x = –2, –1, 0, 1, 3/2, 2. We first check continuity.
Hence, the points where we should check the At x = 2, we have
continuity and differentiability are f(2) = 0 and f(2–) = (2 . 2 – 3) (2 – 1) = 1.
3.24  DIFFERENTIAL CALCULUS FOR JEE MAIN AND ADVANCED
 discontinuity at x = 2.  non-differentiable at x = 3/2
At x = 3/2, we have Hence, f is differentiable on [–2,2] except at

3  x = –1, 0, 1, 3/2, 2.
3
f   f    0
  2 2 If x + 4 y = 6y, then find whether
3  
3  3  y as a function of x is continuous and derivable.
and f     3  2.   1  0 We have x + 4 y = 6y
  2  2  2 
 continuity at x = 3/2. 1 x , x0
At x = 1, we have f(1+) = f(1) = 0 and f(1–) = cos 0 = 1  2
 discontinuity at x = 1.  y = f(x) = 
At x = 0, we have  1 x , x 0
 10

f(0+) = f(0) = cos 0 = 1 and f(0–) = cos  0 1
2 lim f(x) = lim f(0 + h) = lim h=0
 discontinuity at x = 0. x0 h0 2
h0

At x = –1, we have 1
 lim f(x) = lim f(0  h) = lim (0  h) = 0
f(–1+) = f(–1) = cos  0 and f(–1–) = cos0 = 1 x0 h0 h  0 10
2  f is continuous at x = 0 and hence for all x  R.
 discontinuity at x = –1.
At x = –2, we have f(–2+) = f(–2) f  0  h   f (0 )
f (0+) = lim
 continuity at x = –2. h0 h
Hence, we need to check differentiability only at h 0 1
x = –2, 3/2. = lim 2 =
At x = –2 which is the left endpoint, we have
h0 h 2
f ( 2  h )  f ( 2) f  0  h   f (0)
f (–2+) = lim f  (0–) = lim
h 0 h h0 h
  h 0
 10 1
cos ( 4  2 h  2)  cos ( 4  2)
2 2 = lim =
= lim h0 h 10
h 0 h
 f is not derivable at x = 0.
cos (1  h)  1  sin(   h)
= lim  lim 0  f is derivable at all x except x = 0.
h 0 h h 0 1
 f is differentiable at x = –2.
At x = 3/2, we have
 1  x, x  1
3 3
f   h   f   | x 2  1|,
 1  x  0
 3 
f '    lim  2  2 If f(x) =  ,
 2  h0 h  k(  x  1), 0  x 1
3 | x  1|, x 1
2h   h  1  1
 lim  
2 then find the value of k so that f(x) becomes continuous
1
h 0 h at x = 0. Hence, find all the points where the functions
3 3
f    f   h 
is non-differentiable.
 3 
f '    lim  2   2  The only possible value of k is 1 which
  2 h 0 h is obtained by equating the one-sided limits at x = 0.
3
0  2h   h  1 Now, we draw the graph of y = f(x).
 lim  2   1 From the graph, it is clear that the function is non-
h 0 h differentiable at x = 0, ±1 because at each of these
DIFFERENTIABILITY  3.25
points the graph is having a corner. Graph of y = Min {|x|, |x – 2|, 2 – |x – 1| :

If f(x) = |x + 1| (|x| + |x – 1|), then draw


the graph of f(x) in the interval [–2, 2] and discuss the It is clear from the figure that f(x) is continuous  x  R
continuity and differentiability in [–2, 2] and non-differentiable at x = – 1 , 0, 1, 2, 5 .
Here, f(x) = |x + 1| ( |x| + |x – 1| ) 2 2
Consider the function
 (x  1)(2x  1), 2  x  1
 (x  1)(2x  1), 1  x  0 f(x) = min(|x|, 1  x 2 ),–1  x  1

= = [x], –1 < |x|  2
 (x  1), 0  x 1 Plot the curve and discuss its continuity and
 (x  1)(2x  1), 1 x  2 differentiability.
The graph of f(x) is

y = x, x  (–2, –1)  (1, 2)


 y = 1.
which is clearly, continuous for x  R. The curve shown by thick lines represents the curve.
The graph has corners at x = –1, 0, 1. From the graph, we can see that f(x) is
Hence, f(x) is differentiable for x  R – {–1, 0, 1}. continuous on x  (–2, 2) –  1 and
1 
If f(x) = Min {|x|, |x – 2|, 2 – |x – 1|}, differentiable on x  (–2, 2) – 
 1, ,0  .
then draw the graph of f(x) and also discuss its  2 
continuity and differentiability. Let f (x) = x3 – x2 + x + 1 and
f(x) = Min {|x|, |x – 2|, 2 – |x – 1|}
max {f ( t ); 0  t  x}, 0  x  1
We compare the graphs of g(x) =  .
y = |x|, y = |x – 2| and y = 2 – |x – 1| and  3  x; 1 x  2
choose the least value at each x. Discuss the continuity and differentiability of the
function g(x) in the interval (0, 2).
f(t) = t3 – t2 + t + 1

 f ( t )  3t 2  2 t  1
Its discriminant = (– 2)2 – 4.3.1 = – 8 < 0
and coefficient of t2 = 3 > 0
Hence f ( t )  0 for all real t.
 f(t) is strictly increasing
Thus f(t) is maximum when t is maximum and tmax = x
 max f(t) = f(x)
3.26  DIFFERENTIAL CALCULUS FOR JEE MAIN AND ADVANCED

 x3  x 2  x  1, 0  x  1 1 if x  0

 g(x) =  Since f (x) = 0 if x  0
3  x, 1 x  2
1 if x  0
Now it can be easily seen that f(x) is continuous in 
(0, 2) and differentiable in (0, 2) except at x = 1 because 1 if g(x)  0

at x = 1. LHD > 0 while RHD = – 1 < 0. f(g(x)) = 0 if g(x)  0
 1 if g(x)  0
Check the differentiability of the 
function f (x) = max {sin–1 |sin x|, cos–1 |sin x|}. 1 if x  ( , 1)  (0,1)

0 if x  0,1,  1
sin–1 |sin x| is periodic with period  = 
 1 if x  ( 1,0)  (1, )
ì p 
ïx , np £ x £ np + The graph of y = f(g(x)) is shown below
ï 2
 sin–1 |sin x| = í
ï p
ïp - x , np + £ x £ np + p
î 2

Also cos–1 |sin x| = – sin–1 |sin x|
2



 x,   x
 2
f (x)= max 
 , n  x  n 

2
From the graph we can easily conclude that f(g(x)) is
discontinuous and non derivable at – 1, 0, 1.


   x, x  
 2
 , n 

2
 x  n  
Now, g(f(x)) = f (x) ( 1 – f 2(x) )
= sgn x (1 – sgn2 x) = 0 for all x,
Hence g (f(x)) is continuous and derivable for all x.
   x, n  x  n 

2 Find the interval of values of k for
4
 which the function f(x) = |x2 + (k – 1) |x| – k| is non
 x,  
n   n  differentiable at five points.
 4 2

 3
 f (x) =    x, n   x  n  f(x) = |x2 + (k – 1) |x| – k| = |(|x| – 1) (|x| + k)|
 2 4 Also f(x) is an even function and f(x) is not
  3
x  , differentiable at five points. So |(x – 1) (x + k)| is non
n   x  n  
 2 4 differentiable for two positive values of x.
  3
 f (x) is not differentiable at x = 0, , , , ...
4 2 4
n
 f (x) is not differentiable at x = .
4
Let f (x) = sgn x and g (x) = x (1 – x2).
Investigate the composite functions f(g(x)) and g(f(x))  Both the roots of (x – 1) (x + k) = 0 are positive.
for continuity and differentiability.  k < 0  k  (–, 0).

c
1. Indicate the points at which the following 2. Show that for any positive integer n, the root
functions have no finite derivative : function f(x) = x1/n is not derivable at the origin. Is
(a) y = cos–1 x  1 (b) y = sin–1 1 it continuous at the origin ?
2 x 3. (a) Find f (0–)and f (0+) given f(x) = |x|.
DIFFERENTIABILITY  3.27
(b) The function f(x) = |3x – 10| is differentiable 6. Discuss the continuity and differentiability of the
except at a single point. What is this point, function f(x) = sinx + sinx , x R. Draw a rough
and what are the values of its left hand and sketch of the graph of f(x) .
right hand derivatives of f there ? 7. (a) Sketch the graph of the function f(x) = x|x|.
4. Sketch the graph of the function f(x) = x + |x|. Then (b) For what value of x is f differentiable?
investigate its differentiability. Find the derivative (c) Find a formula for f (x).
f (x) where it exists. Also find the one-sided 8. If f(x) = [x] + [1 – x] = – 1  x  3, Draw its graph and
derivatives at the points where f (x) does not exist. comment on the continuity and differentiability
5. Investigate the differentiability of the functions: of f(x).
(i) f(x) = |x3| 9. Show that the Dirichlet function is nowhere
derivable.
 12 if x  3,
 10. Examine the differentiability of
(ii) f(x) =  (5  x)2
 x 2  3x  3 if x  3  xe|x| , x  1
f(x) =  2 .
ex , x  1

C
x 1 1 1  x , ( 0  x  1)
11. Let f(x) = for x  0, f(0) = 0. Is the 
x 17. f(x) =  x  2 , (1  x  2 ) .
function f(x) continuous and differentiable at x = 0? 4  x , ( 2  x  4 )
12. Investigate the differentiability of the function 
Discuss the continuity and differentiability of
11  6x  x 2 if x  3 y = f (f(x)) for 0  x  4 .
f(x) =  2 .
 x  6x  29 if x  3 18. Draw the graph of f(x)=max.{sin x, cos x, 1 – cos x}
 min{x, x 2}, x  0 and find the number of points belonging to
13. If f(x) =  2 then find the (0, 2) where f(x) is non differentiable.
 max{2x, x – 3}, x  0
points of non differentiability of f(x). 0 if x is an int eger
19. Let f(x) = [x] and g(x) =  then
14. Find a and b, given that the derivative of f(x) exists 2
 x otherwise
everywhere, when define g o f and f o g and examine their
ax 2  bx  2, x  3 differentiability.
f(x) =  2 . 20. Let f(x) = cos x and
 bx – 3 , x  3
15. Check differentiability of f(x) at x = 0,  minimum {f (t) ; 0  t  x}, x  [0, ]
g(x)   .
 1 2   sin x 1, x
where f(x) = x 1  sin(ln x ), x  0 Draw the graph of g(x) and comment on
 3 
= 0, x=0 differentiability.
where [.] represents the greatest integer.
16. Copy the graph of the function. Then sketch a 3.7 ALTERNATIVE LIMIT
graph of its derivative directly beneath. FORM OF THE DERIVATIVE
The following is an alternative limit form of the
derivative. The derivative of f at x = a is
f (a  h)  f (a) f (x)  f (a)
f '(a)  lim = lim
h 0 h x c x a
f (x)  f (a)
f (a) = lim .
x a x–a
3.28  DIFFERENTIAL CALCULUS FOR JEE MAIN AND ADVANCED
The use of this formula simplifies some derivative
calculations.
Y y = f(x) secant slope is
f(x) – f(a) f (a  h)  f (a  h)
x–a
The quotient may have a limit as
Q(x), f(x)) 2h
h  0 when f has no derivative at x = a.
f(x) – f(a)
For instance, if f(x) = |x| then
P(a, f(a))
| 0  h | ( 0  h |
lim = 0.
h=x–a h 0 2h
As we can see, the limit exists even though f(x) = |x| has
a x X no derivative at x = 0.
The one-sided derivatives of f at a are given by f (a  h)  f (a  h)
In fact, lim
f(x)  f(a) f(x)  f(a) h 0 2h
f (a+) = lim and f (a–) = lim .
x a x a x a x a 1  f (a  h)  f (a) f (a  h)  f (a) 
f (a) exists if they are equal. = lim 
2 h 0  h h 

If f is a differentiable function with
1
f (x)  f (2) = (f (a+) + f (a–)).
f (2) = 3 then find the value of lim . 2
x 2 x 2 We notice that it is equal to f (a), if f is differentiable at
f (x)  f (2) x 2 x = a.
lim ·
x 2 x 2 x 2 Hence, if f (a) exists, then
f(x)  f(2) f (a  h)  f (a  h)
= lim
x2 x2
· x 2  lim
h 0 2h
= f(a).

= f (2) · 2 2 = 6 2 . If f  (5) = 7 then find the value of

Centred difference quotient f (5  t )  f (5  t )


lim .
t0 2t
The centred difference quotient
f (a + h) - f (a - h) f (5  t )  f (5  t )
lim
2h t0 2t
is used to approximate f (a) in numerical work because f (5  t )  f (5  t )
(i) its limit as h  0 equals f (a) when f (a) exists, and = 7 . lim
(ii) it usually gives a better approximation of f (a) for a t0 2t
given value of h than Fermat's difference quotient  f (5  t )  f (5)  f (5  t )  f (5) 
= Limit
f (a  h)  f (a) t0  2t 2t 
h f  (5 ) f  (5 )
= 2
+ 2 = f  (5) = 7.

(i) If f (a) exists and (h)  0 as h  0, then


f (a  (h))  f (a)
lim = f(a).
h 0 (h)
(ii) If f (a) exists, and (h)  0 and (h)  0 as
h  0, then
DIFFERENTIABILITY  3.29

f (a  (h))  f (a  (h)) h(2  h) (1  h  h 2 )  1


lim = f(a).  
h 0 (h)  (h) h(1  h) f (1  h  h 2 )  f (1)
If f (a) = 3, evaluate the limit 2h 1 1
= f (2) × lim  = 6 × 2 × = 3.
h 0 1  h f (1) 4
lim f(a  2h)  f(a  3h) .
h 0 h If f (2) = 4 then find the value of

lim f(a  2h)  f(a  3h)


2
lim f (2 + h ) - f (1 + cos h) .
h 0 h h 0
h2
f(a  2h)  f(a  3h) 2
= 5 . lim
h0
lim f (2 + h ) - f (1 + cos h)
5h h 0
h2
= f (a) × 5 = 3 × 5 = 15. f (2 + h 2 ) - f (2 + cos h - 1) . h 2 - (cos h - 1)
= lim
h0 2 2
If f is a differentiable function and h - (cos h - 1) h
now as h  0, h2  0 and (cos h – 1) 0
f  (   h)3   f ( 3 ) h 2 - (cos h - 1)
lim = , = f (2) × lim
h0 h h 0
h2
then find the value of f (3). 3
f  (   h)3   f ( 3 ) = 4 × = 6.
2
= lim ...(1)
h 0 h Differentiability of parametric functions

= lim
 3 2 2
f   (3h   3 h  h )  f (  )3
 3
Let the function y = f(x) be defined by
x= x(t) and y= y(t), where t is the parameter.
h 0 h
Suppose we are interested in the derivative of y = f(x)
where ( 3h 2   3 2 h  h 3 ) 0 as h  0. and the possibility of its existence.
f((   h)3 )  f ( 3 ) (3h 2   32 h  h 3 ) For parametric functions, the derivative is defined as
= lim  lim
h 0 (3h 2   3 2 h  h 3 ) h 0 h dy
= f  (3) · 32. dy dt
f (x) = 
1 dx dx
 = f  (3) · 32 f  (3) = .
dt
3
Alternative : The issue of differentiability is investigated using the
Using LHospitals rule in (1), we get existence of the limit :
lim f '  (   h)3  ·3(   h)2 =  y(t  )  y(t)
h0 dy  y(t  )  y(t)
 f  (3) · 32 =   lim  lim
dx 0 x(t  )  x(t) 0 x(t  )  x(t)
 1 
f  (3) = 2 = .
3 3 The one-sided derivatives of f at x = x(t) are given by
y(t  )  y(t)
Given f (2) = 6 and f (1) = 4, evaluate
L.H.D. = f (x–) = lim 
f(2h  2  h 2 )  f(2) 0 x(t  )  x(t)
lim .
h  0 f(h  h 2  1)  f(1) 
y(t  )  y(t)
f(2h  2  h 2 )  f(2) = lim
0 x(t  )  x(t)
...(1)
lim
h 0 f(h  h 2  1)  f(1)
y(t  )  y(t)
f (2  2h  h 2 )  f (2) 
= lim lim
h 0 2h  h 2 R.H.D. = f (x+) = 0 x(t  )  x(t)

3.30  DIFFERENTIAL CALCULUS FOR JEE MAIN AND ADVANCED

y(t  )  y(t) y(0  )  y(0)


= lim ...(2) L.H.D. = f (1–) = lim
0 x(0  )  x(0)
0 x(t  )  x(t)
provided x = x(t) increases with increase in t. []3  0
= lim  0.
If x = x(t) decreases with increase in t, then (1) evaluates 0 1  3  1
the R.H.D. and (2) evaluates the L.H.D. Hence, f is non-differentiable at x = 1.
As before, f (x) exists when both (1) and (2) exist and
are equal. The function f is defined by y = f(x)
Another way is to eliminate t to find the function y in where x = 2t – |t|, y = t2 + t |t|, t  R. Discuss the
terms of x and then proceed as usual. continuity and differentiability at x = 0.
The function f is defined by y = f(x) Here, we eliminate t and get the function
y = f(x) first.
where x = 2t – |t – 1|, y = 2t2 – t |t|, t  R. Discuss the
If t  0 we have x = t, y = 2t2,
differentiability of y = f(x) at x = 2.
hence y = 2x2, x  0
We solve the equation x = 2 to get the
and for t < 0, x = 3t, y = 0, x < 0
corresponding value of t.  The function is defined as
2t – |t – 1| = 2  t = 1.
We notice that at t = 1, x = x(t) increases with increase 2 x 2 , if 0  x  1
f(x) = 
in t. Hence,  0 , if – 1  x  0
y(1  )  y(1) f (0–) = 0
L.H.D. = f (2–) = lim
0 x(1  )  x(1) 2h 2  0
2 f (0+) = lim 0.
2(1  )  (1  ) |1   | 1 h 0 h
= lim It is clear that f(x) is differentiable and hence
0 2(1  )  |1    1| 2
continuous at x = 0.
 2   2 2
= lim = .
 0 3 3 Differentiability using derivative of functions
y(1  )  y(1) Consider the function f(x) = x4/3ex. Let us find f (0)
R.H.D. = f(2+) = lim
 0 x(1  )  x(1) from the formula of its derivative. We differentiate f(x)
2(1  ) 2  (1  ) |1   | 1 using product rule (to be discussed later).
= lim 4
 0 2(1  )  |1    1 | 2 f (x) = x4/3ex + x1/3ex ...(1)
3
2   2 To find f (0), even when we donot know whether it
= lim = 2.
 0  exists , we put x = 0 into the formula (1). We get
 f  (2–)  f  (2+) and hence f  (2) does not exist. f (0) = 0. This provides the result in a simple way
The function f is defined by y = f(x) Now, if f(x) = x1/3 sin x ,
where x = 1 – t3, y = [t]t 3, t  R. Discuss the 1
f (x) = x1/3 cos x + sin x · x–2/3
differentiability of y = f(x) at x = 1. 3
The corresponding value of t is, sin x
= x1/3 cos x + ...(2)
1 – t = 1  t = 0
3 3x 2 3
Here, f (x) is not defined at x = 0. Do we conclude that
We notice that at t = 0, x = 1 – t3 decreases with increase
in t. Hence, f (x) is not differentiable at x = 0 ? The answer is no.
In such cases, we should try to find the derivative using
y(0  )  y(0)
R.H.D. = f (1+) = lim
0 x(0  )  x(0)
its basic definition (i.e. first principles). The derivative
may exist even when the formula f (x) is undefined.
[ ]( )3  0 f(0  h) – f(0)
= lim 1 Now, f (0) = = lim
 0 1  ( )3  1 h0 h
DIFFERENTIABILITY  3.31

h1/3 sinh – 0 If f(x)=xx, then find its derivative.


= lim = 0.
h 0 h f (x) = x x
 f (0) = 0. The function f is differentiable at x = 0.   x 2 x  0
There is another point to be noted. We find the limit of f (x) =  2 ...(1)
f (x) given by (2) as x  0,  x x0
We see that f is continuous.
sin x
lim f(x) = lim ( x1/3 cos x + ) = 0.  2 x x  0
x 0 x 0
3x 2 3 f  (x) =  ...(2)
We are now tempted to take the limit of the derivative  2x x0
Note that the equality sign at x = 0 has been removed
( lim
x 0
f (x)) as the value of the derivative (f (0)). Both because the existence of f  (0) is in doubt.
are equal to 0 in this case. We can see that lim f '(x)  lim ( 2x)  0
x 0 x 0
Suppose that the real valued function and lim f '(x)  lim 2x  0 .
x 0 x 0
f is continuous on R and differentiable on R–{0}, and Thus, at x = 0, L.H.D. = R.H.D. = 0,
that so we have f  (0) = 0.
lim f (x) = L. Now, we extend the definition of f  (x) at x = 0.
x 0
Prove that f is differentiable at 0 and that f (0) = L.  2 x x  0
We are required to prove that Hence, f (x) = 
 2x x0

lim
f ( x ) – f (0 )  f  (x) = 2 x.
exists and equals L.
x  x0
Let F(x) = f(x) – f(0) and G(x) = x.
If we consider the function
Then, since f is continuous, lim F(x) = 0, and of course
x 0   x 2 x0
lim G(x) = 0 too. g(x) =  2
x 0  x  1 x  0
However, F(x) = f (x) and G(x) = 1, and proceed as above, we get
F' (x )  2 x x  0
so xlim exists and equals L. g (x) = 
 0 G' ( x )
 2x x  0
F' (x ) Here, also we get xlim g(x) = 0 and xlim g (x) = 0.
By L Hospitals rule, lim = L too,  0  0
x 0 G ' (x)
Since the function is discontinuous at x = 0, there is no
f ( x ) – f (0 ) question of differentiability. However, if the above limits
i.e. lim = L, as required.
x  x0 are considered as the L.H.D. and R.H.D. then it would
In general, the limit of the derivative may not exist, but be misleading.
when it exists then it is equal to the value of the derivative. In fact when we calculate L.H.D. and R.H.D. using first
In such a case, we say that the derived function f (x) is principles, we get L.H.D. =  and R.H.D. = 0.
continuous, or the function f(x) is continuously Hence, we should check the continuity of the function
differentiable. in advance.
In most of the differentiable functions used in routine, Using the following example, we now show that the
the limit of the derivative exists and this gives us an derivative of a continuous function is not always a
opportunity to find the value of the derivative using continuous function.
the limit of the derivative. If f(x) = x2 sin (1/x) when x  0 and
But, it must be checked before differentiating the f(0) = 0, show that f is differentiable for every value of
function that it is continuous. x but the derivative is not continuous for x = 0.
3.32  DIFFERENTIAL CALCULUS FOR JEE MAIN AND ADVANCED

For x  0,

1  1  1   x tan 1 x  sec1 1 x  , x (1,1)  {0}


f (x) = 2x sin + x2 cos    – 2  If f(x) =  ,
x  x  x   2 if x  0
1 1 then find f (0) , if it exists.
= 2x sin – cos .
x x
At x = 0, we have
x 1  1 
x 2 sin 1 f (x) = + tan–1x +  2 
f ( x ) – f ( 0 ) x 1 x2 1 1  x 
f (0) = xlim = xlim 1
0 x0  0 x x x2

= xlim x sin
1
= 0. x | x |2  1 
0 x = –1
+ tan x +  2 
Thus the function possesses a derivative for every
1 x2 1 x2  x 
value of x given by lim f(x) = – 1 and lim f(x) = – 1
x  0 x 0
1 1 Hence f (0) = – 1
f (x) = 2x sin – cos when x  0, f (0) = 0.
x x Alternative :
We now show that f  is not continuous for x = 0. Using first principles :
 1 1 h tan 1 (h)  sec 1 1 h    2
Here lim f (x), i.e., lim  2 x sin – cos  does not f (0+) = lim
x 0 x 0  x x h0 h
 1 
exist since lim  2x sin   0 exists but lim cos 1 (h)   2  sin 1 (h)
x0  x x0 = lim = lim =–1
h 0 h h 0 h
 cos 1  Similarly, f (0+) = – 1
  does not exist.
 x Hence, f (0) = – 1.
Since, xlim
0
f (x) does not exist, f  is discontinuous at (b) If lim f (x)  lim f (x), then f(x) is non-
x a x a
x = 0. In other words, f(x) is differentiable but not differentiable and its derivative f (x) is
continuously differentiable. discontinuous at x = a.
If the limit of the derivative does not exist, does it  2x 
mean that the function f(x) is not differentiable ? The Let f(x) = cos–1   . Define
 1  x2 
answer is no. This is proved by the above example.
f (x) stating clearly the points where f(x) is not
So, the question arises that in what circumstances can differentiable.
we use limit of the derivative in deciding the
differentiability of the function ?  2x 
f(x) = cos–1  , xR
 1  x2 
Assuming that the function f(x) is continuous at x = a,
two cases arise: 1 2(1  x 2 )  4x 2
f (x) = – ·
(i) If lim f (x) and lim f (x) both exist or are infinite 2x 
2 (1  x 2 )2
x a  x a 1   
 1  x2 
then lim f (x) = f (a–)
x a 2(1  x 2 )
i.e L.H.L. of f (x) = L.H.D. of f(x) at x = a f (x) = –
| 1  x 2 | (1  x 2 )
and lim f (x) = f (a+) lim f(x) = – 1 and lim f(x) = 1
x a
x1 x1
i.e R.H.L. of f (x) = R.H.D. of f(x) at x = a.
Since lim f (x) and lim f (x) are unequal, f (1) does
(a) If lim f (x) = lim f (x), then f(x) is differentiable x1 x1
x a x a not exist.
and its derivative f (x) is continuous at x = a. Similarly f (–1) does not exist.
DIFFERENTIABILITY  3.33

 2 Calculate the derivative of the


 1  x 2 if  1  x  1
 function
Hence, f (x) = non existent if x   1
 2 f(x) = x 2  2x  1 on the interval [0, 2].
 if x  1 or x  1 Since the expression under the radical
1  x 2 sign is a perfect square, we can, according to the
(c) If lim f (x) and lim f (x) are infinite, then f(x) definition of the modulus, represent the given function
xa x a
has an infinite derivative at x = a and hence it is in the following form :
non-differentiable there and f (x) is discontinuous
at x = a.
| x |x , x  0
1  x, 0  x  1,
f(x) = |x – 1| = x  1, 1  x  2

Suppose f(x) =  ,
Differentiating f(x) separately on the intervals [0, 1)
 and (1, 2], we obtain
1, x0
then find whether f(x) is differentiable at x = 0.
f (x) = 1, 0  x  1,
1, 1x 2
For x > 0, f (x) = xx lim f (x) = – 1 and lim f (x) = 1
x 1 x 1
and f (x) = xx(1 + ln x) ...(1)
For x < 0, f (x) = (–x)x = ex ln (–x) Since the left and right derivatives of f (x) at the point
and f (x) = (–x)x[1 + ln (–x)] ...(2) x = 1 do not coincide, the derivative does not exist at
It can be shown using L'Hospital's rule that x = 1. We take the values of the left hand derivative of
lim x x = 1 and lim (  x) x = 1 function at the point 2 and of its right hand derivative
x 0 x 0 at the point 0 as the values of f (x) at the endpoints of
Now, lim f(x) = –  and lim f(x) = –  the interval [0, 2].


x 0 x 0
1, x  [0,1),
lim f '(x) = –  Thus f (x) =
x 0 1, x  (1,2]
Hence f is not differentiable at x = 0.
Check the differentiability of the
(ii) If any of the limits xlim
a 
f (x) or xlim
a 
f (x) does
function f(x) = 1  1  x 2 .
not exist, then we cannot conclude anything about
2
the differentiability of the function. In such a case, The function f(x) = 1  1  x is
we should try to find the derivative using its basic defined for x  [–1, 1]. It is continuous in its domain.
definition (i.e. first principles). Differentiating w.r.t. x, we have
In the previous example, we have seen this situation : 1 1
f(x) = x2 sin (1/x) when x  0 and f(0) = 0, f (x) = . . –2x
2
1 1 2 1 1 x 2 2 1  x
For x  0, f (x) = 2x sin – cos . x
x x =
 1 1 2 1  x 1  1  x2
Here lim f (x) = lim  2 x sin – cos  does not exist.
x 0 x 0  x x The formula of f (x) is undefined at x = – 1, 0, 1.
However, using first principles, we have The differentiability of the function should be checked
1 at x = – 1, 0, 1.
f (h) – f (0) h 2 sin We can see that
f (0) = lim = lim h
h 0 h h 0 h lim f ' ( x)   and lim f ' ( x)   .
x  1 x  1
1
= lim
h 0
h sin = 0. Hence, f has an infinite derivative at x = – 1 and 1.
h To check the existence of f (x) at x = 0, we find whether
Hence, the function f is differentiable at x = 0, even if
the limit of its derivative does not exist. lim f ' ( x) exists or not.
x0
3.34  DIFFERENTIAL CALCULUS FOR JEE MAIN AND ADVANCED
x x
We have lim  lim lim f(x) = 2B and lim f (x) = 3A
x0 x0 x1 x1
1  x2 1  1  x2 1  1  x2
 3A = 2B ...(2)
1 1 x 2 2B
= lim x From (1) and (2), –B+1=0
x0 x2 3
B
2 x  2 as x  0   – = – 1 B = 3
= lim  3
x0 | x |
 2 as x  0
  A = 2.
= does not exist.  |1  4x 2 | , 0  x  1
Hence, f is non-differentiable at x = 0. If f(x) =  2
Thus, the function f is differentiable in its domain [x  2x] , 1  x  2
where [.] denotes the greatest integer function.
except at x = –1, 0, 1.
Discuss the continuity and differentiability of f(x) in [0, 2).
Alternative approach at x = 0 using first principles.
Since 1  x < 2 we have0  x – 1 < 1,
f (0–) = lim 
f ( 0 – h ) – f ( 0) 
  and [x – 2x] = [(x – 1)2 – 1] = [(x – 1)2] – 1
2
h 0  –h  = 0 – 1 = –1
 1 – (1 – h 2 ) – 0 
= lim   1  4x 2 , 0  x  1
  
h0
 – h  2

 f(x) =  2 1
1 – (1 – h 2 )

1  (1 – h 2 )  4x  1 , 2  x  1
= lim 
h 0 –h 1  1 – h2  1 , 1 x  2
h 1 Hence, the graph of f(x) is :
= lim =–
h0
(– h) 1  (1 – h 2 ) 2 Y
f(0  h) – f(0) 3
and f (0+) = lim
h 0 h
1 – (1 – h 2 ) – 0 1
= lim
h 0 h
O 1/2 1 2 X
1 – (1 – h 2 ) 1  (1 – h 2 )
= lim  –1
h 0 h 1  1 – h2 It is clear from the graph that f(x) is discontinuous at
h 1
= lim  1
h 0
h 1  (1 – h ) 2 2 x = 1 and not differentiable at x = and x = 1. To verify
2
 f (0–)  f (0+), f(x) is not differentiable at x = 0. these details we calculate the derivative of f :
 A  Bx 2 , x 1
If f(x) =  find 8x, 0  x  1 / 2
 3Ax  B  2 , x  1 
the value of A and B so that f(x) is differentiable at x = 1. f (x) =  8x, 1 / 2  x  1
Since f(x) will be differentiable at  0, 1 x  2

x = 1, it must be continuous,
This shows that f(x) is not differentiable
 3A – B + 2 = A + B
at x = 1/2 (as RHD = 4 and LHD = –4)
 A–B+1=0 ...(1)
and x = 1 (as RHD = 0 and LHD = 8)
 2Bx , x  1
Now, f (x) =  Therefore, f(x) is differentiable  x  [0, 2) – {1/2, 1}.
3A , x  1
DIFFERENTIABILITY  3.35

D
If it happens that the first derivative has a derivative on
sin( 3  x ) 2 – sin 9
1. Evaluate lim . (a, b) then this derivative is called the second derivative
x 0 x of y w. r. t. x and is denoted by f (x) or (d2y/dx2) or y
2. Find a function f and a number a such that
Once we have found the derivative f  of any function
(2  h )6 – 64 f, we can go on and find the derivative of f.
lim  f ' (a ) .
h 0 h f (a  h)  f (a)
3. Each of the following limit represents the derivative f(a) = lim .
h 0 h
of some function f at some number a. State such Similarly, the third order derivative of y w. r. t. x, if it
an f and a in each case.
d3y d  d 2 y 
x cos (  h)  1  
(i) lim 2  32 (ii) lim exists, is defined by
d x3 dx  d x 2 
 . It is also
h 0 x  5 h 0 h
denoted by f (x) or y .
x2 x0 The third derivative, written f, is the derivative of
4. Let f(x) =  2 . the second derivative, and, in principle, we can go on
x x0
Prove that f is differentiable. Is f is continuous? forever and form derivatives of higher order. We adopt
the alternative notation f(n) for the nth derivative of f.
1 f(a  2h 2 )  f(a  2h 2 ) Notice also that for derivatives higher than the third,
5. If f (a) = , find lim .
4 h0 h2 the parentheses distinguish a derivative from a power.
6. If f (2) = 4 then find the value of For example, f 4  f (4).
f(2 + h) – f(2 + sin h) Twice differentiability
lim .
h 0 h. sin h.tan h Suppose that a function y = f(x) defined in an interval
7. Discuss the differentiability of the function has the derivative f (x) with respect to the independent
2
y =f(x) defined as x = 2 t + | t | and y = [ t] t at x = 0. variable x. If the function f (x) is further differentiable,
its derivative is called the second derivative (or the
 3x 2 , x  1
derivative of the second order) of the original function
8. Let f(x) =  . Find the values of a
ax  b, x  1 f(x) and is denoted f (x).
and b so that f will be differentiable at x = 1. A function f(x) is twice differentiable at x = a if its
derivative f (x) is differentiable at x = a i.e.
 x 2 , x0 f (a  h)  f (a)
9. Let f(x) =  2 . Show that the limit f (a) = lim exists.
x  1, x  0 h 0 h
lim f(x) = lim f (x) but that f (0) does not exist. f '(x) – f '(a)
x 0 x0  Alternatively, f (a) = lim .
x a x–a
3.8 DERIVATIVES OF The value f (x) of the second derivative at a point x
characterizes the rate of change of f (x) at that point,
HIGHER ORDER that is the rate of change of the rate of change of f(x).
Because the derivative of a function is a function, By analogy with mechanics, we can say that f"(x) is
differentiation can be applied over and over, as long the acceleration of the change of the function f(x) at
as the derivative itself is a differentiable function. the given point x.
Let a function y = f(x) be defined on an open interval The geometrical meaning of the second derivative will
(a, b) . Its derivative, if it exists on (a, b) is a certain be discussed later.
function f (x) [or (dy/dx) or y ] and is called the first Further, the nth derivative of f(x) at x = a exists if
derivative of y w. r. t. x. f (n –1) (x) – f (n –1) (a)
f (n)(a) = [f (n–1)] (a) = lim exists.
x a x–a
3.36  DIFFERENTIAL CALCULUS FOR JEE MAIN AND ADVANCED

Find whether the function f(x) = x4/3 Let f be defined in a neighbourhood


sinx is twice differentiable at x = 0. of x and f (x) exists.
Let us find f (x) first. We differentiate f(x  h)  f(x  h)  2f(x)
Show that lim  f "(x)
f(x) using product rule. h 0 h2
Show by an example that the limit may exist ever if f (x)
4 1/3
f (x) = x4/3cos x + x sin x ...(1) does not exist.
3
f ' ( x  h ) – f ' ( x)
f '(0  h) – f '(0) f (x) = lim
Now, f (0) = lim h0 h
h 0 h  f (x  h  k) – f (x  h) f (x  k) – f (x) 
lim  – 
4 k 0  k k
h 4/3 cos h + h1/3 sin h – 0 = lim
3 h 0 h
= lim
h®0 h Let k = – h
f(x) – f(x  h) – 2(x  h)  f(x)
æ 1/3 4h1/3 sin h ö  f (x) = – lim
h 0 2
lim
= h®0 ç ç h cos h + ÷ = 0. h
3h ÷
è ø f ( x  h )  f ( x  h ) – 2 f ( x)
= lim
 f (0) = 0. h0
h2
Hence, the function f is twice differentiable at x = 0.
Find whether The above limit cannot be used as the
test for twice differentiability of f. One can verify this
  x 2  x x0 by using f(x) = x x 
f (x) =  2 Let us calculate the limit first :
sin x  x x0
is twice differentiable and find its second derivative. f (0  h)  f (0  h) – 2f (0)
lim
h 0 h2
 x  x
2
x0
f (x) =  ...(1) h | h |  ( h) | h | – 0 0
sin x  x
2
x0 = lim 2
= lim 2 = 0.
h 0 h h  0 h
We see that f is continuous. In the previous section we had calculated
 2x  1 x0  2 x x  0
f (x) =  ...(2) f (x) = 
cos x  2x x  0  2x x0
Note that the equality sign at x = 0 has been removed from where f (0 ) = – 2 and f (0 +) = 2.

because the existence of f  (0) is in doubt.
Hence f (0) does not exist. However, the given limit exists.
We can see that lim f '(x)  lim f '(x)  1 .  xe x
x 0 x 0 x0
So we have f (0) = 1. If f(x)=  2 3 then
Now, we extend the definition of f (x) at x = 0.  x  x  x x  0
examine whether f (x) is continuous and differentiable
 2x  1 x0 at x = 0.
Hence, f (x) = 
cos x  2x x  0  xe
x
x0
x0 f(x) = 
 2 2 3
Now f (x) =   x  x  x x  0
  sin x  2 x  0 f(x) is continuous at x = 0.
We find that xlim f ''(x)  lim f ''(x)  2 .  xe x  e x , x0
0 x 0
f (x) = 
f (0) = 2 and the function f is twice differentiable at 2
1  2x  3x , x  0
x = 0. We can easily see that f is twice differentiable at
lim f '(x) = 1 = lim f '(x)
other values of x. x 0 x 0
DIFFERENTIABILITY  3.37
Hence f(x) is differentiable at x = 0 and thus (4  2  x ) sin  x  8  x cos  x , x  1
2 2

f  (x) = 
 xe  e ,
x x
x0  6 x  3  2  , x 1
f (x) =  2 is continuous. f (1) =  8  and f (1 +) = 3  2 
1  2x  3x , x  0
 f (x) does not exist at x = 1.
e  (x  1)e , x  0
x x
However, f (x) is twice differentiable for x other than x = 1
Now, f (x) = 
2  6x, x 0
Find
lim f ''(x)  lim f ''(x) = 2
 x sin(1 / x) for x  0,
x 0 x 0 3

 f (x) is also differentiable at x = 0. f (x) if f (x)  


 0 for x  0
 x 3 if x  1 and find whether f (x) is continuous at the point x = 0.
Let f(x) =  2 . For x  0 the derivative f (x) can be
ax  bx  c if x  1
If f  (1) exist then find the value of a2 + b2 + c2. found by differentiating the function x3 sin (1/x)
For continuity at x = 1 we must have according to the rule of differentiation of a product.
This yields
f (1+) = f (1–)
f (x) = 3x2 sin (1/x) – x cos (1/x) (x  0).
 a+b+c=1 ...(1)
lim f (x) = 0 = lim f(x)
3x
2
if x  1 x 0 x 0
f (x) =  This means that f (0) exists
2ax  b if x  1 Thus f (x) exists at all points :
For continuity of f (x) at x = 1 3x 2 sin (1 / x) – x cos(1 / x) for x  0,
f (1+) = f  (1–) 2a + b f (x) = 
 2a + b = 3 ...(2)  0 for x  0.
Now, f (x) = 6x sin (1/x) – 4 cos (1/x)
6x if x  1 – (1/x) sin (1/x) (x  0).
f (x) = 
 2a if x  1 lim f (x) does not exist.
f (1 ) = 6, f (1+) = 2a
– x 0

Hence, 2a = 6  a = 3 To test whether f(x) exists at x = 0, we apply first


principles on f (x)
From (1) and (2), b = – 3, c = 1
f '(h) – f '(0)
Hence, the value of a2 + b2 + c2 = 19. f (x) = lim
h 0 h
 2 x 2 sin  x x 1 1 1
3h 2 sin  h cos  0
Let f (x) =  3 2 = lim h h
 x  a x  b x  1 h 0
h
be a differentiable function . Examine whether it is twice  1 1
differentiable in R . = lim 3h sin  cos 
h 0  h h
 
For continuity at x = 1 we must have which does not exist.
a+b+1=0 Thus f is twice differentiable for all x except x = 0.
4 x sin  x  2x 2 cos  x x 1 Further, f(x) is discontinuous at the point x = 0.
f (x) =  Let f, g be twice differentiable
 3x 2  2a x x 1
For differentiability of f : functions from R to R, and suppose that f(x) = g(x)/x
f (1+) = 3 + 2a, f (1–) = –2  should be equal. for all x  0. Given that g(0) = g (0) = 0 and that
2  3 2  3 g"(0) = 6, determine f(0) and f(0).
 a =  and b = By the definition of g(0), we have
2 2
4 x sin  x  2  x cos  x , x  1
2 g( x) – g(0)
Thus f (x)=  0 = g(0) = lim
2
 3x  (3  2 ) x , x  1 and x  x0
3.38  DIFFERENTIAL CALCULUS FOR JEE MAIN AND ADVANCED

g( x ) To determine this latter limit, we can use LHospitals


= lim (since g(0) = g(0) = 0)
x  x rule. Certainly lim g(x) = 0 and lim x2 = 0.
x 0 x 0
g(x) g' (x)
= xlim
0
f(x) (since f(x) = for x  0), So we look at (differentiating top and bottom).
x 2x
= f(0) (since f is continuous)
g' (x)
So f(0) = 0. To find f (0), note that But g(0) = 0, so lim x 0
= g"(0) = 6, and so
x
f ( x ) – f (0 ) g( x )
f (0) = lim = lim 2 lim g' (x) = 3. Thus by LHospitals rule,
x  x0 x  x
x 0 2x
g(x)
(since f(0) = 0 and f(x) =
x
for x  0). lim g(x) = 3 too, and this gives f (0) = 3.
x 0 x 2

E
x 2 , x  0 3. Let f(x) = x4/3
1. Let f(x) =  3 Show that f(0) exists but (a) Compute f .
x , x  0 (b) Is f a differentiable function?
f (0) does not. (c) Show that f"(0) does not exist, and compute
2. Let f be the function defined by f (x) for x  0.
 x2 4. Let the function g be defined by
 2 if x  0,
  x2 x  0,
f(x) =  x 2 g(x) =  
– if x  0.
 2 2 x – 1 x  1.
(a) Compute f . (a) Compute g and g".
(b) Is f a differentiable function? (b) Are g and g differentiable functions ?
(c) Show that f (0) does not exist, and compute sin(a  2x)  2sin(a  x)  sin a
f (x) for x  0. 5. Evaluate lim .
x 0 x2

D
6. Find y" for the following functions : tan(a  2x)  2 tan(a  x)  tan a
(a) y = |x3| 8. Evaluate lim .
x 0 x2
 x sin(1 / x), x  0, 9. (a) Show that f(x) = x7/3 is twice differentiable at
2
(b) y =  0, but not three times differentiable at 0.
 0, x0
Also find y" (0) if it exists. (b) Find an exponent k such that f(x) = xk is
(n – 1) times differentiable at 0, but not n times
7. A function f is defined as
 1 2
differentiable at 0.
 (b – a 2 ) for 0  x  a
 2
 1
3.9 ALGEBRA OF DIFFER-
x2 a2
f (x )   b 2 – – for a  x  b ENTIABLE FUNCTIONS
2 6 3
 1  b3  a 3  We can deduce from the theorems on limits that the
   for x  b
 2  x  sum, product, difference or quotient of two functions
Prove that f and f  are continuous but f  is which are differentiable at a certain point are
discontinuous. themselves differentiable at that point (except that, in
DIFFERENTIABILITY  3.39
the case of the quotient, the denominator must not Therefore, f(x). g(x) is differentiable at x = a.
vanish at the point in question). Further it is true that
composition of a differentiable function with a The product of a finite number of
differentiable function is a differentiable function. functions differentiable at a point is a differentiable
1. If f(x) and g(x) are differentiable at x = a, then the function at that point.
following functions are also differentiable at x = a. By the above theorems we recognize that the functions
(i) cf(x) is differentiable at x = a, where c is any y = x2 + 3x + 2
constant. y = sin3x – x sin x – (x4 – 1) cos x, and
(ii) f(x)  g(x) is differentiable at x = a. y = (sin x – 2x)/(x2 + 1)
(iii) f(x). g(x) is differentiable at x = a. are differentiable at every point in a domain common
(iv) f(x)/g(x) is differentiable at x = a, provided to all functions involved.
g(a)  0. 2. If f(x) is differentiable at x = a and g(x) is non-
Here, we prove some of these results : differentiable at x = a, then we have the following
Theorem If f and g are differentiable at x = a, then results:
so is their product f + g. (i) Both the functions f(x) + g(x) and f(x) – g(x)
Proof We apply the definition of derivative to are non-differentiable at x = a.
F(x) = f(x) + g(x) as follows : For example, consider, f(x) = x and g(x) =  x .
(f (a  h)  g(a  h)) – (f (a)  g(a)) Here f(x) is differentiable at x = 0 and g(x) is
F(a) = lim non-differentiable at x = 0. Both the sum function
h 0 h
x +  x  and the difference function x –  x  are
 f (a  h) – f (a) g(a  h) – g(a) 
= lim  non-differentiable at x = 0.
h 0  h h 
 (ii) f(x). g(x) is not necessarily non-differentiable
f (a  h) – f (a) g(a  h) – g(a) at x = a. We need to find the result by first
= lim + lim
h 0 h h 0 h principles.
= f (a) + g (a). For example, consider, f(x) = x 3 and
Therefore, f(x) + g(x) is differentiable at x = a. g(x) = sgn(x).
Here f(x) is differentiable at x = 0 and g(x) is
The sum of a finite number of functions non-differentiable at x = 0. But the product
differentiable at a point is a differentiable function at function
the point. x 3 , x0

Theorem If f and g are differentiable at x = a, then f(x)g(x) = 0, x0
so is their product f. g.  x 3 , x  0
Proof Let F(x) = f(x). g(x) is differentiable at x = 0.
f (a  h)g(a  h) – f (a)g(a) As another example, the product of the
F(a) = lim
h 0 h functions
f (a  h)g(a  h) –f(a  h)g(a)  f(a  h)g(a) –f(a)g(a)  1
= lim  x sin x0
h0 h
f(x) = x and g(x)   x
 g(a  h)  g(a) f (a  h)  f (a) 
= lim f (a  h)  g(a) 
 0 x0
h 0  h h 
g(a  h)  g(a) is differentiable at x = 0 even when f(x) is
= lim f (a  h) . lim differentiable at x = 0 and g(x) is non-
h 0 h 0 h
differentiable at x = 0.
f (a  h)  f (a)
+ g(a)  lim However, the product of the functions f(x) = x
h 0 h and g(x) = [x] is non-differentiable at x = 1.
As h approaches zero, f(a+h) approaches f(a) (iii) f(x)/g(x) is not necessarily non-differentiable
because f, being differentiable at a, is continuous at a. at x = a. Here also we need to work on the
= f(a) g(a) + f (a) g(a). function f(x)/g(x) to get the result.
3.40  DIFFERENTIAL CALCULUS FOR JEE MAIN AND ADVANCED

x  1, x  0 (ii) f(x). g(x) is not necessarily non-differentiable


Let f(x) = x2(x2 –1) and g(x) =  . at x = a. We need to find the result by applying
x  1, x  0 first principles on the product f(x). g(x).
Here f(x) is differentiable at x = 0 and g(x) is For example, consider, f(x) = [x] and g(x) = [–x].
non-differentiable at x = 0. Here both f(x) and g(x) are non-differentiable
 x 2 (x  1), x  0 at x = 0 but, the product function [x].[–x] is
f(x)/g(x) =  2 differentiable at x = 0.
 x (x  1), x  0
Further, f(x) = [x] and g(x) = {x} are both
We can check that f(x)/g(x) is differentiable at non-differentiable at x = 0 and, the product
x = 0. function [x].{x} is non-differentiable at x = 0.
Discuss the differentiability of Hence, we cannot comment in advance when
f(x) = [x] + | 2x – 1 |. both the functions are non-differentiable at x = a.
(iii) f(x)/g(x) is not necessarily non-differentiable
Let us study the functions y = [x] and
at x = a. Here also we need to work on the
y = | 2x – 1 |. It is a clear that f(x) = [x] is non-differentiable at all function f(x)/g(x) to get the result.
integral points and g(x) = |2x – 1| is differentiable for all x  R
– {1/2}. The sum of a non-differentiable and a differentiable  x 2  1, x  0
Let f(x) =  and
function is non-differentiable. Hence f(x) + g(x) is non-  x  1, x  0
differentiable at all integral points and x = 1/2.
3. If f(x) and g(x) both are non-differentiable at x = a, x  1, x  0
g(x) =  .
then we have the following results. x  1, x  0
(i) The functions f(x) + g(x) and f(x) – g(x) are Here both f(x) and g(x) are non-differentiable
not necessarily non-differentiable at x = a. at x = 0. But we find that f(x)/g(x) is
However, atmost one of f(x) + g(x) or differentiable at x = 0.
f(x) – g(x) can be differentiable at x = a. That
is, both of them cannot be differentiable Differentiability of Composite Functions
simultaneously at x = a. Theorem If f(x) is differentiable at x = a and g(x) is
Let us assume that both f(x) + g(x) and differentiable at x = f(a) then the composite function
f(x) – g(x) are differentiable. Then the sum of (gof)(x) is differentiable at x = a.
functions (f(x) + g(x)) + (f(x) – g(x)) = 2f(x)
must be differentiable at x = a, which is wrong A function of a function composed of
as it is given that f(x) is non-differentiable at a finite number of differentiable functions is a
x = a. Hence our assumption is wrong. So, differentiable function.
both the functions cannot be differentiable 
simultaneously at x = a. For example, f(x) = sin x is differentiable at x = and
2
For example, consider, f(x) = [x] and g(x) = {x}.
 x  1, x  1
2
Here both f(x) and g(x) are non-differentiable 
g(x) =  is differentiable at x = f( ) = 1.
at x = 0.  2x  2, x  1 2
The sum function [x] + {x} being equal to x is Hence the composite function (gof)(x) is differentiable
differentiable at x = 0. The difference function 
at x = .
[x] – {x} however is non-differentiable at x = 0. 2
But this does not mean that one of the functions
f(x) + g(x) or f(x) – g(x) must be differentiable.
1. Let a function f(x) be differentiable at all points in
We can have both the functions non-
the interval [a, b], and let its range be the interval
differentiable. For example, if f(x) = 2[x] and
[A, B] and further a function g(x) is differentiable
g(x) = {x}, then both the functions f(x) + g(x)
in the interval [A, B], then the composite function
or f(x) – g(x) are non-differentiable
(gof)(x) is differentiable in the interval [a, b].
simultaneously at x = 0 .
DIFFERENTIABILITY  3.41
2. If the function f is differentiable everywhere and n
the function g is differentiable everywhere, then The nth-root function f(x) = x is
non-differentiable at x = 0.
the composition gof is differentiable everywhere.
We may combine this result with the previous theorem.
3. If f(x) is differentiable, then | (f(x))p |, p > 1 is also
Then we see that a root of a differentiable function is
differentiable.
differentiable except possibly at the points where the
sin x given function is zero. That is, the composition
For example, f(x) = and g(x) = xx are
x2  1
differentiable for all x. Hence, the composite
h(x) = n
g( x) = [g(x)]1/n
n
sin x sin x of f(x) = x and the differentiable function g(x) is
function (gof)(x) = 2 is also
x  1 x2  1 differentiable at x = a if g(a)  0.
differentiable for all x.
Show that the function
Check the differentiability of 2/3
 x 1 
x3 f(x) =  2 
f(x) =  x  2x  2 
sin(cos x) is differentiable everywhere except at one point.
The numerator is differentiable for all x. Note first that the denominator
As far as the denominator is concerned, according to x + 2x + 2 = (x + 1)2 + 1 is never zero.
2

the theorem on differentiability of a composite function, Hence the rational function


it is differentiable at points where the function u = cos x x 1
is differentiable, since the function sin u is differentiable r(x) = 2
x  2x  2
everywhere. is defined and differentiable everywhere. It then follows
We must exclude the points at which sin(cos x) = 0. i.e. from the theorem and the differentiability of the cube
the points at which cos x = k (k  I), or cos x = 0. root function that f(x) = [r(x)] 2/3 = 3 [ r ( x)]2 is
Thus, the function f(x) is differentiable everywhere differentiable everywhere except when r(x) = 0.
except at the points x = (2n + 1)/2 (n  I). Here r(x) = 0 at x = – 1.
2/3
What can you say about the  h 
 2  0
differentiability of the function f(x) = 9  x 2 ? h 1
f '(1 )  lim  = 
Because the natural domain of this h 0 h
2/3
function is the closed interval [–3, 3], we will need to  h 
investigate the differentiability of f on the open interval  2  0
h 1
(–3, 3) and at the two endpoints. If c is any number in f '(1 )  lim  = 
h 0 h
the interval (–3, 3), then f is differentiable at c. The Hence, f is differentiable everywhere except at x = – 1.
function f is non-differentiable at the endpoints since
Let f(x) = [sin x] + [cos x], x [0, 2],
9 – (3  h)2  0 where [.] denotes the greatest integer function.
f '(3– )  lim
h 0 h Find the number of points where f(x) is non-
differentiable.
6h  h 2
 lim   
h 0 h [sin x] is non-differentiable at x = , ,
2
9 – ( 3  h) 2  0  3
f '( 3 )  lim 2 and [cos x] is non-differentiable at x = 0, , , 2.
h 0 h 2 2
6h  h 2 Thus, f(x) is definitely non-differentiable at
 lim  3
h 0 h x = 0, , .
Thus, f is differentiable on the open interval (–3, 3). 2
3.42  DIFFERENTIAL CALCULUS FOR JEE MAIN AND ADVANCED
 Continuity of g(x) :
We need to check at x = and 2, since both the
2 g(x) is continuous at all those points where both f(|x|)
functions are non-differentiable at these points. and |f(x)| are continuous.
 
f    1 , f   0   0 At x = 0, f(|x|) is continuous but |f(x)| is discontinuous.
2 2   g(x) is discontinuous at x = 0 and hence non-
and, f(2) = 1, f(2 – 0) = – 1. differentiable.
 At x = 1, f(|x|) is differentiable but |f(x)| is non-
Thus, f(x) is discontinuous at x = and 2and hence
2 differentiable because of corner. Hence, g(x) is non-
non-differentiable at these points. differentiable at x = 1. Similarly g(x) is non differentiable
 3 at x = 2.
Finally, f is non-differentiable at x = 0, , , , 2
2 2 Finally, g(x) is discontinuous at x = 0 and it is non
i.e. at 5 points. differentiable at x = 0, 1, 2.
x  3 x0 Theorem Let f(x) and g(x) be defined on an open
If f(x) =  2 interval containing the point x = a where f is
 x  3x  2 x  0
differentiable at a, f(a) = 0 and g is continuous at a,
and g(x) = f(|x|) + |f(x)|, then comment on the continuity
then the product f.g is differentiable at a.
and differentiability of g(x) by drawing the graph of
f(|x|) and, |f(x)|. f (a  h)g(a  h) – f (a)g(a)
(fg)(a) = lim
Graph of y = f(x) h 0 h
f (a  h)g(a  h) – 0.g(a)
= lim [since f(a) = 0]
h 0 h
f (a  h)  f (a)
= lim .g(a  h)
h 0 h
f (a  h)  f (a)
= lim .lim g(a  h) = f (a). g(a)
h 0 h h 0
As h approaches zero, g(a+h) approaches g(a) because
g is continuous at a.
Therefore, f(x). g(x) is differentiable at x = a.
For example, f(x) = sin x.(x – + 1)is differentiable at
x = since sin x is differentiable at x = , sin = 0 and
(x – + 1) is continuous at x = 
Corollary:
If f(x) is differentiable at x = a, then the product f(x).f(x)
is also differentiable at x = a.
For example, f(x) = sin xsin xis differentiable at x = 0,
since sin x is differentiable at x = 0
Let f(x) = e(x – 1) – ax2 + b and

e , x  1
x 1
g(x) =  2
with f (1) = 2. Find the values
 x , x  1
of a and b so that the function h(x) = f (x) · g(x), is
differentiable at x = 1.
Note that g is continuous at x = 1, but
g (1) does not exist.
DIFFERENTIABILITY  3.43
Also f (x) = ex – 1 – 2ax given above.
1 f (1) = 0 1 – a + b = 0
f (1) = 1 – 2a = 2  a  
2 3
In such a situation, for h (x) = f (x)· g (x) to be  b .
differentiable at x = 1, f(1) should be 0, using the theorem 2

F
1. If it reasonable to assert that the sum F(x) = f(x) + that the product F(x) = f(x) g(x) has no
g(x) has no derivative at the point x = x0 if : derivative at the point x = x0 if :
(a) The function f(x) has a derivative at the point (a) The function f(x) has a derivative at the
x0, and the function g(x) has no derivative at point x0, and the function g(x) has no
this point? derivative at this point ?
(b) Neither function has a derivative at the point x0? (b) Neither function has a derivative at the
2. Suppose that f(x) = x2 and g(x) = |x|. Then the point x0?
composite functions (f o g) (x) = |x|2 = x2 and 4. Find the one-sided derivatives of the function
(g o f) (x) = |x2| = x2 are both differentiable at x = 0 f(x) = |x – x0| g(x) at the point x0, where g(x) is a
even though g itself is not differentiable at x = 0. function continuous at the point x0. Does the
Does this contradict the chain rule? Explain. function f(x) possess a derivative at the point x0?
3. Consider the functions : 5. If f(x) is differentiable at x = a, then prove that
(i) f(x) = x, g(x) = |x|;
(ii) f(x) = |x|, g(x) = |x|. Is it reasonable to assert F(x) = (f(x) – f(a)).x – ais also differentiable at x = a.

E
6. If f(x) = tan x and g(x) = x – 1 Is the product (fof) (x) explicitly, find whether fof is differentiable
function F(x) = f(x) . g(x) differentiable at x = 1 ? at x = 0.

 x sin 1x if x  0
3 1 1  x 0  x  1
7. If f(x) = =  and g(x) =  
 0 if x  0 x 11. Let f(x) =  x  2 1  x  2 , Discuss the
 4  x 2  x  4
Is the product function F(x) = f(x) . g(x)
differentiable for all real x ? continuity and differentiability of fof.
8. Let g(x) be derivable on R and g(x0)  0. x  1 , – 1  x  0
(a) Show that if f(x) = (x – x0)2 g(x), then 12. Let f(x) =  2 , Discuss the
x , 0  x  1
f(x0) = f (x0) = 0 and f"(x0)  0.
continuity and differentiability of
(b What can you say about f(x) = (x – x0)3 g(x)?
(c) Generalize to f(x) = (x – x0)n g(x). h(x) = f(|sin x|) + |f(sin x)| in [0, 2].
9. Suppose u = g(x) is differentiable at x = – 5, e a /x  e  a /x
y = f(u) is differentiable at u = g(–5), and (f o g) 13. Let f(x) = xp when x  0 and f(0) = 0,
e a /x  e  a /x
(–5) is negative. What, if anything, can be said
with p, a > 0. Find all possible values of p so that
about the values of g(–5) and f (g(–5))?
f(x) is differentiable at x = 0.
1  x 2 , x  0 14. Discuss the differentiability of the function

10. Let f(x) = 2x  1 , 0  x  1 without finding 2 2
f(x) = (x –a) |x –5x+6|+(sin x) |sin x| for all a R.
 x 2  2 , x  1
3.44  DIFFERENTIAL CALCULUS FOR JEE MAIN AND ADVANCED

3.10 FUNCTIONAL EQUATIONS If x = 0, f (0) = 1  a = 1


 f (x) = ekx where k = f (0).
We follow the following steps to determine the
Hence, the functions are f(x) = 0 and f(x) = ekx.
functions which are differentiable( or which can be
proved to be differentiable) and satisfying a given If f (x + y) = f (x) + f (y)  x, y  R
functional rule : and f (x) is a differentiable function, then prove that
(i) First we write down the expression for f (x) : f(kx) = k f (x) for  k, x  R.
f (x  h)  f (x)
f (x) = lim
h 0
Given f (x + y) = f (x) + f (y)
h
(ii) We manipulate f (x + h) – f (x) in such a way that f(x  h)  f(x)
the given functional rule is applicable. Now we f (x) = lim
h0 h
apply the functional rule and simplify the R.H.S.
f(x)  f(h)  f(x)
to get f  (x) as a function of x. = lim
(iii) Then we integrate f  (x) to get f (x) as a function of h0 h
x and a constant of integration. In some cases a f(0  h)  f(0)
differential equation in formed which can be solved = lim = f (0). [as f (0) = 0]
h 0 h
to get f (x).  f (x) = f (0)
(iv) Finally we apply the boundary conditions to  f (x) = f (0)x + c
determine the value of the constant of integration.
If x = 0, f (0) = 0 c = 0
If f (x + y) = f (x) · f (y),  x , y  R  f (x) = f (0)x
and f (x) is a differentiable function, then find f (x). f (x) = ax, where a = f (0)
f(x  h)  f(x) f (kx) = a kx = k ax = kf (x)
f (x) = lim Hence, f (kx) = kf (x).
h0 h
f(x)·f(h)  f(x) A function f : (0, )  R satisfies
= lim the equation f(x/y) = f(x) – f(y). If f(x) is differentiable
h 0 h
f (1  x)
= lim f(x)
 f(h)  1 ...(1) on (0,  ) and lim  3 , then determine f(x).
x 0 x
h 0 h
Put x = 0, y = 0; f (0) = f 2 (0) We have f(x/y) = f(x) – f(y) ...(1)
 f (0) = 0 or f (0) = 1 Putting x = 1 and y = 1 in equation (1), we have
If f (0) = 0 then put y = 0 in the rule f(1) = f(1) – f(1)  f(1) = 0.
f (x) = f (x) · f (0) = 0 f ( x  h )  f ( x)
We have f (x) = lim
 f (x) = 0. f (x) h 0 h
 f (x) = 0. xh
If f(0) = 1, then we proceed from (1) as follows : f  
= lim  x  [from (1)]
 f(h)  f(0) h 0 h
f (x) = f(x) = f(x) · f (0)
h h
f 1  
[Let k = f (0)] 1  x  1 f 1   
f '(x) lim .
= h 0 x h = . lim
 =k x   0 
f(x) x
 ln  f (x) = k x + c h
 f (x) = ekx + c = ekx ·ec [  0 as x  0  x  (0, ) ]
x
f(x) = ± ekx ·ec 3 f (1  x)
f(x) = a.ekx where a = ± ec = [from lim 3]
x x 0 x
DIFFERENTIABILITY  3.45
To find f(x), write the above equation as g(h)
= ex lim  1. g(x)
df 3 h0 h
 = g(x) + 2ex.
dx x
On integrating both sides w.r.t. x we get Thus, g(5) – 2e5 = g(5) = 32.
f(x) = 3 ln x + c, where c is a constant. A differentiable function f(x) satisfies
Now, using the condition f(1) = 0, we have
the condition f(x + y) = f(x) + f(y) + xy for all x, y  R
f(1) = c which gives c = 0
Hence, we have f(x) = 3 ln x. and
1
A differentiable function satisfies lim f(h)  3 then find the least value of f(x).
h0 h
the relation f (x + y) = f (x) + f (y) + 2xy – 1  x, y  R. f(x  h)  f(x)
f(x) = lim
If f (0) = 3  a  a 2 , find f (x) and prove that f (x) > 0 h 0 h
 x  R. f(x)  f(h)  hx  f(x)
= lim
f (x  h)  f (x) h 0 h
f (x) = lim f(h)
h 0 h = lim +x=3+x
h0 h
f (x)  f (h)  2xh  1  f (x)
= lim
h 0 h x2
f (h)  1 Integrating f(x) = 3x + +k ...(1)
= 2x + lim [Put x = 0, y = 0 to get f (0) = 1] 2
h 0 h Putting x = 0, y = 0 in the given relation
f (h)  f (0) f(0) = f(0) + f(0) + 0  f(0) = 0
= 2x + lim
h 0 h Now from (1) we have f(0) = 0 + k  k = 0
 f (x) = 2x + f (0)
x2
Integrating, f (x) = x2 + f (0)x + c  f(x) = 3x + .
If x = 0 ; f (0) = 1  c = 1 2
It is a quadratic function whose least value occurs at
 f (x) = x2 +  
3  a  a2 x  1
x = – 3. The least value is 
9
.
Now D = 3 + a – a2 – 4 = – (a2 – a + 1) < 0 2
 f (x) > 0  x  R.
 x  y   f(x)  f(y)
Given a function g which has Let f   for all
 2  2
derivative g(x) for all x satisfying g(0) = 2 and g(x + y) real x and y. If f (0) = – 1 and f(0) = 1, then find f(2).
= ey g(x) + ex g(y) for all x, y  R, g(5) = 32. Find the
 x  y   f(x)  f(y)
value of g(5) – 2e5 . Given f  
 2  2
Putting x = y = 0 in f (2 x )  f (0 )
Replacing x by 2x and y by 0, then f(x) =
g(x + y) = ey g(x) + ex g(y) we get g(0) = 2g(0) 2
 g(0) = 0  f(2x) + f(0) = 2f(x)  f(2x) – 2f(x) = – f(0) ...(1)
g(h)  g(0) g(h) f (x  h ) – f (x)
So 2 = g(0) = lim  lim f (x) = lim
h 0 h h0 h h 0 h
g(x  h)  g(x) f 2 x  2 h 
Also g(x) = lim  – f (x)
h  2 
h0 = lim
h 0 h
e x g(h)  e h g(x)  g(x)
= lim  f (2 x )  f (2 h ) 
h 0 h  – f (x) 
= lim  2
 x g(h) e h  1  
= lim  e  g(x)  h 0
 h 
h0
 h h   
3.46  DIFFERENTIAL CALCULUS FOR JEE MAIN AND ADVANCED

 f (2 x )  f (2 h ) – 2 f ( x )   x  y   f(x)  f(y)
= hlim   But given f   which is possible
0  2h   2  2
when P  M, i.e., P lies on AB. Hence, y = f(x) must be
 f (2 h ) – f (0) 
= hlim 
0 
 (from (1)) a linear function.
2h 
Let f(x) = ax + b  f(0) = 0 + b = 1 (given)
= f (0)
and f (x) = a  f (0) = a = – 1 (given)
=–1xR (given)
Integrating, we get f(x) = – x + c  f(x) = – x + 1
Putting x = 0, then f(0) = 0 + c = 1 (given)  f(2) = – 2 + 1 = – 1.
 c = 1 then f(x) = 1 – x  x  y   2  f(x)  f(y)
If f   for
 f(2) = 1 – 2 = – 1.  3  3
Alternative 1 : all real
x and y and f (2) = 2 then determine y = f(x).
 x  y  f (x)  f (y)
f  x  y  2  f(x)  f(y)

 2  2 f   ...(1)
Differentiating both sides w.r.t x treating y as constant.  3  3
Replacing x by 3x and y by 0 then
 x  y  1 f ' (x)  0 xy 2  f (3 x )  f ( 0 )
 f ·   f'  = f (x).
 2  2 2  2  f(x) =
3
Replacing x by 0 and y by 2x,  f(3x) – 3f(x) + 2 = – f(0) ...(2)
then f (x) = f (0) = – 1 (given) Putting x = 0 and y = 0 in (1),
Integrating, we have f(x) = – x + c. we get f(0) = 2 ...(3)
Putting x = 0, f(x) = 0 + c = 1 (given)
f (x  h) – f (x)
 c=1 Now, f (x) = lim
h 0 h
Hence, f(x) = – x + 1 then f(2) = – 2 + 1 = – 1.
Alternative 2 : f  3x  3h  – f (x)
Suppose A(x, f(x)) and B(y,f(y)) be any two points on = lim  3 
the curve y = f(x).
h 0 h
2  f (3x)  f (3h)
If M is the mid point of AB, then coordinates of M are – f (x)
= lim
3
 x  y . f(x)  f(y)  h 0 h
 
 2 2  f (3x )  3f ( x )  f (3h )  2
According to the graph, coordinates of P are = lim
h 0 3h
 x  y  x  y  f (3 h )  f ( 0 )
 . f   = lim {from 2)}
 2  2  h 0 3h
= f (0) = c (say)
 x  y  f(x)  f(y)  f (x) = c
and PL > ML   >
 2  2 At x = 2, f (2) = c = 2 (given)
 f (x) = 2
Integrating both sides, we get f(x) = 2x + a
Putting x = 0 then
f(0) = 0 + a = 2 {from (2)}
 a=2
then f(x) = 2x + 2.
Alternative :
 x  y   2  f(x)  f(y)
We have f  
 3  3
DIFFERENTIABILITY  3.47
Differentiating both sides w.r.t. x treating y as constant, On integrating w.r.t.x and taking limit 1 to x,
 x  y  1   2  f '(x)  0 f(x) f(1)

we get f '    we have = f (1) (ln x – ln 1)
 3  3  3 x 1
Now replacing x by 0 and y by 3x, then f(x)
f (x) = f(0) = c (say)  – 0 = f (1) ln x (f(1) = 0)
x
At x = 2,  f(x) = f (1) (x ln x).
f (2) = c = 2 (given) Alternative :
 f (x) = 2 Given f(xy) = xf(y) + yf(x)
On integrating we get Differentiating both sides w.r.t. x treating y as constant,
f(x) = 2x + a f (xy) . y = f(y) + y f (x)
Putting x = 0, then f(0) = 0 + a = 2 {from (1)} Putting y = x and x = 1, then
 f(x) = 2x + 2. f (xy). x = f(x) + xf(x)
Let f(xy) = xf(y) + yf(x) for all xf (x)  f(x) f (1)
 
x, y  R and f(x) be differentiable in (0, ) then
+
x2 x
determine f(x). d  f(x)  f (1)
Given f(xy)= xf(y) + yf(x)   
dx  x  x
Replacing x by 1 and y by x then we get x f(1) = 0 Integrating both sides w.r.t. x taking limit 1 to x,
 f(1) = 0, x  0 ( x, y,  R+) f(x) f(1)
f(x  h)  f(x) – = f (1){ln x – ln 1}
Now, f (x) = lim x 1
h 0 h f(x)
h  – 0 = f(1) ln x ( f(1) = 0)
f  x  1     f(x) x
  x  Hence, f(x) = f (1)(x ln x).
= lim
h 0 h  xy 
 h  h If f(x) + f(y) = f   for all
xf  1     1   f(x)  f(x)  1  xy 
= lim  x  x
f ( x)  1 
h 0 h x, y  R (xy  1) and lim =2. Find f   and f (1).
x 0 x  3
h h
xf  1    f(x)  xy 
= lim  x  x f(x) + f(y) = f   ...(1)
h 0 h  1  xy 
h Putting x = y = 0, we get f(0) = 0.
f  1   Putting y = – x, we get f(x) + f(–x) = f(0)
 x   lim f(x)
= lim  f(–x) = – f(x) ...(2)
h0 h h 0 x
  f ( x)
x Also, lim =2
f(x)
x 0 x
= f (1) + f ( x  h ) – f ( x)
x Now, (x) = lim
h 0
...(3)
f(x) h
 f (x) – = f(1) f ( x  h ) – f ( x)
x = lim
h 0
(using (2))
h
xf (x)  f(x) f (1)
   xhx 
x2 x  
 1  ( x  h)(– x) 

x  
f(x) f(x)
x

f (1)
x
 f (x) = lim
h 0 h
(using (1))
3.48  DIFFERENTIAL CALCULUS FOR JEE MAIN AND ADVANCED

f h  f (1)
  1  x(x  h   = f(x) . 1 + ex – 1.
x
 f (x) = lim h 0

h

e x1.e
  = f(x) + ( f (1) = e)
  x
h ex 1
f  2 

f (x) = f(x) +  e–xf (x) – e–x f(x) =
 1  xh  x    1  x x
 f (x) = lim h0  
 h   1  xh  x 2  d –x 1
   (e f(x)) =
 1  xh  x 2  dx x
h On integrating we have
f  
 e–xf(x) = ln x + c at x = 1, c = 0
 1  xh  x 2   lim 1
 f (x) = lim h h 0 1  xh  x 2
 f(x) = ex ln x.
h 0  
   ,
 1  xh  x 2 
A function f : (–1, 1)   
 using lim f ( x)  2   2 2
  satisfies the equation
 x 0 x 
1 2 f(x) + f(y) = f(x 1  y 2 + y 1  x 2 ).
 f (x) = 2 ×  f (x) = (i) Show that f(x) is odd.
1  x2 1  x2
Integrating both sides, we get (ii) If f(x) is differentiable on (–1, 1) and f (0) = 1, then
f(x) = 2 tan–1(x) + ck, where f(0) = 0  c = 0 1
Thus, f(x) = 2 tan–1 x. show that f (x) =
1  x2
 1    (iii) Hence, determine f(x).
Hence, f   = 2  , and
 3 6 3
2 2
f (1) =   1. (i) f(x) + f(y) = f(x 1  y 2 + y 1  x 2 ). ...(1)
1  12 2
If e–xyf(xy) = e–xf(x) + e–yf(y)  x, Putting y = –x in equation (1), we have
y  R+, and f(1) = e, determine f(x). f(x) + f(–x) = f(x 1  x 2 – x 1  x 2 )
Given e–xy f(xy) = e–xf(x) e–yf(y) ...(1)  f(x) + f(–x) = f(0)
Putting x = y = 1 in (1), we get, f(1) = 0 ...(2) Putting x = 0 and y = 0 in equation (1), we have
f(x  h)  f(x) f(0) + f(0) = f(0)  f(0) = 0.
Now, f (x) = lim Hence, f(x) + f(–x) = 0 ...(2)
h 0 h
  h  Thus f(x) is odd.
f  x  1    f(x.1)
(ii) Now, f (0) = 1
= lim   x 
h0 h f ()  f (0)
  x
h
 h   x  x
 lim =1 [using f(0) = 0]
e xh
1
1
e f (x)  e x f  1     2 (e f (x)  e f (1))
 0 
  x  
= lim f ( )
h 0 h  lim =1
h 0 
x  h 1  h x 1
e h f (x)  e xf
1  x   f (x)  e f (1) f ( x  h )  f ( x)
= lim   Now f (x) = lim
h 0 h
h 0 h
h
h f ( x  h )  f (  x)
x f 1 
h
h e   = lim [using(2)]
 e 1  x h 0 h
= f(x) lim   +e
(x – 1)
lim
h 0
 h  h 0 h f {x  h) 1  x 2  x 1  ( x  h)2 }
x. = lim
x h 0 h
( f(1) = 0) [using (1)]
DIFFERENTIABILITY  3.49

f() (x  h) 1  x 2  x 1  (x  h)2 A differentiable function f satisfies


= lim .lim
 0  h 0 h f(x + y) + f(x – y) – (y + 2) f(x) + y(x2 – 2y) = 0  x ,
y  R. Find f (x).
assuming  = (x  h) 1  x 2  x 1  (x  h) 2
Put y = h
2 2
(x  h) 1  x  x 1  (x  h) f(x + h) – f(x) + f(x – h) – f(x) – hf(x) + h(x2 – 2h) = 0
= lim Dividing by h on both sides
h 0 h
f (x  h)  f (x) f (x  h)  f (x)
x( 1  x 2  x 1  ( x  h ) 2 ) h 1  x 2  – f(x) + x2–2h= 0
= lim  h h
h 0 h h and applying limit h 0 on both sides, we get
2 2
x(1  x  1  ( x  h ) ) f (x) – f (x) – f(x) + x2 = 0
2
= 1  x  lim
h 0
 f(x) = x2.
h( 1  x 2  1  ( x  h ) 2 )
A twice differentiable function f
x2 1
= 1  x2   satisfies the relation f(x2 + y2) = f(x2 – y2) + f(2xy)  x,
1  x2 1  x2 y  R. If f(0) = 0 and f (0) = 2, find f(x).
1 f(x2 + y2) = f(x2 – y2) + f(2xy)
Hence, f (x) = ...(3)
1 x 2 Put y = h
(iii) Integrating both sides of (3) f(x2 + h2) = f(x2 – h2) + f(2xh)
gives f(x) = sin–1x + c (c is a constant). Dividing by h2 on both sides
Using the condition f(0) = 0 gives c = 0, and hence f (x 2  h 2 )  f (x 2 ) f (x 2  h 2 )  f (x 2 )
we have f(x) = sin–1x. 
h2 h2
A differentiable function f satisfies f (2xh)
+ ...(1)
the relation f(x + y) – 2f(x – y) + f(x) –2f(y) = y – 2 h2
 x , y  R. Find f (x). f(2xh) f '(2xh).2x
Now lim  lim
f(x + y) – 2f(x – y) + f(x) –2f(y) = y – 2 h 0 h 2 h 0 2h
...(1) (using LHospitals rule)
Put x = y = 0 in (1) xf "(2xh).2x
 f(0) = 1 = lim
h 0 1
Put y = h in (1) = 2x2f"(0).
f(x + h) –2f(x – h) + f(x) – 2f(x) = h – 2 Applying limit h  0 on both sides of (1), we get
f(x + h) – f(x) – 2[(f(x – h) – f(x)] – 2(f(h) – 1) = h f (x2) = – f (x2) + 2x2f"(0)
Dividing by h on both sides and applying limit h 0 2f (x2) = 2x2f"(0)
Replacing x2 by t
f (x  h)  f (x) 2  f (x  h)  f (x) 
lim  2f (t) = 2t f"(0) f (t) = 2t
h 0 h h Integrating both sides w.r.t. t
 2f (h)  1 

 h  =1
  f '(t)dt   2t dt
 f (x) + 2f (x) – 2f (0) = 1 [f is differentiable] f(t) = t2 + c
Since f(0) = 0, c = 0.
 3f (x) = 1 + 2f (0) ...(2)
 f(t) = t2 or, f(x) = x2.
Putting x = 0 in (2) we get f (0) = 1.
 f (x) = 1 f(x) is a differentiable function satisfy
 f(x) = x + c the relationship f 2(x) + f 2(y) + 2 (xy –1) = f 2(x + y)
Since f(0) = 1, c = 1, we have f(x) = x + 1.
3.50  DIFFERENTIAL CALCULUS FOR JEE MAIN AND ADVANCED

 x, y  R. Also f (x) > 0  x R , and f ( 2) = 2. x


We have f(xy) = 2f(x) – f   ...(1)
Determine f (x). y
(i) Putting x = 1 in (1), we have
Put x = 0 and y = 0  f 2(0) = 2
1
f(x  h)  f(x) 2f(1) = f(y) –  
f (x) = lim y
h 0 h
1
f 2 (x  h)  f 2 (x)  f(y) = –f   [since f(1) = 0] ...(2)
= lim y
h 0 [f(x  h)  f(x)].h (ii) In (1) we exchange x and y to get
2
f (h)  2(xh  1) y
= lim f(xy) = 2f(y) – f  
h 0 2f (x) h x
 2xh f 2 (h)  2  x
1   f(xy) = 2f(y) + f   [using (2)] ...(3)
= lim   y
2 f ( x ) h 0  h h 
Now we subtract (1) from (3)
1  f 2 (h)  f 2 (0)  x
=  2x  lim 
2 f (x)  h 0 h  0 = 2f(y) – 2f(x) + 2 f  
y
1  2x  lim f(h)  f(0) . f(h)  f(0)  x
=   
2 f (x) h 0 h   f(x) – f(y) = f   ...(4)
y
1 We have
f (x) = [2 x  2 f (0) . f ' (0)]
2 f ( x ) xh
f
f (x  h)  f (x) 
 f (x) . f  (x) = x + f (0). f (0) f (x)= lim  lim 
x 
[using (4)]
 f (x) . f  (x) = x + , where  = f (0) · f (0)
h 0 h h 0 h
Integrating both sides,  h
f 1  
x  f ' (1)
f 2 (x) x 2 lim 
= h 0
= + x + c h = x
2 2 x.
x
f 2(x) = x2 + 2x + c
1
at x = 0 , f 2 (0) = 2  c=2 i.e. f (x) =
x
at x = 2 , f 2( 2 )=4 =0 This gives f(x) = ln x + c
 f 2(x) = x2 + 2 Now, using the condition f(1) = 0 gives c = 0.
Hence, we have f(x) = ln x.
 f (x) = x2  2 ( as f (x) > 0 )
A differentiable function f satisfies
function f : (0, )  R satisfies the the relation f(x + y) + f(xy – 1) = f(x) + f(y) + f(xy)  x ,
y  R.
x
equation f(xy) = 2f(x) – f   .If f is differentiable on R+ If f(1) = 2, f (0) = 1 and f (– 1) = – 1, find f(x).
y
Putting x = 0, y = 0 in the given rule,
and f(1) = 0, f(1) = 1, then show that
f(0) + f(–1) = 3f(0)
1 x  f(–1) = 2f(0) ...(1)
(i) f(y) = – f   (ii) f(x) + f(y) = f  
y y Now, putting y = h
and hence determine f(x). f(x + h) + f(xh – 1) = f(x) + f(h) + f(xh)
DIFFERENTIABILITY  3.51
 f(x + h) – f(x) + f(– 1+ xh ) – f(– 1) f ( x )  f ( y)
= f(h) + f(xh) – f(– 1) Since lim = f (x), we see from (2) that
y x xy
 f(x + h) – f(x) + f(– 1+ xh ) – f(– 1)
|f (x)| < 0  f (x) = 0
= f(h) – f(0) + f(xh) – f(0) [using(1)]
Dividing both sides by h,  f (x) = c.
Hence f(x) is a constant function.
f(x  h)  f(x) f( 1  xh)  f( 1)
 .x Suppose
h xh
f(h)  f(0) f(xh)  f(0) p(x) = a0 + a1x + a2x2 + ... + anxn.
=  .x If | p(x) |  | ex–1 – 1 | for all x  0, prove that
h xh
Now, applying limit h  0 on both sides , we get | a1 + 2a2 + ... + nan |  1.
f (x) + xf (–1) = f (0) + x f (0) Given p(x) = a0 + a1x + a2x2 + ... + anxn
Given that f (0) = 1, f (–1) = –1, we have  p(x) = 0 + a1 + 2a2 x + ... + nanxn–1
f (x) = 2x + 1  p(1) = a1 + 2a2 + ... + nan
f(x) = x2 + x + c Now, | p(1) |  | e1–1 – 1| = |e0 – 1| = 0
f(1) = 2  c = 0  | p(1) |  0  p(1) = 0
 f(x) = x2 + x. As | p(x) |  | ex–1 – 1 |
Let f : R  R be such that for all x we get | p(1 + h) |  | eh – 1|  h > – 1, h  0
and y in R, |f(x) – f(y) | < |x – y|3 . Prove that f(x) is a  | p(1 + h) – p(1) |  |eh – 1| ( p(1) = 0)
constant function. ph – 1
p(1  h) – p(1)
We are given that   h
h
| f(x) – f(y) | < |x – y|3 ...(1)
Taking limit as h  0 on both sides, then
Let x be any real number and let y be chosen arbitrarily
close to x but not equal to x. Then writing (1) as p(1  h ) – p(1) h
 lim  lim e – 1
f ( x )  f ( y) h 0 h h 0 h
< |x – y|2
xy p(1  h) – p(1) h
 lim  lim e – 1
and letting y  x, we get h 0 h h 0 h
f ( x )  f ( y) 2  | p(1)|  1
lim  lim x  y ...(2)
yx xy y x  | a1+ 2a2 + ... + nan|  1 {from (1)}

F
1. A function f : R R+ satisfies f(x + y) = f(x) .  x  y  f(x)  f(y)
5. Let f  = x, y  R, n > 2 and
f(y)  x R. If f (0) = 2, then show that f (x) = 2f(x).  n  n
f (0) = 2 then find f(x).
2. Let f(xy) = f(x) f(y)  x, y  R and f(1)  0,
f (1) = 1, prove that f is differentiable for all x 0.  xy  f(x)f(y)
6. Let f   = for all real x and y. If
Hence determine f(x).  2  6
f(1) = f (1) = 3, then prove that one of the functions
 x  y  f (x)  f (y)
3. Let f   for real x and y. If f satisfies f(x) + f(1 – x) = 3 for all non-zero real x.
 2  2
f (0) exists and equals –1 and f(0) = 1 then find the x +
7. If 2f(x) = f(xy) + f    x, y R , f(1) = 0 and
value of f(2) y
4. If f(x) satisfies f(1 – x) = f(x) x R and f (1) = 0 f (1) = 1 then find f (2) and f(2).
then find f (0) if it exists. 8. Let f : R R such that f(x + y)
3.52  DIFFERENTIAL CALCULUS FOR JEE MAIN AND ADVANCED
x+y x y
= f(x) + f(y) + e (x + y) – xe – ye + 2xy  x, y n 1
n 2 (n  1)2
R then find f(x) given that f (0) = 1.  (f(r))
r 2
3
=–
4
.

 2x  3y  2f(x)  3f(y) 10. A differentiable function f satisfies the relation


9. If f   =  x, y  R, f(x + y)[f(x) – f(y)] = f(x –y)[f(x) + f(y)], where
 5  5
f(0) = 1, f (0) = – 1, prove that f(1) = 2. Find f(x).

Check the differentiability of f(x) at h()  h(0) h ( 0 )  h (  )


lim  lim
x = 0, where  0   0 
h()  h()
 2  e|x|{x}   lim  lim
f(x) = x  , x0  0   0 
 | x | {x}  h ()
1  2 lim =0 ...(1)
= , x=0  0 
2 Now consider f (x) at x = 0.
where { } represents the fractional part function. f ( )  f ( 0 ) h() cos(1 / )
R.H.D. = lim  lim
 0    0 
 2  e|h|{h}  1 h()
= lim . lim cos(1 / )
h   0   0
f (0+) = lim f(0  h)  f(0)  lim  | h | {h}  2 =0 [ using (1)] ...(2)
h 0 h h 0 h
 2  e 2h  1 f ( 0 )  f (  )  h(  ) cos( 1 / )
L.H.D.= lim  lim
h   0   0 
= lim  2h  2
 h()
h 0 h = lim . lim cos(1 / )
[ |h| + {h} = h + h = 2h for small h > 0]  0   0
=0 [ using (1)] ...(3)
1  e2h Since R.H.D. = L.H.D. = 0, we have f (0) = 0.
= lim = –ln e = –1.
h 0 2 h ì -æç 1 + 1 ö÷
f(0)  f(0  h) ï ç ÷
If f(x) = íxe è x x ø , x¹ 0 ,
f (0–) = lim ïa,
h 0 h î x= 0
1  2  e| h|{ h}  find the value of ‘a’ such that f(x) is differentiable at x= 0.
 (  h)  
= lim
2  |  h | { h} 
h 0 h 1 1 
  
h h 
1 f(h)  f(0) h e a
 h(2  e) f (0) = lim = lim
= lim 2 h0 h h 0 h
1 1
h0 h   
[ |–h| + {–h} = h + 1– h = 1 for small h > 0] + h e h h
a
f (0 ) = lim
1 h 0 h
= lim 
h0 2 h h.e 2/h
a a
Hence, f is non-differentiable at x = 0. = lim = 0  lim = 0 , provided a = 0.
h0 h h0 h
1 1
   
Find f (0) if f(x) = h(x)cos(1/x), x  0, h . e  h h
a

f(0) = 0, where h(x) is an even function differentiable at Also f (0 ) = lim
h0 h
x = 0 and h(0) = 0. a
The existence of h(0) implies that = 1 + lim = 1, provided a =0.
h0 h
Hence, a = 0.
DIFFERENTIABILITY  3.53

 x  3  x , x  1 continuous there.

 2  
Let f(x) =  sin  x , x  1
(iv) f (3/2 ) = lim

f 23  h  f 23   
where [.] denotes the greatest integer function. h0 h
Find whether f(x) is
(i) is continuous at x = 0, = lim
3  2 h  3 23  h  0  
(ii) is differentiable at x = 0, h0 h
(iii) is continuous but not differentiable at x = 1, 2h . 1
(iv) is continuous but not differentiable at x = 3/2. = lim =2
h0 h

   h f (3/2+) = lim

f 23  h  f 23  
(i) lim  f(x) = lim f(0  h) = lim sin   =0 h0 h
x0 h0 h0  2 
lim f(x) = lim f(0 + h) = lim sin
h
=0 = lim
2h  3
2h 0 = 
lim
2h . 1
=2
x0 h0 h0 2 h0 h h h0

 f(x) is f(0) =0
continuous at x = 0. 3
f 0  h  f 0  f(x) is not differentiable at x = but it is
(ii) f (0–) = Lim
2
h0
h continuous there.
h h
 sin 2  0  sin 2 
= lim = lim =
h0 h h  0 2  h 2  1
2 
 (1  {x}) 1  {x}

f 0  h   f 0   {x} 
f (0 ) = lim
+
  , x  Integer
h0 h Let f(x) =  e  .
h  
 sin 2  2
= lim 
=
h0 2 h 2  , x  Integer
2 e
f (0–) = f  (0+) Discuss the continuity and differentiability of f(x) at
 f(x) is differentiable at x = 0. any integral point, where {.} denotes the fractional part.
f 1  h  f 1 Let x = I0 be any arbitrary integers.
(iii) f (1–) = hlim
0
h 2
f(I0) = .
  1  h  e
sin   h   1 h
  cos 2  1 f(I0 – ) = lim f(I0 – h)
= lim = lim h0
h0 h h0 h 1
  1
2 h  (1  {I 0  h}){I0 h}   (1  1)1  2
2 sin 4  2 h  2 h 
= lim . = lim   =0
= lim 
h 0 e   e  e

h0   h 
2
16 h0  8     
 4   
1/h
f (1  h)  f (1) 1  h  1  (1  h)1/h 
f (1+) = lim = lim f(I0 + h) = lim (I0 + h) = lim  
h0 h h0 h h0 h 0
 e 
1  h  1 1 h 1 1/h
= lim = lim =1 lim 
 (1 h)1/h 

1  (1 h)1/h 
lim  1
h h 0  h 0 h  
h0 h0 h =e  e 
e  e 
f (1–)  f  (1+)
1  (1 h)1/h  (1 h)1/h  e
Hence f(x) is not differentiable at x = 1 but it is lim  1
h0 h  e  lim
e  
e h0 eh
3.54  DIFFERENTIAL CALCULUS FOR JEE MAIN AND ADVANCED
1
Now (1 + h)1/h 
2  5e h
1 1  h 2 h3   1 h h2  = lim =2
ln(1 h)  h   ....   h    .....  h 0 
1
h 2 3 2 3 4
= eh e 
 e.e 
3e  1 h
 1 h h2
 h  

.... 
Hence f is continuous but not derivable at x = 0.
2 3 4 
e   1
lim Let f be a continuous function from
 f(I0 +) = e h 0 h
R to R such that f is differentiable at 0. Suppose that
  1 h h2  
  h  2  3  4 ....   1 h h 2
  1 
 f(1/n) = 0 for all n  N.
e    .... ( 1)
  2 3 4  (i) Prove that f(0) = 0. (This fact does not requires
 
lim  
the differentiability of f)
h0  1 h h2 
 h    .... 
2 3 4  
1
(ii) Prove that f (0) = 0.
e e 2
 

Since f(I0 )  f(I0 )


+ –

 f(x) is discontinuous at all integral points and 1


(i) Since f is continuous at 0 and lim    0 ,
hence non-differentiable. n  n 

Let we must have f (0)  lim f  1   0 ,


n   n 
   {x}  { x}  
since we are given that f  1   0 for all n  N.
  2e x   5 


 n  e [x]  [ x] 
x
 

1


for x  0
n
  3 e |x|
 (ii) By part (i) we have f(0) = 0 and so
f(x) =   
 0 for x  0  f ( x ) – f (0 )   f (x) 
 f (0) = xlim   = lim  
 1 e
|x |  {x}   x  0  x   x 
 x  | x | {x} for x  0
 Thus, since 1  0 and 1  0 as n ,
n n
where [ ], { } represents integral and fractional part
 1
functions respectively. Compute the one-sided  f  
derivatives at x = 0 and comment on the continuity and f (0) = lim 
 n    0,
differentiability at x = 0. n   1 
 n 
 1  ehh   
h 0
f (0+) = lim  h  h  because f  1   0 for all n  N.
h 0 n
h
2h

(1 e ) If f(x) = 3x10 – 7x8 + 5x6 – 21x3 + 3x2 – 7 then
= lim h =–1
h  0 (2h)h f(1  h)  f(1)
find the value of lim .
 1 h  h


x 1 h 3  3h
2 e | h| 5
 h  n  e  1  0    f (x) = 3x10 – 7x8 + 5x6 – 21x3 + 3x2 – 7
 1 
f (0 ) = lim
–  3  e h 

f(1  h)  f(1) h

h 0 l = lim ·
h h 0 h h(h 2  3)
 1h  1 1
2e  5  l = lim  f '(1)· = – f ' (1) ...(1)
h   h 3 3 2
 1  h 0
 3  eh 
= lim   Now, f (x) = 30x – 56x + 30x – 63x2 + 6x
9 7 5
h 0 h f (1) = 30 – 56 + 30 – 63 + 6 = – 53
1 53
From (1), l = – (– 53) = .
3 3
DIFFERENTIABILITY  3.55

If f (x + y) = f (x) + f (y) + | x | y + xy2, 2a 3  2b3


 x, y  R and f (0) = 0, then prove that f is differentiable = .
9a 2
for all x  R, but it is not twice differentiable at x = 0.
and f (a–) = hlim f ' (a  h )
0

f (x  h)  f (x) f (h)  | x | h  xh 2  8 2b3 


f (x) = lim  lim
= lim  (a  h) 
h h 2
9 a  h  
h 0 h 0
 f(0) = 0, we have
h0
9
 f (h)  f (0)  8a 2b3 8a 3  2b3
f (x) = lim   | x |  xh  = + = .
h 0  h  9 9a 2 9a 2
f (x) = f  (0) + | x | = | x |.  f (a+)  f  (a–) and hence f (a) does not exist.
Hence, f is differentiable for all x  R.
It is easily seen that f  (x) = | x | is not differentiable at Let f : [0, 1]  R be defined as
x = 0. So, f is not twice differentiable at x = 0.  3  1 
 x (1  x)sin  2  if 0  x  1
If the function f(x) = |x – a|.g(x), where f(x) =  x  .
0 if x  0
g(x) is a continuous function, is differentiable at x = a 
then find g(a). Then prove that (i) f(x) is differentiable in [0, 1]
We have (i) f(x) is bounded in [0, 1]
(ii) f (x) is bounded in [0, 1].
f(a  h)  f(a) hg(a  h)  0
f (a+)= lim  lim = g(a). 1
h0 h h0 h h 3 (1  h)sin 0
[ g is continuous at x = a we have f (0+) = lim h2 =0
lim g(a  h )  lim g(a  h ) = g(a)] h 0 h
h0 h 0
1
f (a )  f (a  h ) 0  hg(a  h ) (1  h)3 (  h)sin 0
f (a–) = lim  lim = –g(a) (1  h)2
h 0 h h  0 h f (1–) = lim
Since, f is differentiable at x = a, therefore we have h0 h
g(a) = – g(a) 1
= lim  (1  h)3 sin = – sin 1
 g(a) = 0. h0 (1  h)2
 2 (a 2  b 2 ), 0  x  b Hence f is derivable in [0, 1].
3
 Obviously f is continuous in a closed interval [0, 1]
3
 2 2 4 2 2b and hence f is bounded.
If f(x) =  a  x  , b  x  a,
3 9 9x Now f (x)
 2 3  3
3
 9x (a  b ), x  a 4  1  2  1 2 3
 (x  x ) cos  2   3   sin 2 (3x  4x ) x  0
=  x  x  x
then find whether f (a) exists. 0 if x  0

We have,
and xlim f (x)  (0)  sin1(3  4)   sin1 .
 1
 0, 0  x  b Thus, f (x) is continuous in [0, 1] and hence it is also
 bounded.
 8 2b3
f (x) =  x  2 , b  x  a,  x  a if x  0
9 9x f (x) =  and
 2 3  x  1 | if x  0
3
 9x 2 (a  b ), x  a x  1 if x  0
g (x) =  2
 2 (a 3  b3 )  (x  1)  b if x  0
 f (a+) = hlim f ' a  h  = lim  ·
0 h 0 9 (a  h) 2  where a and b are non negative real numbers.
 
3.56  DIFFERENTIAL CALCULUS FOR JEE MAIN AND ADVANCED
Determine the composite function gof. If (gof) (x) is Let f : R  R be a real valued
continuous for all real x, determine the values of a
function  x, y  R such that |f(x) – f(y)| < sin4(x – y).
and b. Further, for these values of a and b, is gof
differentiable at x = 0 ? Justify your answer. Prove that f(x) is a constant function.
 x  a if x  0 Since |f(x) – f(y)| < sin4(x – y) ,
f(x) =  and x  y|f(x) – f(y)| < | sin4(x – y) |
 x  1 | if x  0
Dividing by | x – y | on both sides,
x  1 if x  0
g(x) =  f (x) – f (y) sin 4 (x – y)
2
(x  1)  b if x  0  
xy xy
Taking lim y  x, we get using domination law of
limits that
f (x)  f (y) sin 4 (x – y)
lim  lim
y x xy y x xy
f (x)  f (y) sin 4 (x – y)
 lim  lim
yx xy y x xy
f(x)  1 f(x)  0
G(x) = g(f(x)) =  2
 |f (x)|  0
(f(x)  1)  b f(x)  0  |f (x)| = 0 ( |f (x)|  0, in general)
 x  a  1 if f(x)  0 or x  a  f (x) = 0
 
 (x  a  1)  b if f(x)  0 or  a  x  0
2
f(x) = c (constant)

 (1  x  1)2  b if f(x)  0 or 0  x  1
 A function f : R  [1, ) satisfies
=   x2  b the equation f(xy) = f(x)f(y) – f(x) – f(y) + 2. If f is
 (x  1  1)2  b if f(x)  0 or x  1 differentiable on R and f(2) = 5, then show that

   (x  2)  b
2
f ( x)  1
(gof) (x) is continuous : f (x) = .f ' (1) .Hence, determine f(x).
x
Continuity at x = – a We have
lim G( a  h) = lim ( a  h)  a  1 = 1
h0 h0 f(xy) = f(x)f(y) – f(x) – f(y) + 2 ...(1)
lim G(a  h) = lim ( a  h  a  1)2  b h
h 0 h0 f ( x  h )  f ( x) f x1     f ( x)
Now, f (x) = lim = lim   x 
=b+1 h0 h h0 h
 b + 1 = 1  b = 0. h h
Continuity at x = 0 f (x)f 1    f (x)  f 1    2  f ( x)
 x  x
lim G (0  h) = (a – 1)2 (as b = 0) = lim
h0 h 0 h
[using (1)]
lim G(0  h) = 0  a – 1 = 0  a = 1
h
f 1    2
h 0
x  2 x  1
 x f ( x)  1
 G (x) =  x = lim . ...(2)
2
1  x  1 h
 2
h0 x
(x  2) x  1 x
Y Putting x = 1 and y = 2 in equation (1), we have
f(2) = f(1) f(2) – f(1) – f(2) + 2
X
5 = 5(f(1) – f(1) – 5 + 2 [ f(2) = 5 ]
 f(1) = 2.
 G is differentiable at x = 0. Now (2) reduces to
DIFFERENTIABILITY  3.57
h When x = 0, f(0) = 0 + c, i.e., b = c
f 1    f (1)
f ( x)  1  x f ( x)  1 [ f(0) = b (given)]
f(x) = . lim  .f ' (1)
x h0 h x
Thus, f  x  =
ax  b ...(1)
x
To find f(x), write the above equation as 2 2
df dx  x  y   f(x)  f(y)
 f ' (1) Again, putting y = 0 in f   we get
f 1 x  2  2
Integrating both sides w.r.t. x f ( x )  f (0 ) f ( x )  b
we get ln |f–1| = f (1).ln|x| + lnc. f  x  = 
2 2 2
Let f (1) = a
f ( x )  b ax
ln|f–1| = a.ln|x|c  From (1), = +b
Now, using the condition f(1) = 2, we have 2 2
ln |2 – 1| = a.ln c  c = 1
 f(x) = ax + b.
Now, f – 1 = ± |x|a Find a function continuous and
f(x) = 1 ± |x|a derivable for all x and satisfying the functional relation,
Using the condition f(2) = 5, we have f (x + y) . f (x  y) = f2 (x) , where x and y are independent
5 = 1 ± 2a
variables and f (0)  0 .
This gives a = 2, using + sign.
Hence, we have f(x) = 1 + |x|2 Put y = x and x = 0 to get
which can also be written as f(x) = 1 + x2. f (x) . f ( x) = f2 (0) ...(1)
 x  x 2  x 3  ....  x n  f (x  h)  f (x)
Now f  (x) = lim
Let f  1  h0 h
 n  f 2 (0)
 f (x1 )  f (x 2 )  f (x 3 )  ....  f (x n )  f (x  h)  f (  x)
= lim
=  h0
h
 n 
where xi are any real numbers and n  N. If f(x) is 1 f (x  h) . f (  x)  f 2 (0)
= . lim
differentiable and f (0) = a and f(0) = b, find f(x). f (  x) h  0 h
 x  y   f(x)  f(y) h h h h
f     x   . f     x    f 2 (0)
We have, f   1
lim  2  2   2  2 
 2  2 =
f ( x) h  0
{taking x1 = x, x2 = y, n = 2} h
h
This holds for any real x, y. So y is independent of f 2    f 2 (0)
dy 1  
2
x, i.e., 0. = lim
dx f (  x) h  0 h
Differentiating w.r.t x,  f  h   f(0)  f  h   f(0) 

 
     2 
f  
x  y  1  dy  1 dy 1   2  
 · 1    f (x)  f (y) = lim
 2  2  dx  2 dx 2 f (  x) h  0 h
1 xy 1 2
 f    f (x) f (x) 
f (0)
2  2  2
= . 2 f (0) . f  (0) = f(x)
Taking x = 0 and x in place of y we get 2 f 2 (x) f (0)
1 f '  0  x   1 f ' (0 ) f  (x) f  (0)
   = = k (say)
2  2  2 f (x) f (0)
   ln  f (x) = k x + c
 f '  x   f ' (0)  a (given ).
2   f (x) = ekx + c = ekx ·ec
x   x  f(x) = ± ekx ·ec
Integrating w.r.t. , f  x  = a. + c
2 2 2  f(x) = a.ekx where a and k are arbitrary constants.
3.58  DIFFERENTIAL CALCULUS FOR JEE MAIN AND ADVANCED

Let f be a function such that Since f(0) = 0, c = 0


f (0) = 1  b = 1 and f(1) = 2  a = 1.
f(x + f(y)) = f(f(x)) + f(y)  x, y  R Hence f(x) = x2 + x.
and f(h) = h for 0 < h <  where  is a small positive
quantity. Determine f”(x) and f(x). Let f(x) be a positive differentiable
function satisfying
Given f(x + f(y)) = f(f(x) + f(y))...(1)
Putting x = y = 0 in (1), xy
f(0 + f(0))=f(f(0)) + f(0)  f(f(0))=f(f(0)) + f(0)
f   f(x).f(y)  x , y  R, where f(0) = 2.
 2 
 f(0) = 0 ...(2) Find f(x).
f(x  h)  f(x) Put x = y = 0, f(0) = f (0 ) 2
Now f(x) = lim (for 0 < h < )
h0 h  f(0) = 0, 1
putting x as h and y as x in (1)
Since f is a positive function f(0) = 1.
f(f(h)) f(h)
= lim = lim ( f(h) = h) f  x  h   f(x)
h0 h h  0 h f (x) = lim
h
h0 h
= lim  1. f(2x).f(2h)  f(x)
h0 h
=
Integrating both sides with limits 0 to x h
f(x) = x Replace x by 2x and y = 0 in the rule :
 f(x) = 0. f(x) = f (2 x ).f (0)

Let f(x) be a thrice differentiable func- f(2h)


f(x)  f(x)
tion satisfying f(x + y) = f(x – y) + y[f(x + y) + f(x – y)] f(0)
f (x) = lim
where f(0) = 0, f(0) = 1, and f(1) = 2. Find f(x). h 0 h
Put y = x f(x)( f(2h)  1)
= lim
f(2x) = f(0) + x[f(2x) + f(0)] h 0 h
f(2x) = xf(2x) + xf(0) + f(0) (f(2h)  1)
Differentiate both sides w.r.t. x = f(x). lim
h 0 h.( f(2h)  1)
f (2x). 2 = f (2x) + xf(2x).2 + f (0)
f (2x) = 2xf(2x) + f (0) f(2h)  1
= f(x). lim
Replace 2x by x, h 0 2h
f (x) = xf(x) + f (0) f (x) = f(x) . f (0)
Again differentiate both sides w.r.t. x f (x) = 2f(x)
f(x) = xf(x) + f(x) n f(x) = 2x + c
 xf(x) = 0 f(x) = e2x + c
 f(x) = 0 f(0) = 1 c = 0
 f(x) = ax2 + bx + c  f(x) = e2x

1. Right Hand Derivative (R.H.D.) f is differentiable at x = a if and only if


 f (a  h)  f (a) L.H.D. = R.H.D.
= f '(a )  lim 2. A function will fail to have a derivative at a point
h 0 h
Left Hand Derivative (L.H.D.) where the graph has
f (a  h)  f (a) (i) a corner, where the one-sided derivatives

= f '(a )  lim differ.
h 0 h
DIFFERENTIABILITY  3.59
(ii) an oscillation point, where the one-sided (ii) Modulus function and signum function are
derivative(s) does (do) not exist. non differentiable at x = 0. Hence, y =  f(x)
(iii) a vertical tangent, where the absolute value and y = sgn(f(x)) should be checked at points
of slope of the secant approaches  where f(x) = 0.
(iv) a discontinuity. (iii) Power function y = xp, 0 < p < 1 is non-
differentiable at x = 0. Hence, y = f(x)p should
3. The curve y = f(x) has a vertical tangent line at the
be checked at points where f(x) = 0.
point (x0, f(x0)) provided that f is continuous at x0 (iv) The inverse trigonometric functions
and f (x)   as x  x0 –1 –1 –1
y = sin x, cos x, cosec x, and sec x
–1

4. The graph of a continuous function f has a cusp are not differentiable at the points x = ± 1.
–1 –1
at x0 if f(x) approaches  from one side and – Hence, y = sin (f(x)), cos (f(x)),
–1 –1
from the other side. cosec (f(x)), and sec (f(x)) should be
checked at points where f(x) = ± 1.
5. If a function f is differentiable at x = a then it
(v) Greatest integer function and fractional part
must be continuous at x = a. functions are non differentiable at all integral
If f is continuous at x = a, then f may or may not be x. Hence, y = [f(x)] and y = {f(x)} should be
differentiable at x = a. checked at points where f(x) = n, n I.
6. If a function f is discontinuous at x = a then it is (vi) Further, a function should be checked at all
non-differentiable at x = a. those points where discontinuity may arise.
7. If a function f is non-differentiable at x = a but 12. An alternative limit form of the derivative
both the one-sided derivatives exist (though being f (x)  f (a)
f (a) = lim .
unequal), then f is continuous at x = a. x a x–a
8. If a function f is defined only in the left f (a  h)  f (a  h)
13. If f (a) exists, then lim = f (a).
neighbourhood of a point x = a (for x  a), it is said h 0 2h
to be differentiable at a if 14. (i) If f (a) exists and (h)  0 as h  0, then
f (a  h)  f (a) f (a   (h))  f (a)
L.H.D. = f (a–) = hlim exists. In such lim
0
h = f (a)
a case f (a) = f (a ).
– h 0  (h)
Similarly, if a function f is defined only in the right (ii) If f (a) exists, and (h)  0 and (h)  0 as
neighbourhood of a point x = a (for x  a), it is said f (a   (h))  f (a  (h))
to be differentiable at a if h  0, then lim
h 0  (h)  (h)
R.H.D. = f (a+) = lim f (a  h)  f (a) exists.
h0 = f (a)
h 15. Let the function y = f(x) be defined by
In such a case f (a) = f (a+). x= x(t) and y= y(t), where t is the parameter.
9. A function f(x) is said to be differentiable in an y(t  )  y(t)
open interval (a, b) if it is differentiable at all dy  y(t  )  y(t)
 lim  lim
interior points in (a, b). Then, dx 0 x(t  )  x(t) 0 x(t  )  x(t)
10. A function f(x) is said to be differentiable in a 
closed interval [a, b] if 16. In general, the limit of the derivative may not exist,
(i) it is differentiable at all interior points in (a, b). but when it exists then it is equal to the value of
(ii) R.H.D. = f (a+) exists at the left endpoint a. the derivative.
(ii) L.H.D. = f (a–) exists at the right endpoint b. In such a case, we say that the derived function
f (x) is continuous, or the function f(x) is
11. (i) All polynomial, exponential, logarithmic and continuously differentiable.
trigonometric functions (inverse
trigonometric not included) are differentiable 17. (i) If xlim
a 
f (x) and xlim
a 
f (x) both exist or are
at each point in their domain.
infinite then xlim
a 
f (x) = f (a–)
3.60  DIFFERENTIAL CALCULUS FOR JEE MAIN AND ADVANCED
i.e L.H.L. of f (x) = L.H.D. of f(x) at x = a (i) cf(x) is differentiable at x = a, where c is any
and lim f (x) = f (a+) constant.
x a (ii) f(x)  g(x) is differentiable at x = a.
i.e R.H.L. of f (x) = R.H.D. of f(x) at x = a. (iii) f(x). g(x) is differentiable at x = a.
(a) If lim f (x) = lim f (x), then f(x) is
x a x a
(iv) f(x)/g(x) is differentiable at x = a, provided
differentiable and its derivative f (x) is g(a)  0.
continuous at x = a. 20. If f(x) is differentiable at x = a and g(x) is
non-differentiable at x = a, then we have the
(b) If xlim f (x)  xlim f (x), then f(x) is
a  a  following results :
non-differentiable and its derivative f (x) (i) Both the functions f(x) + g(x) and f(x) – g(x)
is discontinuous at x = a. are non-differentiable at x = a.
(c) If xlim
a 
f (x) and xlim
a 
f (x) are infinite, (ii) f(x). g(x) is not necessarily non-differentiable
at x = a. We need to find the result by first
then f(x) has an infinite derivative at x = a
and hence it is non-differentiable there principles.
and f (x) is discontinuous at x = a. 21. If f(x) and g(x) both are non-differentiable at x = a,
then we have the following results.
(ii) If any of the limits xlim
a
f (x) or xlima 
f (x)
(i) The functions f(x) + g(x) and f(x) – g(x) are
does not exist, then we cannot conclude not necessarily non-differentiable at x = a.
anything about the differentiability of the However, atmost one of f(x) + g(x) or
function. In such a case, we should try to f(x) – g(x) can be differentiable at x = a. That
find the derivative using its basic definition is, both of them cannot be differentiable
(i.e. first principles). simultaneously at x = a.
18. A function f(x) is twice differentiable at x = a if its (ii) f(x). g(x) and f(x)/g(x) are not necessarily non-
derivative f (x) is differentiable at x = a i.e. differentiable at x = a. We need to find the
f (a  h)  f (a) result by applying first principles.
the limit f"(a) = lim exists.
h 0 h 22. If f(x) is differentiable at x = a and g(x) is
19. If f(x) and g(x) are differentiable at x = a, then the differentiable at x = f(a) then the composite
following functions are also differentiable at x = a. function (gof)(x) is differentiable at x = a.

SINGLE CORRECT ANSWER TYPE


x (A) a = x0, b = –x0 (B) a = 2x0, b = –x02
1. If f(x) = , x  0 and f(0) = 0 then, (C) a = a = x0 , b = –x0 (D) a = x0, b = – x02
2
1  e1 / x
(A) f(x) is continuous at x = 0 and f (x) = 1 3. The number of points where function
3
(B) f(x) is discontinuous at x = 0 f(x) = minimum {x – 1, –x + 1, sgn (–x)} is
(C) f(x) is continuous at x = 0 and f (x) does not continuous but not differentiable is
exists (A) One (B) Two
(D) f(x) is continuous at x = 0 and f (x) = 0 (C) Zero (D) None of these
 x2 if x  x 0 4. Total number of the points where the function
2. Let f(x) =  f(x) = min {|x| – 1, |x – 2| – 1| is not differentiable
ax  b if x  x 0 (A) 3 points (B) 4 points
The values of the coefficients a and b for which (C) 5 points (D) None of these
the function is continuous and has a derivative at
5. The set of values of x for which the function
x0, are
defined as
DIFFERENTIABILITY  3.61

1  x x 1 (D) f(x) is discontinuous at finite number of


(1  x)(2  x) 1  x  2 points.
f (x) = 
3  x x2 12. Let f (x) is a function continuous for all x  R
fails to be continuous or differentiable, is except at x = 0 such that f (x) < 0  x (–, 0)
(A) {1} (B) {2} and f(x) > 0  x  (0, ). If lim f(x)  3 ,
(C) {1, 2} (D)  x 0
lim– f(x)  4 and f (0) = 5, then the image of the
6. The number of points where x 0
2/3 3 point (0,1) about the line y limf(cos3 x  cos2 x)
f(x) = (x + 1) + |x – 1| , is non-differentiable is x0
(A) 1 (B) 2 2 3
= x lim f(sin x  sin x) , is
(C) 3 (D) none x0

3  12  9   12 9 
7.
2
The set of all points where f(x) = x | x | – |x| – 1 (A)  ,  (B)  , 
 25 25   25 25 
is not differentiable is
(A) {0} (B) {–1, 0, 1}  16  8   24  7 
(C) {0, 1} (D) none of these (C)  ,  (D)  , 
 25 25   25 25 
8. It f(x) is a differentiable function from R  Q 13. Let f be an injective and differentiable function
100 such that f(x). f(y) + 2 = f(x) + f(y) + f(xy) for all
then  (1) f(r) is
r 0
r
non negative real x and y with f (0) = 0, f (1) = 2
 f(0), then
(A) 0 (B) –1 (A) x f (x) – 2f(x) + 2 = 0
(C) 1 (D) none (B) x f (x) + 2 f(x) – 2 = 0
| x |  x(31/x  1) (C) x f (x) – f(x) + 1 = 0
9. The function f(x) = , x  0, (D) 2 f(x) = f’(x) + 2
31/x  1
f(0) = 0 is 14. Let f be a function such that f(x + y) = f(x) + f(y)
2
(A) discontinuous at x = 0 for all x and y and f(x) = (2x + 3x) g(x) for all x
(B) continuous at x = 0 but not differentiable there where g(x) is continuous and g(0) = 3. Then f (x)
(C) both continuous and differentiable at x = 0 is equal to
(A) 9 (B) 3
(D) differentiable but not continuous at x = 0 (C) 6 (D) none
15. Let f(x) be a function such that f(x + y) = f(x) + f(y)
 g(x).cos 1 if x  0
and f(x) = sin x g(x) for all x, y  R. If g(x) is a
10. Let f(x) =  x where g(x) is
0 if x  0 continuous function such that g(0) = K, then f’(x)
is equal to
an even function differentiable at x = 0, passing (A) K (B) Kx
through the origin. Then f (0) (C) Kg(x) (D) none
(A) is equal to 1 (B) is equal to 0
(C) is equal to 2 (D) does not exist  3x 2  2x  1 1
 6x 2  5x  1 for x  3 1
(x 2  2x  3  sin  x)n  1 16. Let f(x) =  then f   
11. Let f (x) = lim , then 1 3
n (x 2  2x  3  sin  x) n  1
 4 for x 
 3
(A) f(x) is continuous and differentiable for all (A) is equal to –9 (B) is equal to –27
x  R. (C) is equal to 27 (D) does not exist
(B) f(x) is continuous but not differentiable for 17. Given f(x) is a differentiable function of x,
all x  R. satisfying f(x) . f(y) = f(x) + f(y) + f(xy) – 2 and that
(C) f(x) is discontinuous at infinite number of f(2) = 5. Then f(3) is equal to
points.
3.62  DIFFERENTIAL CALCULUS FOR JEE MAIN AND ADVANCED
(A) 10 (B) 24 (A) Continuous but non-differentiable at x = 0
(C) 15 (D) none (B) Differentiable at x = 0
é 2x + 1 ,x Î Q (C) Discontinuous at x = 0
18. The function f(x) = ê 2 is (D) None of these
ëx - 2x + 5 ,x Ï Q
ê
(A) continuous no where 26. Let f(x) = cos x and g(x) =
min imum {f(t) : 0 £ t £ x} , x Î [0, p ]
(B) differentiable no where
(C) continuous but not differentiable exactly at
g(x) ={ sin x - 1 , x >p
one point then
(D) differentiable and continuous only at one (A) g(x) is discontinuous at x = 
point and discontinuous elsewhere (B) g(x) is continuous for x [0, )
(C) g(x) is differentiable at x = 
19. Let f(x) be differentiable at x = h then
(D) g(x) is differentiable for x [0, )
(x  h) f(x)  2h f(h)
lim is equal to 27. If f(x + y + z) = f(x). f(y) . f(z) for all x, y, z and
x h xh f(2) = 4, f (0) = 3, then f’(2) equals
(A) f(h) + 2hf (h) (B) 2 f(h) + hf (h) (A) 12 (B) 9
(C) hf(h) + 2f (h) (D) hf(h) – 2f (h) (C) 16 (D) 6
20. If f(x) . f(y) = f(x) + f(y) + f(xy) – 2  x, y R and if 28. If f(x) = klim
2k –1
(cos x) + cos (sin x); then the points

f(x) is not a constant function, then the value of of non-differentiability of f(x) is
f(1) is (A) x = 1 (B) x = – 1
(A) 1 (B) 2 (C) x = n(n  1) (D) none of these
(C) 0 (D) – 1 29. If f(x) = Max. {1, (cos x + sin x),
–1
21. If f(x) = |1 – x|, then the points where sin (f |x|) is (sin x – cos x)} 0  x  5/4, then
non-differentiable are (A) f(x) is not differentiable at x = /6
(A) {0, 1} (B) {0, –1} (B) f(x) is not differentiable at x = 5/6
(C) {0, 1, –1} (D) none of these (C) f(x) is continuous for x [0, 5/4]
22. Let f(x) be defined for all x  R and the continuous. (D) None of these
2
Let f(x + y) – f(x– y) = 4xy  x, y =  R and 30. Let f : R  R satisfying |f(x)|  x  x  R, then
f(0) = 0 then (A) ‘f’ is continuous but non-differentiable at x = 0
(A) f(x)is bounded (B) ‘f’ is discontinuous at x = 0
1  1 (C) ‘f’ is differentiable at x = 0
(B) f(x) + f    f  x    2 (D) None of these
x  x
1 1 31. Choose the incorrect statement given that f is
(C) f(x)  f    f  x    2 differentiable
x  x
(D) none of these (A) If f is odd and f (c) = 3 then f (–c) = 3
(B) If f is even and f (c) = 3 then f (–c) – 3
23. If g(x) exists for all x, g (0) = 2 and (C) If f is even then f (0) = 0
y x
g(x + y) = e g(x) + e g(y)  x, y. Then (D) If f is even then f (0) = 0
x x
(A) g(2x) = –2e g(x) (B) g’(x) = g(x) + 2e
32. Consider the function f(x)
(C) lim g(h)/h = 3 (D) None of these
h 0 x3 if x  0
24. If y = |1–|2–|3–|4–x|||| ; then number of points where  2
x if 0  x  1
y is not differentiable; is = ,
(A) 1 (B) 3 2x  1 if 1  x  2
(C) 5 (D) > 5 x 2  2x  3 if x  2

ì x then f is continuous and differentiable for
ï , x¹0
25. f(x) = í 2x 2 + | x | then f(x) is (A) x  R (B) x  R – {0, 2}
ï1 , x=0 (C) x  R – {2} (D) x  R – {1, 2}
î
DIFFERENTIABILITY  3.63
33. The number of points on [0, 2] where [x]  {x} x  1
 x{x}  1 0  x  1 
40. If f(x) =  1 , then [where [ . ] and
f(x) =   [x]  {x}2 x  1
2  {x} 1  x  2 
fails to be continuous or derivable is { . } represents greatest integer part and fractional
(A) 0 (B) 1 part respectively.]
(C) 2 (D) 3 (A) f(x) is continuous at x = 1 but not differentiable
34. Which one of the following functions best (B) f(x) is not continuous at x = 1
represent the graph as shown adjacent ? (C) f(x) is differentiable at x = 1
(D) lim f(x) does not exist
x1
41. If f(x) has isolated point discontinuity at x = a
such that |f(x)| is continuous at x = a then
(A) |f(x)| must be differentiable at x = a
(B) lim f(x) does not exist
x a
(C) lim f(x) + f(a) = 0
1 1 x a
(A) f(x) = 2
(B) f(x) = (D) f(a) = 0
1 x 1 | x |
–| x | |x| 42. Let f be a differentiable function on the open
(C) f(x) = e (D) f(x) = a (a > 1) interval (a, b). Which of the following statements
35. The number of points where the function must be true?
2 2
f(x) = (x – 1) | x – x – 2| + sin (| x |) I. f is continuous on the closed interval [a, b]
is not differentiable is II. f is bounded on the open interval (a, b)
(A) 0 (B) 1 III. If a < a1 < b1 < b, and f(a1) < 0 < f(b1), then
(C) 2 (D) 3 there is a number c such that a1 < c < b1 and
36. Let f(x) be differentiable at x = h then f(x) = 0
(x  h) f(x)  2h f(h) (A) I and II only (B) I and III only
lim is equal to (C) II and III only (D) only III
x h xh
(A) f(h) + 2hf (h) (B) 2 f(h) + hf (h) 43. Let f : R  R, f(x – f(y)) = f(f(y)) + xf(y) + f(x) – 1 
(C) hf(h) + 2f (h) (D) hf(h) – 2f (h) x, y  R, if f(0) = 1 and f (0) = 0, then
x2
37. The function f(x) = maximum  x(2  x),2  x  (A) f(x) = 1 –
2
(B) f(x) = x2 + 1
is non-differentiable at x equal to  2x  1 
(A) 1 (B) 0, 2 (C) f(x) =   (D) none of these
 x 1 
(C) 0, 1 (D) 1, 2
44. If f : [–2a, 2a]  R is an odd function such that
 sin | x 2  5x  6 | f(x) = f(2a – x) for x  (a, 2a). if the left hand
 , x  2,3
38. Let f(x) =  x 2  5x  6 . derivative of f(x) at x = a is zero, then the left hand
 1 , x  2 or 3 derivative of f(x) at x = –a is
The set of all points where f is differentiable is (A) 1 (B) –1
(A) (, ) (B) (–, ) – {2} (C) 0 (D) none
(C) (–, ) – {3} (D) (–, ) – {2, 3} 45. Suppose that the differentiable functions u, v, f,
1 f(a  2h 2 )  f(a  2h 2 ) g : R  R satisfy lim u(x) = 2, lim v(x) = 3,
39. If f (a) = , then lim
h 0 f(a  h 3  h 2 )  f(a  h 3  h 2 )
= x x 
4 f (x) f(x)
(A) 0 (B) 1 lim f(x) = lim g(x) =  and  u(x) = (vx)
x x g(x) g(x)
(C) –2 (D) none
3.64  DIFFERENTIAL CALCULUS FOR JEE MAIN AND ADVANCED
f(x) (A) 2 (B) 3
then lim is equal to (given that it exists)
x g(x) (C) 4 (D) dne
(A) 1 (B) 1/2 [x]  {x} x  1
(C) 2 (D) None 
49. If f(x) =  1 , then
46. Suppose f is a differentiable function such that  [x]  {x}2 x  1
f(x + y) = f (x) + f (y) + 5xy for all x, y and f (0) = 3. 
The minimum value of f (x) is [where [ . ] and { . } represent greatest integer and
(A) – 1 (B) – 9/10 fractional part functions respectively]
(C) – 9/25 (D) None (A) f(x) = is continuous at x = 1 but not differentiable
(B) f(x) is not continuous at x = 1
47. Let h(x) be differentiable for all x and let
x (C) f(x) is differentiable at x = 1
f(x) = (k x + e ) h(x) where k is some constant. If
h(0) = 5, h’(0) = –2 and f’(0) = 18 then the value of (D) lim f(x) does not exits
x 1
k is equal to
(A) 5 (B) 4 50. Let f (x) be continuous at x = 0 and f (0) = 4 then
(C) 3 (D) 2.2 2f(x)  3f(2x)  f(4x)
value of lim is
48. If for a function f(x) : f(2) = 3, f (2) = 4, then x0 x2
(A) 11 (B) 2
lim [f(x)], where [ . ] denotes the greatest integer
x2 (C) 12 (D) none
function, is
MULTIPLE CORRECT ANSWER TYPE FOR JEE ADVANCED
51. Let f (x)=x 1( [x]  [x]) , then which of the the greatest integer function then f(x) is
following statement(s) is/are correct? (A) discontinuous  x 
(where [x] denotes greatest integer function) (B) continuous  x R
(A) f (x) is continuous at x = 1 (C) non differentiable  x 
(B) f (x) is derivable at x = 1 (D) a periodic function with no fundamental period
(C) f (x) is non-derivable at x = 1
55. Let f and g be two functions defined as follows :
(D) f (x) is discontinuous at x = 1.
x x  x for x  0
52. Which of the following function will have tangent f(x) = for all x & g(x) =  2 then
at indicated point 2 x for x  0
 x 2 sin 1 , x  0 (A) (gof)(x) & (fog)(x) are both continuous for all
 xR
(A) f(x) =  x at x = 0
 0 (B) (gof)(x) & (fog)(x) are unequal functions
, x0
(C) (gof) is differentiable at x = 0
(B) f(x) = sgn (sin x) at x = 
2 (D) (fog)(x) is not differentiable at x = 0 .
(C) f(x) = sgn (x ) at x = 0
11/10 56. If a function f : R  R satisfies
(D) f(x) = |x – 1| at x = 1
| 3k (f(x + ky) – f(x – ky))|  1 for every n  N and
53. Let f (x) = sin x, g (x) = sgn (x) and h (x) = gof(x) for all x y  R, then
then (A) |3n (f(x + ny) – f(x – ny))|  2
(A) h(x) is discontinuous at infinite number of points 2
(B) h(x) = 0 for all x  R – I (B) |f(u) – f(v)|  n
3
(C) lim h(x) does not exist (C) f(x) is a constant function
x I
(D) f (x) = 1  x  R
(D) h(x) is periodic with period 1.
57. If we define new derivative of function f to be
tan[x]
54. If f(x) = , where [.] denotes f 3 (x  x)  f 3 (x)
[1 | log(sin 2 x  1) |] (f(x)) = lim , then
x  0 x
DIFFERENTIABILITY  3.65
3
(A) (x) = 3x f '(x)
 (D) lim does not exist.
(B) (tan x) at x = equals 6 x 0 g'(x)
4
(C) (sin x) at x = 0 equals 1 x 1 , x  1
63. If f(x) =  and g(x) = f(x) + f(|x|),
–1 3 2 2  x , x  1
(D) (cos x) at x = 0 equal – then g(x) is
4
(A) not continuous at x = –1
 1 
58. if f(x) = minimum  cos x, ,{sin x}  , 0  x  2, (B) not continuous at x = 1
 2  (C) continuous at x = 0
where { . } represents fractional part function, then (D) not differentiable at x = 0
f(x) is differentiable at
64. A function f(x) satisfies the relation
3 
(A) (B) f(x + y) = f(x) + f(y) + xy (x + y)  x, y R.
2 4 If f (0) = –1, then
 
(C) (D) (A) f(x) is a polynomial function
3 2 (B) f(x) is an exponential function
3
59. If f(x) = minimum {x – 1, – x + 1, sgn (–x)}, then f(x) (C) f(x) is twice differentiable for all x R
is (D) f (3) = 8
(A) continuous at x = 0
65. Function f(x) = max (|tan x|, cos|x|) is
(B) differentiable at x = 0
(A) not differentiable at 4 points in (–)
(C) continuous at x = 2
(D) differentiable at x = 2 (B) discontinuous at 2 points in (–)
(C) not differentiable at only 2 points in (–)
60. If [ . ] denotes greatest integer function, then for
(D) in (–) there are only 2 points where f(x) is
the function y = sin  (|x| + [x]), which of the
continuous but not differentiable
following is correct
(A) not continuous at x = –1/2 66. Let a differentiable function f(x) be such that
(B) continuous at x = 0 1 1
|f(y) – f(y)|  |x – y|  x, y R and f (x)  .
(C) differentiable in [–1, 0) 2 2
(D) differentiable in (0,1 ] Then the number of points of intersection of the
61. If f(x) is differentiable everywhere and f(a) = 0, graph of y = f(x) with
then at x = a (A) the line y = x is one
3
(A) | f | is differentiable (B) the curve y = –x is one
2
(B) | f | is differentiable (C) the curve 2y = |x| is three
(C) f | f | is differentiable (D) None of these
2
(D) f + | f | is differentiable 67. Given that the derivative f (a) exists. State which
of the following statements are true
2 1
62. Let f (x) = x sin for 0 < x  1 and f (0) = 0. If f ( h ) – f (a )
x (A) f (a) = lim
h 0 ha
g(x) = x2 for x  [0, 1] then which of the following
f (a ) – f (a  h )
statement(s) is/are correct? (B) f (a) = lim
h 0 h
(A) f (x) is differentiable in [0, 1]
f (a  2 t ) – f (a )
f(x) (C) f (a) = lim
(B) lim does not exist. t 0 t
x 0 g(x) f (a  2 t ) – f (a  t )
f(x) (D) f (a) = lim
(C) lim does not exist t 0 2t
x 0 g(x)
3.66  DIFFERENTIAL CALCULUS FOR JEE MAIN AND ADVANCED

 2 1 f : R  R such that
sin x.cos , x  0 f(x) f(y) = f(x) + f(y) – f(xy) ...(1)
68. Let f (x)   x
 1 + f(x + y) = f(xy) + f(x) f(y) ...(2)
0, x0
Reason (R) : Adding (1) and (2), we get
(A) f is continuous at x = 0 1 + f(x + y) = f(x) + f(y).
(B) f (0) = 1
(C) f (0) does not exist f(x  h)  f(x) f(h)  1
Now f (x) = lim  lim
(D) f  is not continuous at x = 0. h0 h h  0 h
= f (0).
e a /x  e  a /x
69. Let f(x) = g(x) where g is the  f(x) = ax + b where a & b can be determined by
e a /x  e  a /x using function’s value at x = 0, 1.
derivative of g and is a continuous function and –1
74. Assertion (A) : The function y = sin (cos x) is
a > 0 then lim f(x) exists if not differentiable at x = n , n I is particular
x0
(A) g(x is polynomial at = .
(B) g(x) = x dy  sin x
Reason (R) : = so the function is not
(C) g(x) = x2 dx | sin x |
(D) g(x) = x3 h(x) where h(x) is a polynomial differentiable at the points where sin x = 0.
70. Let f : R  R be a function such that, 75. Consider two functions
f(x) = sin x and g(x) = | f(x)|.
x  y f (x)  f (y)
f  = , f(0)=3 and f (0)=3, Assertion (A) : The function h(x) = f(x) g(x) is not
 3  3
then differentiable in [0, 2].
Reason (R) : f (x) is differentiable and g (x) is not
f (x) differentiable in [0, 2]
(A) is differentiable in R
x 76. Assertion (A) : The function
(B) f  (x) = 3 |x| 3
f (x) = c1e  c 2 | x | , where c1, c2 are constants
(C) f(x) is continuous in R
(D) f(x) is bounded in R is differentiable at x = 0, provided c1 = 0.
Reason (R) : e|x| is not differentiable at x = 0.
Assertion (A) and Reason (R) 77. Consider two functions f(x) = sin x and g(x) = | f(x) |.
(A) Both A and R are true and R is the correct Assertion (A) : The function h (x) = f (x) g(x) is
explanation of A. not differentiable in [0, 2]
(B) Both A and R are true but R is not the correct Reason (R) : f (x) is differentiable and g (x) is not
explanation of A. differentiable in [0, 2]
(C) A is true, R is false.
–1   [x] 
(D) A is false, R is true. 78. Consider the function f(x) = cot  sgn  
2   2x  [x] 
71. Consider the function f(x) = x – 2x and g(x) = –|x| where [ ] denotes the greatest integer function
Assertion (A) : The composite function Assertion (A) : f(x) is discontinuous at x = 1.
F(x) = f(g(x)) is not derivable at x = 0. Reason (R) : f(x) is non derivable at x = 1.
+ –
Reason (R) : F (0 ) = 2 and F(0 ) = –2.
 x 1 0  x  2
72. Assertion (A) : There is no polynomial function f 79. Let f(x) = sgn x and g(x) =  2 then
1  x 2  x  4
such that f(x + y) = f(x) + yf(f(x)) for all x, y  R
Assertion (A): The function (fog)(x) is
f(x  h)  f(x) differentiable at x = 2.
Reason (R) : f (x) = lim = f(f(x)).
h 0 h Reason (R) : If (fog) (x) is differentiable at x = a
If f is of degree n then the equation n – 1 = n2 has then g(x) is differentiable at x = a and g(x) is
no positive integral solution. differentiable at x = g(a).
73. Assertion (A) : There are exactly three functions
DIFFERENTIABILITY  3.67
80. Assertion (A) : at some points
2 p 1 2 p 1 2 p 1
f(x) = ( x  1) 1 ( x  2) 2 ....( x  n ) n is (C) Discontinuous at exactly one point
differentiable everywhere where p1, p2, ......pn N. (D) Discontinuous at infinitely many points
Reason (R) : f(x) = x|x| is differentiable everywhere. Comprehension - 3
Comprehension - 1 Suppose f, g and h be three real valued function defined
on R.
 e{x }  1, x  0
2

1
 sin x  tan x  cos x  1 Let f (x) = 2x + |x|, g (x) = (2x – |x| ) and h(x) = f(g(x))
Let f(x) =  2 , x0 3
 2x  ln(2  x)  tan x 87. The range of the function
 0, x0
where { } represents fractional part function. Lines L1 1
and L 2 represent tangent and normal to curve k(x) = 1 + (cos–1(h(x)) + cot–1(h(x))) is equal to

y = f (x) at x = 0. Consider the family of circles touching
both the lines L1 and L2. 1 7  5 11
(A)  ,  (B)  , 
81. Ratio of radii of two circles belonging to this family 4 4 4 4 
cutting each other orthogonally is 1 5  7 11
(C)  ,  (D)  , 
(A) 2 + 3 (B) 3 4 4 4 4 
(C) 2 + 2 (D) 2 – 2 88. The domain of definition of the function
l(x) = sin–1 (f(x)) – g(x)) is equal to
82. A circle having radius unity is inscribed in the
triangle formed by L1 and L2 and a tangent to it. 3 
(A)  ,   (B) (–, 1]
Then the minimum area of the triangle possible is 8 
(A) 3 + 2 (B) 3 – 2  3
(C) [– 1, 1] (D)   , 
 8 
(C) 3 + 2 2 (D) 3 – 2 2
83. If centres of circles belonging to family having equal 89. The function T(x) = f(g(f(x))) + g(f(g(x))), is
radii 'r' are joined, the area of figure formed is (A) continuous and differentiable in (– , ).
(A) 2r2 (B) 4r2 (B) continuous but not derivable  x  R.
(C) 8r 2 (D) r2 (C) neither continuous nor derivable  x  R.
Comprehension - 2 (D) an odd function.
Let f(x) be a real valued function not identically zero, Comprehension - 4
which satisfies the following conditions. A function f (x) is said to be differentiable at x = c, if
2n + 1 2n + 1
I. f(x + y ) = f(x)+ {f(y)} , n N, x, y are any f (c+) = f (c–) = a finite quantity. Also f (x) is said to be
real numbers bounded if | f (x) | < n  x for which f (x) is defined
II. f (0) 0. (where n is finite)
84. The value of f (10) is 90. Let f : R  R and c  R such that
(A) 10 (B) 0 1
(C) 2n + 1 (D) 1 lim n  f  c    f(c)  = a (a  R) then
n    n 
85. The function f(x) is (A) f (c) exists and is equal to a
(A) odd (B) f (c) does not exists
(B) even (C) f (c) exists but is not equal to a
(C) neither even nor odd (D) f (c) may or may not exist.
(D) even as well as odd
91. If lim f(x)  0 and lim f(x)  0 and f(x) be
86. The function f(x) is x x  
(A) Continuous and differentiable everywhere continuous on R. Then
(B) Continuous everywhere but not differentiable
3.68  DIFFERENTIAL CALCULUS FOR JEE MAIN AND ADVANCED
(A) f (x) is bounded on R and attains both  x  y   2  f(x)  f(y)
maximum and minimum on R. f  for all real x and y and
 3  3
(B) f (x) is unbounded on R but attains minimum f (2) = 2.
on R.
(C) f (x) is bounded on R and attains either 93. The range of f(|x|) is
maximum or minimum on R. (A) [0, ) (B) [1, )
(D) f is unbounded. (C) [2, ) (D) None of these
92. f (x) is defined on [0, 1] and 94. If f(x) = |f(|x|) – 3| for all x  R, then for g(x)
 a c (A) one non-differentiable point
f(x)= x sin( x ) x  0 where a, c  R and c > 0 (B) two non-differentiable point
0 x0 (C) three non-differentiable point
(A) f (0) exists if 0 < a < 1 (D) four non-differentiable point
(B) f (x) is continuous if a < 0
(C) f (x) is bounded and continuous if a > c 95. The number of solutions of the equation
2 2
(D) f (x) and f (x) are continuous if a > 1 + c. x + (f (|x|) ) = 9 are
(A) 0 (B) 2
Comprehension - 5 (C) 3 (D) 5
Let f : R  R be a differential function satisfying

MATCH THE COLUMNS FOR JEE ADVANCED


96. Column-I Column-II
1
 tan x, | x | 1 2
 x 1
(A) Let f(x) =  x 2  1 , | x | < 1, then f(x) is (P) –1
 , | x | 1 4
 4
not differentiable at x equal to
2 2
(B) f(x) = (x – 4) |x – 5x + 6| + cos |x| is non derivable at x equal to (Q) 1
x +y dy
(C) If sin(x + y) = e – 2, then is equal to (R) 2
dx
(D) Let f: R R is defined by the equation (S) 3
f(x + y) = f(x) f(y)  x, y R, f (0) 0 and f’(0) = 2
f '(x)
then is equal to (T) None of the above values
f(x)
97. Column - I Column - II
a x  1 if 0  x  3
(A) Let g(x) =  , if g(x) is differentiable on (0, 5) (P) 0
 bx  2 if 3  x  5
then (a + b) equals
(B) Let a and b be real numbers and let f(x) = a sin x + b 3 x + 4,  x  R (Q) 1
If f(log10 (log3 10)) = 5 then the value of f(log10 (log10 3)) is equal to
sin 4 x  sin 4 2x  sin 4 3x
(C) lim equals (R) 2
x 0 sin 3 x  sin 3 2x  sin 3 3x
1
log10 x log10 2
(D) Number of solution(s) of the equation 2  8  (x  8) is (S) 3
(T) 4
DIFFERENTIABILITY  3.69
98. Column – I Column – II
(A) If f(xy) = f(x) . f(y) and f is differentiable at x = 1 such that (P) 1
f (1) = 1 also f(1)  0, then f (7) equals
(B) If [.] denotes greatest integer function, then number of points (Q) 13
2
at which the function f(x) = |x – 3x + 2| + |sin x| – [x – 1/2],
–   x , is non differentiable, is
(C) Let f(x) = [a + 7 sin x], x  (0, ), a  I, [.] denotes greatest integer (R) 5
function. Then number of points at which f(x) is not differentiable is
(D) If for a continuous function, f, f(0) = f(1) = 0, f’(1) = 2 and (S) 3
x f(x)
y (x) = f(e ) e , then y’(0) is equal to (T) 2
99. Column-I Column-II
    sin 1 (1  {x}2 )  sin 1 1  {x}
  2   
 , x0

 2 ({x}  {x}3 )

(A) If f (x) =  k, x0 (P) 2


 Asin 1 1  {x} cos 1 1  {x}
 , x0
 2{x} 1  {x}

2  A 
is continuous at x = 0, then the value of sin2k + cos   , is
 2
(where {x} denotes fractional part function)
(B) If f (x) = [2 + 5 | n | sin x] where n  I has exactly 19 points (Q) – 2
of non-differentiability in (0, ), then possible value(s) of 'n' are
(where [x] denotes greatest integer function)
1
 1

  3    3 
x x

 x  4   4  , x  0
1 1
(C) Let f (x) =  . (R) 3
 3
     x  3  x

 4 4
 0, x0
[x 1]

If P = f (0–) – f (0+), then lim  exp   x  2  ln 4   4


 16
x
xP 
4  16
is less than (where [x] denotes greatest integer function)
1
(D) The number of points at which g(x) = is not (S) – 3
2
1
f(x)
1
differentiable where f(x) = , is (T) 5
1
1
x
3.70  DIFFERENTIAL CALCULUS FOR JEE MAIN AND ADVANCED
100. Column I Column – II
(A) If f : [0, 1]  [0, 1] is continuous, then the number (P) one
of roots of the equation f(x) = x3 is
(B) If f is derivable, then between the consecutive roots of (Q) at least one
f (x) = 0, the number of roots of f(x) = 0 is
(C) The number of values of x at which f(x) = 1 – x – 1 is (R) at most one
not differentiable
x, if x is rational
(D) Let f(x) = 2  x, if x is irrational (S) more than one

The number of values of x at which f(x) is continuous is

3
7. Let f(x) = x – 3x and
1. Let f(x) = x  1  1 for x  0, f(0) = 0. Is the
x  min(f(t) : 0  t  x) , 0  x  2

function f(x) continuous and differentiable at g(x) =  2x  5 ,2x3 .
x=0? (x  2)2 , x3
2. Let g(x) be polynomial of degree atmost two and Draw g(x) and discuss continuity and
f(x) be continuous at x = 0 and differentiable at differentiability of g in (0, 4).
x = 1. Find x2  a , 0  x  1
8. Let f(x) = 
 5xe1/x  2x 2x  b , 1  x  2
 1/x
, x0 3x  b , 0  x  1 d(f(x))
 3  e and g(x) =  3
x , 1  x  2
. If
, 0  x  1.  d(g(x))
g(x) if f(x) =  g(x) exists at x = 1, find the values of a, b and also its
 1
(x  1)2 sin  x , x 1 value.
 (1  x)2 9. If f and g be two functions having the same domain
D, if f and g be derivable at x0  D, and if
x  1 ,  2  x  0 f(x0)  g(x0), then prove that each of the function
 2
3. Let f(x) =  x  x , 0  x  2 and max. {f, g} and min. {f, g} is derivable at x0. What
2  x , 2  x  3 happens if f(x0) = g(x0)?
g(x) = f(|x|) + |f(x)| check continuity and 10. (i) f : [0, 1]  [1, 2] be a differentiable function,
differentiability of g(x) over [–2, 3]. then prove that the number of solution of
f(x) – ex(x – 1) – log2(1 + x) = 0 will be at least
4. Check the continuity and differentiability of
one.
 2 x2  1  (ii) Let f : [1, e]  [0, 1] be continuous then
f(x) = sin  x 2 .
–1
 prove that f(x) = n x has atleast one solution
 
2 3 2 x in [1, e].
5. Let f (x) = (x – 4) |(x – 6x + 11x – 6)| + .
1 | x | 11. If a differentiable function f(x) satisfies the relation
Find the set of points at which the function f(x) is
not differentiable. x x
= x f(x) – yf(y)  x, y  R & f(e) = 1/e,
y  y 
f
6. Examine the differentiability of
f(x) = x  2 2x  4 + log x
x – 2 2x  4 over its then prove that f(x) = .
domain. x
DIFFERENTIABILITY  3.71
12. Given a real valued function f(x) as follows :
x 2  2 cos x  1 1
f(x) = 4 for x < 0; f(0) = &
x 12
sin x  n(e x cos x)
f(x) = for x >. Test the
6x 2 20. Let f (x) = max. {|x2 – 2 |x| |, |x|} and g (x) = min.
continuity and differentiability of f(x) at x = 0. {|x2 – 2 |x| |, |x|} then show that f(x) is not
13. Let f be given by differentiable at 5 points and g (x) is non
x 2 sin( x –4 / 3 ) when x  0 differentiable at 7 points.
f(x) =  . 21. Consider function f : R – {–1, 1}  R.
 0 when x  0 x
Show that f has a derivative for all values of x, and f(x) = . Find f (x), draw the graph of f and
1 | x |
that f (0) = 0. Prove that f  is not, however, prove that it is not derivable at the origin.
continuous at x = 0
22. Suppose the function f satisfies the conditions :
14. Draw the graph of the function f(x) = x – |x – x2|, (i) f(x + y) = f(x)f(y) for all x and y.
–1  x  1 and discuss its continuity in [–1, 1] and
differentiability in (–1, 1). (ii) f(x) = 1 + x.g(x) where lim g(x )  1
x 0
Show that the derivative f(x) exists and f (x) = f(x)
 tan[x 2 ] for all x.
 2
 ax3  b , 0  x  1
15. If f(x) =  ax is 23. If f(x) = |x – 1| . ([x] – [–x]), then find f (1+) and
 2 cos x  tan 1 x , 1  x  2 f (1–) where [x] denotes greatest integer function.
1 24. Let f : R R be continuous function | f(x) | x4n,
differentiable in [0, 2], then a = and n N. Prove that f(x) is differentiable at x = 0.
k1
 26 2 2
25. Find the values of
b= – . Find k1 + k2 { where [ ] denotes f(3  h)2  f(3  h 2 )
4 k2 (a) lim if f (3) = 2 ;
h 0
greatest integer function}. 2h 2
f(a  2h 2 )  f(a  2h 2 ) 1
 a  1  (b) lim if f (a) = .
 x . cos  3  , x  0 h 0 h 2
4
16. If f (x)   x  is continuous
 0,   5e  2
1/x
 x 0  , x0
26. If f(x) =  3  e1/x Examine f(x), xf(x) and
but non-differentiable at x = 0, then all possible
 0 , x0
values of a. 2
x f(x) for continuity and differentiability at x= 0
a1/x  a 1/x
17. Examine the function , f (x) = x . , 1  x ,(0  x  1)
a1/ x  a 1/ x
27. f(x) =  x  2,(1  x  2) . Discuss the continuity
x  0 (a > 0) and f (0) = 0 for continuity and  4  x ,(2  x  4)

existence of the derivative at the origin . & differentiability of y = f (f(x)) for 0  x  4.
4 3 2
18. Let f(x) = x – 8x + 22x – 24 and
ax(x  1)  b , x  1

min f(t) ;x  t  x  1, –1  x  1
g(x) = x  10 ; x  1 Discuss

28. The function f(x) =  x  1 , 1  x  3 find
 px 2  qx  2 , x  3
the differentiability of g(x) in [–1, 2].
the values of the constant a, b p and q so that
19. For the function g whose graph is given, arrange (i) f(x) is continuous for all x.
the following numbers in increasing order. (ii) f (1) does not exist.
0, g(–2), g(0), g(2), g(4) (iii) f (x) is continuous at x = 3.
3.72  DIFFERENTIAL CALCULUS FOR JEE MAIN AND ADVANCED
29. Discuss the continuity on 0  x  1 & 31. Let f(x + y) – 2f(x – y) + f(x) – 2f(y) = y – 2 for all real
differentiability at x = 0 for the function f(x) = x. x and y. If f’(x) exist, find f(x).
  32. Let f(xy) = xf(x) + yf(y) for all real x and y. If f is a
1 1
sin sin  
1  where x  0, x  1/r &
differentiable function and f(0) = 0, find f(x).
x  x.sin
x 33. A differentiable function f satisfies the relation
f(0) = f(1/r) = 0, r = 1, 2, 3, .......... f(xyz) = f(x) + f(y) + f(z)  x, y, z  R+ and
30. Suppose that instead of the usual definition of f (1) = 1. Find f(x).
the derivative Df(x), we define a new of derivative, 34. A differentiable function f satisfies the relation
D*f(x), by the formula, xf(y) + yf(x) = (x + y) f(x) . f(y)  x , y  R.
If f (0) = 0, find f(x).
f 2 (x  h)  f 2 (x)
D*(x) = lim
h 0
, 35. A differentiable function f satisfies the relation
h
2 2
where f (x) means [f(x)] . Derive the formula for f(x + y) = f(x) + f(y) + f(x).f(y)  x , y  R.
computing the derivative D* of a product & quotient. If f(0)  –1 and f (0) = – 1, find f(x).

1. Let 'f' be a function whose graph is obtained by max{f (u) , 0  u  x} , 0  x  1


summing the ordinate values of the graph of h(x) = 
another function g after shifting its graph by 1 min{g(v) , 1  v  x} ,1  x  2
unit leftward and rightward. Discuss the continuity and differentiability of h(x)
1  x , | x |  1
in the interval [0, 2].
If g(x) =  discuss the continuity  0 for  e  x  1
0 , | x | 1
 1 
and differentiability of f(x). 7. If g(x) =  1  sin(l nx )  for 1  x  e .
2 
  3 
2. Check the differentiability of
ì æ 1 ö
çç ÷÷
where { } denotes the fractional part function.
ï(x - e)2 -2è e-x ø , x ¹ e
f(x) = í at x = e.  xg(x) for 1  x  e
ïî 0 , x=e f(x) =  .
 x(g(x)  1) otherwise
ì Discuss continuity and differentiability of f(x) over
ï 1 its domain.
ï x (1 + x sin ) , x>0
ïï x
8. Suppose f is differentiable on the interval (a,  )
3. Let f(x) = í – | x |(1 + x cos 1 ) , x < 0 .
ï
ï x and lim(f (x)  f (x))   (   R) .
x 
ï 0 ,x=0
ïî Prove that lim f (x)   and lim f (x)  0 .
x  x 
Show that f(x) is not differentiable at x = 0. 9. Let f be a continuous and differentiable function
4. A differentiable function f satisfies the relation in (a, b), lim f (x)  
f(x2 + y2) = (f(x))2 + (f(y))2  x , y  R. Find f(x). x a

and lim f (x)   and f (x) + f 2(x)  – 1 for


2 2 x b
5. If f(x) + f(y) = f (x 1  y + y 1  x )  x, y
a < x < b. Prove that b  a   .
 1 1 f(x)
   ,  and lim = 1, find f(x). 10. If
 2 2 x  0 x
x x x
6. Let f(x) = x3 – x2 + x + 1 and g(x) = 3 – x 1  cos2 x cos 2 .cos 2 .......cos2 n 1
f(n) = lim 2 8 2
We define a function X 0 x2
DIFFERENTIABILITY  3.73
(iv) Give any sets of consecutive points for which
and  = Xlim f(n). Find the absolute value of 27abc
 the average rates of change of the function
for which the function are approximately equal.
 a cot 1 x x  0 15. Given f(x + y + z) = f(x) f(y) f(z) for all real x, y, z if

 f(2) = 4 and f (0) = 3, then show that
g(x) =   x0 f(0) = 1 and f (2)  12 .
 b
 xc x0 16. Let f(x) = x3 – 3x2 + 6  x  R and
 
is differentiable at x = 0.  max .{f (t), x  1  t  x  2},  3  x  0
g(x)   .
 1  x for x  0
ax(x  1)  b ; x 1
 Test continuity of g(x) for x  [ 3, 1] .
11. Let f(x) =  x  2 ; 1  x  3 is
 px 2  qx  2 ; x3 17. If f(x) = x2 – 2 | x | and

continuous for all x except x = 1 but |f(x) | is  min.{f (t) : 2  t  x}, 2  x  0
g(x)  
differentiable every where and f’(x) is continuous  max .{f (t) : 0  t  x}, 0  x  3 .
k (a) Draw the graph of f(x) and discuss its
at x = 3 and | a + b + p + q | = , then find the continuity and differentiability.
18
value of k. (b) Find and draw the graph of g(x). Also, discuss
12. Two real valued, continuous and differentiable the continuity.
functions f(x) and g(x) defined on R satisfy the 18. Suppose that f(x) is continuous at a point ‘a’. Prove
conditions that it is differentiable at ‘a’ if and only if there
f(x + y) = f(x)g(y) + g(x)f(y) exists some linear function L(x) such that
g(x + y) = g(x)g(y) – f(x)f(y) | f(x)  L(x) |
tends to ‘0’ as x tends to ‘a’.
If g(0) = 0, prove that f2(x) + g2(x) = 1. |xa |
19. Suppose that f(x) is a differentiable function such
13. Sketch the graph of the function
that f (x) = g(x), g(x) exists and |f(x)|  1 for all real
 x
 1 | x | if | x | 1 x. If {f(0)}2 + {g(0)}2 = 9, then show that for some
c  (–3, 3), g(c)g"(c) < 0.
f(x) =  x
 if | x | 1  1 
 1 | x | 20. If f(x) = cos x. cos  cos x  when
and find the domain of f (x).  
x  (0, 2 – {/2, 3/2},
14. Use the graph of f to answer each question. and f(x) = 0 when x  {/2, 3/2}.
Show that f is continuous but not differentiable
at x = /2, 3/2.
e –1 / x 2 for x  0
21. Prove that the function f(x) = 
 0 for x  0
(i) Between which two consecutive points is the is infinitely differentiable at the point x = 0.
average rate of change of the function greatest ?
22. A function f : R  [1, ) satisfies the equation
(ii) Is the average rate of change of the function
f(xy) = f(x) f(y) – f(x) – f(y) + 2
between A and B greater than or less than the
If f is differentiable on R and f(2) = 5, then show
instantaneous rate of change at B ?
(iii) Sketch a tangent line to the graph between f(x)  1
that f (x) = . f (1) Hence, determine f(x).
the points C and D so that the slope of the x
2 x
tangent line is the same as the average rate of 23. Let f : R  R satisfy |f(x)|  x e  x  R, then
change of the function between C and D. show that f(x) is differentiable at x = 0.
3.74  DIFFERENTIAL CALCULUS FOR JEE MAIN AND ADVANCED

| x 1| 1 f (x0). (Note that the magnification function A


24. Let f(x) = and g(x) = , then find (x – x0) is called the differential of f at x0, and A is
11/ x 1
1 called the differential coefficient.)
f(x)
30. A differentiable function f satisfies
the points where g(x) is not differentiable.
(x – y) f(x + y) – (x + y) f(x–y)
25. If f(x + y) = f(x) + f(y)  x, y  R, f(1) = 1 then find = 4xy(x2 – y2)  x , y  R, where f(1) = 1.
2 f (tan x)  2 f (sin x) Find f(x).
lim
x 0 x 2 f(sin x) 31. A differentiable function f satisfies the relation
f(x+y) – f(x – y) = f(x). f(y)  x , y  R.
x  1 ,  1  x  0 If f (0) = 0, find f(x).
26. Let f(x) =  2 . Discuss the
x , 0  x  1
differentiability of h(x) = f(|sin x|) + |f(sin x)| in 32. Let f be defined on (a, b) and differentiable at c,
[0, 2]. where a < c < b. Suppose xn  c and zn  c,
where
27. Let f : R  R satisfy f(xy) = f(x). f(y) – f(x + y) + 1
xn < c < zn for all n.
for all x, y  R. If f(x) is differentiable and
f (0) = 1 then determine all functions ‘f’. f(x n )  f(z n )
Prove that xn  zn  f (c).
28. Suppose that function g(t) and h(t) are defined
for all values of t and that g(0) = h(0) = 0. Can 33. A differentiable function f satisfies the relation
lim g(t)/(h(t) exist? If it does exist, must it equal  xy 
t 0
f  = f(x) . f(y)  x , y  R – {–1}, where
 1  xy 
zero? Give reason for your answer. f(0)  0 and f(0) = 1. Find f(x).
29. Given a function f, we say that f is differentiable at 34. A differentiable function f satisfies the relation
x0  Df if there exists a real number A such that [1 + f(x).f(y)] f(x + y) = f(x) + f(y)  x , y  R.
| f ( x ) – f ( x 0 ) – A( x – x 0 ) | If f(0) = 0 and f (0) = 1, find f(x).
lim .
x x 0 | x – x0 | 35. Given a differentiable function f(x) defined for all
Prove that if f is differentiable at x0 then the value real x, and is such that f(x + h) – f(x)  6h2 for all
of A in the above definition is the derivative real h and x. Show that f(x) is constant.

A. Fill in the blanks : tan([x  ])


f(x) = is [IIT - 1981]
 2 1 1  [x]2
(x  1) sin  | x | if x  1 (A) discontinuous at some x
 (x  1)
1. Let f(x) = (B) continuous at all x, but the derivative f (x)
 1 if x  1
 does not exist for some x
be a real-valued functions. Then the set of points (C) f (x) exist for all x, but the derivative f (x does
where f(x) is not differentiable is...........[IIT - 1981] not exist for some x
2. Let f(x) = x |x|. The set of points where f(x) is twice (D) f (x) exits for all x
differentiable is............... [IIT - 1992]
4. If f(x) = x ( x  x  1) , then [IIT - 1985]
B. Multiple Choice Questions with ONE (A) f(x) is continuous but not differentiable at x = 0
correct answer : (B) f(x) is differentiable at x = 0
(C) f(x) is not differentiable at x = 0
3. For a real number y, let [y] denotes the greatest
(D) none of these
integer less than or equal to y : Then the fuction
DIFFERENTIABILITY  3.75
5. Let [x] denote the greatest integer less than or 13. The domain of the derivative of the function
equal to x. If f(x) = [x sin  x], then f(x) is  tan 1 x, if | x | 1
[IIT - 1986] 
f (x)   1 is [IIT - 2002]
(A) continuous at x = 0  2 | x | 1, if | x | 1
(B) continuous in (–1, 0)
(A) R – {0} (B) R – {1}
(C) differentiable at x = 1
(C) R – {–1} (D) R – {–1, 1}
(D) differentiable in (–1, 1)
(E) none of these 14. If f(x) is differentiable and strictly increasing
6. The set of all points where the function f(x 2 )  f(x)
x function, then the value of lim is
f(x) = is differentiable, is [IIT - 1987]
x0 f(x)  f(0)
(1 | x |) [IIT - 2004]
(A) (–) (B) [0, ) (A) 1 (B) 0
(C) (–, 0) (0, ) (D) (0, ) (C) –1 (D) 2
(E) none of these 15. The function given by y = ||x| – 1| is differentiable
7. Let f : R  R be a differentiable function and for all real number except the points [IIT - 2004]
f (x)
2t (A) {0, 1, –1} (B) ± 1
f(1) = 4. Then the value of lim
x 1 
4
x 1
dt is (C) 1 (D) –1

[IIT - 1990] 16. If f is a differentiable function and f(1/n) = 0  n  1


(A) 8f (1) (B) 4 f (1) and n , then [IIT - 2005]
(C) 2 f (1) (D) f (1) (A) f(x) = 0, x (0, 1]
8. Let [ . ] denote the greatest integer function and (B) f(0) = 0, f (0) = 0
2
f(x) = [tan x], then : [IIT - 1993] (C) f(0) = 0 = f (x), x (0, 1]
(D) f(0) = 0 and f (0) need not to be zero
(A) lim f(x) does not exist
x0
17. Let f(x) be differentiable on the interval (0, ) such
(B) f(x) is continuous at x = 0
(C) f(x) is not differentiable at x = 0 t 2 f(x)  x 2 f(t)
f(1) = 1, and lim = 1 for each x > 0.
(D) f (0) = 1 tx tx
2 2 Then f(x) is [IIT - 2007]
9. The function f(x) = (x – 1) |x – 3x + 2| + cos (|x|) is 2
Not differentiable at - [IIT - 1999] 1 2x 1 4x 2
(A)  (B) 
(A) – 1 (B) 0 3x 3 3x 3
(C) 1 (D) 2 1 2 1
(C)  (D)
10. Find left hand derivative of f(x) =[x]sin (x) at k, k x x2 x
an integer if [IIT - 2001] (x  1) n
18. Let g(x)  ; 0 < 2, m and n are
k
(A) (–1) (k – 1) (B) (–1)
k–1
(k – 1) log cosm (x  1)
k k–1 integers, m  0, n > 0 and let p be the left hand
(C) (–1) (k –1)k (D) (–1) (k – 1)k
11. Let f : R  R be a function which is defined by derivative of |x – 1| at x = 1. If lim
x  1 g(x) = p, then :
3
f(x) = max {x, x }. The set of all points where f(x) is [IIT - 2008]
not differentiable is - [IIT - 2001] (A) n = 1 n = 1 (B) n = 1, m = –1
(A) {–1, 1} (B) {–1, 0} (C) n = 2, m = 2 (D) n > 2, m = n
(C) {0, 1} (D) {–1, 0, 1}
 2 
12. Which of the following functions is differentiable  x cos , x  0
at x = 0 ? [IIT - 2001] f (x)   x ,x ,
19. Let then f is
(A) cos (|x|) + |x| (B) cos (|x|) – |x|  0, x  0

(C) sin (|x|) + |x| (D) sin (|x|) – |x|
3.76  DIFFERENTIAL CALCULUS FOR JEE MAIN AND ADVANCED
(A) differentiable both at x = 0 and at x = 2 (B) g is differentiable while f is not
(B) differentiable at x = 0 but not differentiable at (C) both f and g are differentiable
x=2
(D) g is differentiable and g’ is continuous
(C) not differentiable at x = 0 but differentiable at
x=2 26. The function f(x) = max {(1 – x), (1 + x), 2},
(D) differentiable neither at x = 0 nor at x = 2 x  (–) is [IIT - 1995]
[IIT - 2012] (A) continuous at all points
(B) differentiable at all points
C. Multiple Choice Questions with ONE or (C) differentiable at all points except at x = 1 and
MORE THAN ONE correct answer : x = –1
20. If x + |y| = 2y, then y as a function of x is (D) continuous at all points except at x = 1 and
[IIT - 1984] x = –1, where it is discontinuous
(A) defined for all real x 27. Let f : R  R be any function. Define g : R  R
(B) continuous at x – 0 by g(x) = |f(x)| for all x. Then g is - [IIT - 2000]
(C) differentiable for all x (A) onto if f is onto
dy 1 (B) one-one if f is one-one
(D) such that = for x < 0 (C) continuous if f is continuous
dx 3
21. The function f(x) = 1 + |sin x| is [IIT - 1986] (D) differentiable if f is differentiable
2 3
(A) continuous now here 28. If f(x) = min. {1, x , x }, then [IIT - 2006]
(B) continuous everywhere (A) f(x) is continuous  x  R
(C) differentiable nowhere (B) f(x) is continuous & differentiable everywhere
(D) not differentiable at x = 0 (C) f(x) is continuous but not differentiable
(E) not differentiable at infinite number of points.  xR
| x  3 |, x 1 (D) f(x) is not differentiable at two points
 2
22. The function f(x) =  x 3x 13 is  
   , x  x
4 2 4  x  , 2
[IIT - 1988]  2
 cos x, 
(A) continuous at x = 1 29. If f ( x)     x  0 , then
 x 1 2
(B) differentiable at x = 1 0  x 1

(C) continuous at x = 3  n x, x 1
(D) differentiable at x = 3
2
[IIT - 2011]
23. Let h(x) = min {x, x }, for every real number of x. 
Then [IIT - 1998] (A) f(x) is continuous at x = –
2
(A) h is continuous for all x (B) f(x) is not differentiable at x = 0
(B) h is differentiable for all x (C) f(x) is differentiable at x = 1
(C) h’(x) = 1 , for all x > 1 3
(D) h is not differentiable at two values of x’ (D) f(x) is differentiable at x = –
2
 0, x  0 30. Let f : R R be a function such that f(x + y) = f(x)
24. Let f(x) =  2 then for all x [IIT - 1994] + f(y), x, y  R. If f(x) is differentiable at x = 0,
x , x  0
(A) f  is differentiable (B) f is differentiable then [IIT - 2011]
(C) f  is continuous (D) f is continuous (A) f(x) is differentiable only in a finite interval
containing zero
25. Let g(x) = x f(x), where f(x) = xsin(1
0
/ x) , x  0
, x0
(B) f(x) is continuous x  R
(C) f (x) is constant x  R
At x = 0 [IIT - 1994] (D) f(x) is differentiable except at finitely many points
(A) g is differentiable but g’ is not continuous
DIFFERENTIABILITY  3.77
31. Let f :    be a function such that 2 3
= 2x – 3x + ,1x2
f (x  y)  f (x)  f (y),  x, y   2
If f(x) is differentialbe at x = 0, then [IIT - 2011] Discuss the continuity of f, f  and f  on [0, 2].
(A) f(x) is differentiable only in a finite interval 3 2
37. Let f(x) = x – x + x + 1 and [IIT - 1985]
containing zero g(x) = max {f(t); 0  t  x}, 0  x  1
(B) f(x) is continuous  x   =3–x 1x2
Discuss the continuity and differentiability of the
(C) f '(x) is constant  x   function g(x) in the interval (0, 2).
(D) f(X) is differentiable except at finitely many
38. Let f(x) be defined in the interval [–2, 2] such that
points.
  f(x) =1, –2  x  0

 x  2 , x   2 = x – 1, 0 < x  2
 and g(x) = f(|x|) + |f(x)|
 
32. If f (x)   cos x,   x  0 , then Test the differentiability of g(x) on x  [–2, 2]
 2
[IIT - 1986]
 x  1, 0  x  1
ln x, x  1 39. Let f(x) be a function satisfying the condition
 f(–x) = f(x) for all real x. If f’(0) exists, find its value.
[IIT - 2011] [IIT - 1987]

(A) f(x) is continuous at x   40. Draw a graph of the function y = [x] + |1 – x|,
2
(B) f(x) is not differentiable at x = 0 –1  x  3. Determine the points, if any, where this
(C) f(x) is differentiable at x = 1 function is not differentiable. [IIT - 1989]
3 41. A function R  R satisfies the equation
(D) f(x) is differentiable at x = 
2 f(x + y) = f(x) f(y) for all x, y in R and f(x) 0 for any
D. Assertion & Reasoning : x in R. Let the function be differentiable at
x = 0 and f’(0) = 2. Show that f’(x) = 2 f(x) for all x in
33. Let f and g be real valued functions defined on R . Hence, determine f(x). [IIT - 1990]
interval (–1, 1) such that g(x) is continuous,
 x  y  f(x)  f(y)
g(0) 0, f(0) = 0 and f(x) = g(x) sin x. 42. Let f  = for all real x and y. If
 2  2
Statement–1 : xlim
0
[g(x) cot x – g(0) cosec x] = f(0) f (0) exists and equals – 1 and f(0) = 1. Find f(2)
Statement–2 : f (0) = g(0) [IIT - 2008] [IIT - 1995]

E. Integer Answer Type : ì æçç- 1 + 1 ö÷÷


ï è |x| x ø ; x ¹ 0
43. Let f(x) = íxe test whether
34. If the function f(x) = x3 + ex/2 and g(x) = f–1(x), then ïî 0 ; x¹0
the value of g(1) is : [IIT - 2009] [IIT - 1997]
35. Let y'(x) + y(x) g'(x) = g(x), y(0) = 0, x   , where (a) f(x) is continuous at x = 0
df (x) (b) f(x) is differentiable at x = 0
f'(x) denotes and g(x) is a given non-
dx 44. Determine the values of x for which the following
constant differentiable function on  with g(0) = function fails to be continuous or differentiable
g(2) = 0. Then the value of y(2) is [IIT - 2011] ì1 - x ; x <1
ï
f(x) = í(1 - x)(2 - x) ; 1 £ x £ 2
F. Subjective Problems : ï3 - x
î ; x>2
x2 Justify your answer. [IIT - 1997]
36. Let f(x) = ,0x<1 [IIT - 1983]
2 45. Discuss the continuity and differentiability of the
function [IIT - 1998]
3.78  DIFFERENTIAL CALCULUS FOR JEE MAIN AND ADVANCED

ì
ï2 + 1 - x 2 ; | x |£ 1 ì x +c
ïbsin -1 ; -1/ 2 < x < 0
f(x) = í (1- x)2 ï 2
î2e
ï ; | x |> 1 ï
49. Let f(x) = í1 / 2 ; x=0
46. Let  R. Prove that a function f : R  R is ï ax /2
ï e - 1 1
differentiable at if and only if there is a function ï ; 0<x<
î x 2
g: R R which is continuous at and satisfies If f(x) be a differentiable function at x = 0, &
f(x) – f() = g(x) (x – ) for all x R. |c| < 1/2, then find the value of ‘a’ and prove that
[IIT - 2001] 64 b2 = (4 – c2). [IIT - 2004]

47. Let f(x) = |xxa1|ififxx00 , and


50. If f(x – y) = f(x) . g(y) – f(y) . g(x) and
g(x – y) = g(x) . g(y) + f(x) . f(y) for all x, y  R.
If right hand derivative at x = 0 exists for f(x). Find
 x  1 if x  0
g(x) =  2 , where a and b are derivative of g(x) at x = 0 [IIT - 2005]
(x  1)  b if x  0
non-negative real numbers Determine the G. Match the Columns
composite function g of. If (g o f) (x) is continuous 51. Column I Column II
for all real x, determine the values of a and b. (A) x |x| (P) continuous in (–1, 1)
Further, for these value of a and b, is gof
differentiable at x = 0 ? Justify your answer. (B) |x| (Q) differentiable in (–1, 1)
[IIT - 2002] (C) x + [x] (R) strictly increasing in
48. If a function f : [– 2a, 2a]  R, is an odd function (–1, 1)
such that f(x) = f(2a – x) for x  [a, 2a] and left (D) |x – 1| + |x + 1| (S) not differentiable at
hand derivative at x = a is 0, then find the left hand least at one point in
derivative at x = – a. [IIT - 2003] (–1, 1)

CONCEPT PROBLEMS—A 14. 0


1. f (0+) = – 1 ; f (0–) = 1; non-differentiable 15. 0
2. m = – 1, n =  CONCEPT PROBLEMS—B
3. f (1+) = – 1; f (1–) = 3 ln 3 hence f is not 1. (a) –1, 2 (b) 1, 3
differentiable at x = 1 2. (a) 0 (b) 1, 3
4. (b) (2a–1/3)/3 3. (a) 5 (b) 2, 3
5. yes; y = 1. 4. – 4 (discontinuity), – 1 (corner), 2 (discontinuity),
6. non-differentiable at x = 0 5 (vertical tangent).
PRACTICE PROBLEMS—A
7. – h; 0.
2 –2
8. (a) –1,1 (b) ,
a a 5. No
(c) 1, 0 (d) 0, 0.
9. f(x) is not differentiable at x = 0
DIFFERENTIABILITY  3.79
7. a = 2, b = 0 14. a = 35/9, b = 10/3
8. continuous and differentiable. 15. Non differentiable.
9. cont. but not diff.at x = 0 ; diff. & cont. at x = /2 16.
10. a = 1, b = 1
11. discontinuous and non-differentiable at x = 1.
PRACTICE PROBLEMS—B
12. continuous; diff at x = 1, non diff. at x = 0
13. continuous but non diff at x = 0
14. a=2
15. diff.
17. f is cont. but not diff. at x = 1, discont. at x = 2 &
16. discontinuous & not derivable at x = 1, x = 3. cont.& diff. at all other points
continuous but not derivable at x = 2
18. 3 ; The bold line represents the graph of y = f(x).
18. discontinuous at x = 1 and hence non-
differentiable.
CONCEPT PROBLEMS—C
1. (a) x = – 3, 1 (b) x = ± 1.
2. yes
3. (a) –1, 1
(b) x = 10/3, L.H.D. = –3, R.H.D. = 3
5. f (o) does not f  (o–) = 0 f (o+) It is non-differentiable at three points namely
6. f(x) is cont. but not diff. at x = 0   5
7. (a) x , , .
4 2 3
19. g o f (x) = 0 for all x  R and
0 if x is an int eger
 
f o g =  n if x  R and n  x  n  1 or
 
 x  R and  n  1  x   n
g o f is differentiable everywhere and f o g is
differentiable on R – I.
(b) All x (c) f (x) = 2|x|
 cos x , 0x
8. discontinuous at x = 0, 1, 2, 3 hence non- 20. g(x)  
differentiable  sin x  1 , x  
9. The Dirichlet function is nowhere continuous, and Adjacent figure represents the graph of g(x).
hence it is nowhere derivable. Clearly, g(x) is continuous but non-differentiable
10. diff every where. at x =.

PRACTICE PROBLEMS—C
11. continuous but non-differentiable.
12. differentiable.
13. x = 0, ± 1
3.80  DIFFERENTIAL CALCULUS FOR JEE MAIN AND ADVANCED
CONCEPT PROBLEMS—D PRACTICE PROBLEMS—E
1. 0 6. Yes
2. f(x) = x6 , a = 2 7. No
3. (i) f(x) = 2x, a = 5 8. (b) f is twice but not thrice differentiable at x = x0
(ii) f(x) = cos x, a =  or f(x) = cos ( + x), a = 0 and the derivatives are 0.
9. They are of opposite signs.
4. The only doubtful point is 0. But |f(x)/x| = |x|  0
10. non-differentiable.
as x  0, so f (0) = 0. For x > 0, f (x) = 2x and for
11. Cont. but non diff. at x = 1, discont. and non diff.
x < 0, f (x) = –2x. therefore f (x) = 2|x| is continuous.
at x = 2, 3
5. 1 12. non diff. at x = , 2
6. 2/3 13. p>1
7. non diff. at x = 0 14. non-diff at x = 2, 3 if a  4, 9; non diff at x = 3 if
8. a = 6, b = –3. a = 4; non-diff at x = 2 if a = 9.

CONCEPT PROBLEMS—E PRACTICE PROBLEMS—F


2. (a) x for x  0, – x for x  0 2. x, |x|
(b) Yes 3. –1
(c) 1 for x > 0, – 1 for x < 0 4. 0
4 x1 / 3 5. 2x
3. (a) (b) Yes
3 1
4 7. , n2
(c) 2
9x 2 / 3 2 x
8. x + x e
4. (a) g(x) = 2x for x  1, g(x) = 2 for x > 1
10. f(x) = 2x
g"(x) = 2 for x  1, g"(x) = 0 for x > 1
(b) g is differentiable ; g is not
5. – sin a OBJECTIVE EXERCISES
PRACTICE PROBLEMS—D 1. C 2. B 3. A
6. (a) y" = 6 |x|; y"(0) = 0. 4. A 5. B 6. B
7. D 8. D 9. B
(b) y" = 2 sin 1 – 2 cos 1 – 12 sin 1 at x  10. B 11. A 12. D
x x x x x
13. A 14. A 15. A
0, y"(0) does not exist, since y  (x) is 16. B 17. A 18. D
discontinuous at x = 0. 19. A 20. B 21. C
x
8. 2sec x tan x 22. C 23. B 24. D
9. n–1<k<n 25. C 26. B 27. A
28. C 29. C 30. C
CONCEPT PROBLEMS—F 31. D 32. A 33. C
1. (a) Yes (b) No 34. C 35. C 36. A
2. No 37. D 38. D 39. A
3. (a) No (b) No 40. A 41. C 42. D
43. A 44. C 45. A
4. If g(x0)  0 then f is non-diff. as x = x 0. If 46. B 47. D 48. D
g(x0) = 0 then f is differentiable is x = x0. 49. A 50. C 51. A,C
DIFFERENTIABILITY  3.81
52. A,D 53. A,B,C 54. B,D  continuous but not derivable.
55. A,C 56. A,B,C 57. A,B,D 18. non diff. at x = 1
58. B,D 59. A,B,C 60. B,C,D
61. B,D 62. A,B,D 63. A,B,C,D 19. g(0), 0, g(4), g(2), g(– 2)
64. A,C,D 65. A,B,D 66. A,B 20. f(x) is non differentiable at x = , , 0, ,  and
67. A,B 68. A,D 69. C,D g(x) is non differentiable at x = , , 0, – 2, 2
70. B,C 71. A 72. A
73. D 74. A 75. D
76. A 77. D 78. B
79. C 80. B 81. A
82. C 83. B 84. D
85. A 86. A 87. B
88. D 89. B 90. D  x
 if x  0, x  1
91. C 92. D 93. C 21. f(x) =  1  x
94. C 95. B x
 if x  0, x   1
96. (A)–(P,Q), (B)–(S), (C)–(Q), (D)–(R)  x
1 
97. (A)–R,(B)–S, (C)–P, (D)–Q
 1
98. (A)–(P), (B)–(R), (C)–(Q), (D)–(T)  if x  0, x  1
99. (A)–P; (B)–(P, Q), (C)–(P, R), (D)–(R) 1  x  2

and f (x) = 
100. (A)–(P), B–(R), (C)–(S), (D)–(P)  1
 if x  0, x   1
 1  x
2

REVIEW EXERCISES for JEE ADVANCED


1. continuous but non-differentiable.
2. g(x) = x
3. discont and non-diff at x = 0, 2; cont but non-diff
at x = ± 1.
4. discont and non-diff at x = 0; cont but non-diff at
x=  2
5. {1, 3} 23. f (1 )  3, f (1 )  1
6. non-diff at x = 2, 4
25. (a) 2 (b) 1
7. discontinuous and non-differentiable at x = 2. 2
26. f(x) is not cont. not diff, xf(x) is cont. diff. x f(x) is
8. a = – 1, b = – 2, value = 2/3. cont and diff.
9. If f(x0) = g(x0), then the two functions may or may 27. cont but not diff at x = 1, discont. at x = 2, 3
not be differentiable at x = 0.
28. a  1, b = 0, p = 1/3, q = – 1
12. f is cont. but not derivable at x = x0.
29. cont in 0  x  1, but not diff. at x = 0
14. continuous but non-differentiable at x = 0. 2 2
30. D*[f(x). g(x)] = f (x) D*g(x) + g (x). D*f(x);
15. 180
f(x) g 2 (x).D * f(x)  f 2 (x).D * g(x)
16. a (0, 1] D* =
g(x) [g(x)]4
17. If a  (0, 1),f  (0+) = 1,f  (0) = 1
31. x + 1
 continuous but not derivable ; If a = 1, f (x) = 0
32. f(x) = 0
which is constant  continuous and derivable;
33. f(x) = ln x
If a > 1, f (0) =  1, f (0+) = 1
3.82  DIFFERENTIAL CALCULUS FOR JEE MAIN AND ADVANCED
34. f(x) = 1, f(x) = 0.
1 x e 2x  1
35. f(x) = e–x – 1 33. f(x) = . 34. f(x) = .
1 x e2x  1

TARGET EXERCISES for JEE ADVANCED


PREVIOUS YEAR'S QUESTIONS
1. Continuous but non-differentiable at x = –2, –1,
0, 1, 2
(JEE ADVANCED)
2. not differentiable at x = e; f (e+) = , f (e–) = 0. 1. {0} 2. R – {0}
–1
4. f(x) = 0, f(x) = 1/2, f(x) = x. 5. f(x) = sin x) 3. D 4. B
6. continuous but non-differentiable at x = 1. 5. ABD 6. A
7. f is continuous and differentiable for all
7. A 8. B
x  domain , except non-differentiable at x = 1
10. 256 9. D 10. B
11. 54 11. D 12. D
13. R – {–1, 1} 13. D 14. C
15. A 16. B
17. A 18. C
19. B 20. ABD
21. BDE 22. ABC
23. ACD 24. BCD
25. AB 26. AC
14. (i) A and B (ii) Greater 27. C 28. AC
29. ABCD 30. BC
(iii) (iv) B and C, D and E 31. BC 32. ABCD
33. B 34. 2
16. g(x) is continuous in [–3, 1] 35. 0
17. (a) continuous for all x  R not differentiable for 36. f and f  are continuous and f  is discontinuous on
all x  R  {0} [0, 2]
(b) g(x) is not continuous at x = 0 37. cont. on (0, 2) and differentiable on (0, 2) = {1}
2 38. not differentiable at x = 0, 1
22. 1 + x 24. {±1, 0, 1 – 2 })
39. f (0) = 0 40. x = 0, 1, 2, 3
n2 2x
25. 26. Non - diff. at x = , 41. f(x) = e 42. – 1
2
43. cont. but not diff.
2
44. f(x) is not continuous and thus not differentiable
27. x + 1
at x = 2
28. For g(t) = mt and h(t) = t, limitt  0 g(t)/(h(t) = m,
45. continuous but not differentiable at discontinuous
which need not be zero.
and not differentiable at x = –1.
30. f(x) = x3 31. f(x) = 0
DIFFERENTIABILITY  3.83
x  a  1 if x  a 48. 0
(x  a  1)2  b if a  x  0
 49. a=1
47. g(f(x)) =  2 a = 1, b = 0,
x  b if 0  x  1 50. 0
(x  2)  b (A)(P, Q, R), (B)(P, S), (C)(R, S), (D)(P, Q)
2
if x  1 51.
gof differentiable at x = 0

You might also like